MedSurg II Exam 1 (39, 40, 41, 42, 43, 44)

Lakukan tugas rumah & ujian kamu dengan baik sekarang menggunakan Quizwiz!

Which assessment data should the nurse expect to find for the client who had an upper gastrointestinal (UGI) series? a. Chalky white stools. b. Increased heart rate. c. A firm hard abdomen. d. Hyperactive bowel sounds.

**a. A UGI requires the client to swallow barium, which passes through the intestines, making the stools a chalky white color. b. This would be abnormal data and would be cause for further assessment. c. This would not be expected from the test. d. This is a serious finding and should be treated.

Which medication should the nurse question before administering to the client with peptic ulcer disease? a. E-mycin, an antibiotic. b. Prilosec, a proton pump inhibitor. c. Flagyl, an antimicrobial agent. d. Tylenol, a nonnarcotic analgesic

**a. E-mycin is irritating to stomach, and its use in a client with peptic ulcer disease should be questioned. b. Prilosec, a proton pump inhibitor, decreases gastric acid production and its use should not be questioned by the nurse. c. Flagyl, an antimicrobial, is given to treat peptic ulcer disease secondary to H. pylori bacteria. d. Tylenol can be safely administered to a client with peptic ulcer disease.

The nurse is caring for a client diagnosed with rule-out peptic ulcer disease. Which test confirms this diagnosis? a. EGD b. MRI c. Occult blood test d. Gastric acid stimulation

**a. The EGD is an invasive diagnostic test that visualizes the esophagus and stomach to accurately diagnose an ulcer and evaluate the effectiveness of the client's treatment. b. MRIs show cross-sectional images of tissue or blood flow. c. An occult blood test shows the presence of blood, but not the source. d. A gastric acid stimulation test is used to understand the pathophysiology of ulcer disease, but it has limited usefulness.

The client who has had an abdominal surgery has a Jackson Pratt (JP) drainage tube. Which assessment data would warrant immediate intervention by the nurse? a. The bulb is round and has 40 mL of fluid. b. The drainage tube is pinned to the dressing. c. The JP insertion site is pink and has no drainage. d. The JP bulb has suction and is sunken in.

**a. The JP bulb should be depressed, which indicates suction is being applied. A round bulb indicates that the bulb is full and needs to be emptied and suction reapplied. b. The tube should be pinned to the dressing to prevent accidentally pulling the drain out of the insertion site. c. The insertion site should be pink and without any signs of infection, which include drainage, warmth, and redness. d. The JP bulb should be sunken in or depressed, indicating that suction is being applied.

The client is in end-stage liver failure and has vitamin K deficiency. Which interventions should the nurse implement? Select all that apply. a. Avoid rectal temperatures. b. Use only a soft toothbrush. c. Monitor the platelet count. d. Use small-gauge needles. e. Assess for asterixis.

**a. Vitamin K deficiency causes impaired coagulation; therefore rectal thermometers should be avoided to prevent bleeding. **b. Soft toothbrushes will help prevent bleeding of the gums. **c. Platelet count, PTT/PT, and INR should be monitored to assess coagulation status. **d. Injections should be avoided, if at all possible, because the client is unable to clot, but if they are absolutely necessarily, the nurse should use small-gauge needles. e. Asterixis is a flapping tremor of the hands when the arms are extended and indicates an elevated ammonia level, but it is not associated with vitamin K deficiency.

Which method is preferred for immediate treatment of an acute episode of constipation? a. An enema b. Increased fluid c. Stool softeners d. Bulk-forming medication

a. An enema Enemas are fast acting and beneficial in the immediate treatment of acute constipation but should be limited in their use. Bulk-forming medication stimulates peristalsis but takes 24 hours to act. Stool softeners have a prolonged action, taking up to 72 hours for an effect, and fluids can help to decrease the incidence of constipation

The client has been experiencing difficulty and straining when expelling feces. Which intervention should be taught to the client? a. Explain that some blood in the stool will be normal for the client. b. Instruct the client in manual removal of feces. c. Encourage the client to use a cathartic laxative on a daily basis. d. Place the client on a high-residue diet.

a. Blood may indicate a hemorrhoid, but it is not normal to expel blood when having a bowel movement. b. Nurses manually remove feces; it is not a self-care activity. c. Cathartic use on a daily basis creates dependence and a narrowing of the lumen of the colon, creating a much more serious problem. **d. A high-residue diet provides bulk for the colon to use in removing the waste products of metabolism. Bulk laxatives and fiber from vegetables and bran assist the colon to work more effectively.

A nurse is teaching a client who has GERD. Which of the following should the client be instructed to limit in his diet? (Select all that apply) a. Coffee b. Tomatoes c. Bananas d. Chocolate e. Pasta

a. Coffee b. Tomatoes d. Chocolate

When obtaining a nursing history from the patient with colorectal cancer, the nurse should specifically ask the patient about a. Dietary intake. b. Sports involvement. c. Environmental exposure to carcinogens. d. Long-term use of nonsteroidal antiinflammatory drugs (NSAIDs).

a. Dietary intake. A diet high in red meat and low fruit and vegetable intake is associated with development of colorectal cancer (CRC), as are alcohol intake and smoking. Family and personal history of CRC also increases the risk. Other environmental agents are not known to be related to colorectal cancer. Long-term use of nonsteroidal antiinflammatory drugs (NSAIDs) is associated with reduced CRC risk.

A nurse is planning care for a client who has acute gastritis. Which of the following nursing interventions should the nurse include in the plan of care? (Select all that apply) a. Evaluate intake and output b. Monitor laboratory reports of electrolytes c. Provide three large meals a day d. Administer ibuprofen for pain e. Observe stool characteristics

a. Evaluate intake and output b. Monitor laboratory reports of electrolytes e. Observe stool characteristics

A stroke patient who primarily uses a wheelchair for mobility has diarrhea with fecal incontinence. What should the nurse assess first? a. Fecal impaction b. Perineal hygiene c. Dietary fiber intake d. Antidiarrheal agent use

a. Fecal impaction

The nurse is admitting a male client to a medical floor with a diagnosis of adenocarcinoma of the rectosigmoid colon. Which assessment data support this diagnosis? a. The client reports up to 20 bloody stools per day. b. The client states that he has a feeling of fullness after a heavy meal. c. The client has diarrhea alternating with constipation. d. The client complains of right lower quadrant pain with rebound tenderness.

a. Frequent bloody stools are a symptom of inflammatory bowel disease (IBD). IBD is a risk factor for cancer of the colon, but the symptoms are different when the colon becomes cancerous. b. Most people have a feeling of fullness after a heavy meal; this does not indicate cancer. **c. The most common symptom of colon cancer is a change in bowel habits, specifically diarrhea alternating with constipation. d. Lower right quadrant pain with rebound tenderness would indicate appendicitis.

The client diagnosed with ulcerative colitis is prescribed a low-residue diet. Which meal selection indicates the client understands the diet teaching? a. Grilled hamburger on a wheat bun and fried potatoes b A chicken salad sandwich and lettuce and tomato salad c. Roast pork, white rice, and plain custard d. Fried fish, whole grain pasta and fruit salad

a. Fried potatoes, along with pastries and pies, should be avoided. b. Raw vegetables should be avoided because this is roughage. **c. A low-residue diet is a low-fiber diet. Products made of refined flour or finely milled grains, along with roasted, baked, or broiled meats, are recommended. d. Fried foods should be avoided, and whole grain is high in fiber. Nuts and fruits with peels should be avoided.

A patient had a stomach resection for stomach cancer. The nurse should teach the patient about the loss of the hormone that stimulates gastric acid secretion and motility and maintains lower esophageal sphincter tone. Which hormone will be decreased with a gastric resection? a. Gastrin b. Secretin c. Cholecystokinin d. Gastric inhibitory peptide

a. Gastrin

Which GI assessment data should the nurse expect to find when assessing the client in end-stage liver failure? a. Hypoalbuminemia and muscle wasting. b. Oligomenorrhea and decreased body hair. c. Clay-colored stools and hemorrhoids. d. Dyspnea and caput medusae.

a. Hypoalbuminemia, decreased albumin, and muscle wasting are metabolic effects, not gastrointestinal effects. b. Oligomenorrhea is no menses, which is a reproductive effect, and decreased body hair is an integumentary effect. **c. Clay-colored stools and hemorrhoids are gastrointestinal effects of liver failure. d. Dyspnea is a respiratory effect, and caput medusae (dilated veins around the umbilicus) is an integumentary effect, although it is on the abdomen.

Two days following a colectomy for an abdominal mass, a patient reports gas pains and abdominal distention. The nurse plans care for the patient based on the knowledge that the symptoms are occurring as a result of a. Impaired peristalsis. b. Irritation of the bowel. c. Nasogastric suctioning. d. Inflammation of the incision site.

a. Impaired peristalsis.

The nurse is preparing a client diagnosed with GERD for surgery. Which information warrants notifying the HCP? a. The clients bernstein esophageal test was positive b. The abdominal x-ray shows a hiatal hernia c. The clients WBC count is 14,000 d. The clients hgb is 13.8

a. In a Bernstein's test, acid is instilled into the distal esophagus; this causes immediate heartburn for a client diagnosed with GERD. This would not warrant notifying the HCP. b. Hiatal hernias are frequently the cause of GERD; therefore, this finding would not warrant notifying the HCP. **c. The client's WBC is elevated, indicating a possible infection, which warrants notifying the HCP. d. This is a normal hemoglobin result and would not warrant notifying the HCP.

The emergency department nurse knows the client diagnosed with acute gastroenteritis understands the discharge teaching when the client makes which statement? a. "I will probably have some leg cramps while I have gastroenteritis." b. "I should decrease my fluid intake until the diarrhea subsides." c. "I should reintroduce solid foods very slowly into my diet." d. "I should only drink bottled water until the abdominal cramping stops."

a. Leg cramps could indicate hypokalemia, which is a potential complication of excessive diarrhea and should be reported to the health-care provider. b. The client should increase the fluid intake because oral rehydration is the primary treatment for gastroenteritis to replace lost fluid as a result of diarrhea and to prevent dehydration. **c. Reintroducing solid foods slowly, in small amounts, will allow the bowel to rest and the mucosa to return to health after acute gastroenteritis states. d. Bottled water should be consumed when contaminated water is suspected, and an oral glucose-electrolyte solution, such as Gatorade or Pedialyte, should be recommended.

The nurse performs a detailed assessment of the abdomen of a patient with a possible bowel obstruction, knowing that manifestations of an obstruction in the large intestine are (select all that apply) a. Persistent abdominal pain. b. Marked abdominal distention. c. Diarrhea that is loose or liquid. d. Colicky, severe, intermittent pain. e. Profuse vomiting that relieves abdominal pain.

a. Persistent abdominal pain. b. Marked abdominal distention

Assessment findings suggestive of peritonitis include a. Rebound abdominal pain. b. A soft, distended abdomen. c. Dull, continuous abdominal pain. d. Observing that the patient is restless.

a. Rebound abdominal pain.

The nurse is caring for a 55-year-old man patient with acute pancreatitis resulting from gallstones. Which clinical manifestation would the nurse expect the patient to exhibit? a. Hematochezia b. Left upper abdominal pain c. Ascites and peripheral edema d. Temperature over 102 F (38.9 C)

b. Left upper abdominal pain

A nurse is completing an assessment of a client who has GERD. Which of the following is an expected finding? a. Absence of saliva b. Loss of tooth enamel c. Client reports sweet taste in mouth d. Client reports absence of eructation

b. Loss of tooth enamel

Several patients are seen at an urgent care center with symptoms of nausea, vomiting, and diarrhea that began 2 hours ago while attending a large family reunion potluck dinner. You question the patients specifically about foods they ingested containing a. Beef. b. Meat and milk. c. Poultry and eggs. d. Home-preserved vegetables.

b. Meat and milk.

The nurse should administer an as-needed dose of magnesium hydroxide (MOM) after noting what information while reviewing a patient's medical record? a. Abdominal pain and bloating b. No bowel movement for 3 days c. A decrease in appetite by 50% over 24 hours d. Muscle tremors and other signs of hypomagnesemia

b. No bowel movement for 3 days

A nurse is caring for a client who has chronic gastritis and is scheduled for a selective vagotomy. The purpose of this procedure is to a. Increase duodenal gastric emptying b. Reduce gastric acid secretions c. Increase gastric mucus secretion d. Reduce histamine secretion

b. Reduce gastric acid secretions

A nurse is caring for an older adult client in an extended care facility. Which of the following indicates the client has a stool impaction causing a large intestine obstruction? a. The client reports he had a bowel movement yesterday b. The client is having small, frequent liquid stools c. The client is flatulent d. The client indicates he vomited once this morning

b. The client is having small, frequent liquid stools

The patient has peritonitis, which is a major complication of appendicitis. What treatment will the nurse plan to include? a. Peritoneal lavage b. Peritoneal dialysis c. IV fluid replacement d. Increased oral fluid intake

c. IV fluid replacement IV fluid replacement along with antibiotics, NG suction, analgesics, and surgery would be expected. Peritoneal lavage may be used to determine abdominal trauma. Peritoneal dialysis would not be performed. Oral fluids would be avoided with peritonitis.

The patient receiving chemotherapy rings the call bell and reports the onset of nausea. The nurse should prepare an as-needed dose of which medication? a. Morphine sulfate b. Zolpidem (Ambien) c. Ondansetron (Zofran) d. Dexamethasone (Decadron)

c. Ondansetron (Zofran)

In developing an effective weight reduction plan for an overweight patient who states a willingness to try to lose weight, it is most important for the nurse to first assess which factor? a. The length of time the patient has been obese b. The patient's current level of physical activity c. The patient's social, emotional, and behavioral influences on obesity d. Anthropometric measurements, such as body mass index and skinfold thickness

c. The patient's social, emotional, and behavioral influences on obesity

Match the descriptions with the following surgical procedures used to treat peptic ulcer disease. a. Often performed with a vagotomy to increase gastric emptying b. Severing of a parasympathetic nerve to decrease gastric secretion c. Removal of distal two thirds of stomach with anastomosis to jejunum d. Removal of distal two thirds of stomach with anastomosis to duodenum 1. Billroth I 2. Billroth II 3. Pyloroplasty 4. Vagotomy

a. 3 b. 4 c. 2 d. 1

A patient with type 2 diabetes and cirrhosis asks the nurse if it would be okay to take silymarin (milk thistle) to help minimize liver damage. The nurse responds based on what knowledge? a. Milk thistle may affect liver enzymes and thus alter drug metabolism. b. Milk thistle is generally safe in recommended doses for up to 10 years. c. There is unclear scientific evidence for the use of milk thistle in treating cirrhosis. d. Milk thistle may elevate the serum glucose levels and is thus contraindicated in diabetes.

a. Milk thistle may affect liver enzymes and thus alter drug metabolism.

The nurse identifies that which patient is at highest risk for developing colon cancer? a. A 28-year-old male who has a body mass index of 27 kg/m2 b. A 32-year-old female with a 12-year history of ulcerative colitis c. A 52-year-old male who has followed a vegetarian diet for 24 years d. A 58-year-old female taking prescribed estrogen replacement therapy

b. A 32-year-old female with a 12-year history of ulcerative colitis

The patient with right upper quadrant abdominal pain has an abdominal ultrasound that reveals cholelithiasis. What should the nurse expect to do for this patient? a. Prevent all oral intake. b. Control abdominal pain. c. Provide enteral feedings. d. Avoid dietary cholesterol.

b. Control abdominal pain.

The nurse should recognize that the liver performs which functions? Select all that apply. a. Bile storage b. Detoxification c. Protein metabolism d. Steroid metabolism e. Red blood cell (RBC) destruction

b. Detoxification c. Protein metabolism d. Steroid metabolism

After administering a dose of promethazine (Phenergan) to a patient with nausea and vomiting, what common temporary adverse effect of the medication does the nurse explain may be experienced? a. Tinnitus b. Drowsiness c. Reduced hearing d. Sensation of falling

b. Drowsiness

A patient is seeking emergency care after choking on a piece of steak. The nursing assessment reveals a history of alcoholism, cigarette smoking, and hemoptysis. Which diagnostic study is most likely to be performed on this patient? a. Barium swallow b. Endoscopic biopsy c. Capsule endoscopy d. Endoscopic ultrasonography

b. Endoscopic biopsy

The health care team is assessing a male patient for acute pancreatitis after he presented to the emergency department with severe abdominal pain. Which laboratory value is the best diagnostic indicator of acute pancreatitis? a. Gastric pH b. Blood glucose c. Serum amylase d. Serum potassium

c. Serum amylase

A patient has been told that she has elevated liver enzymes caused by nonalcoholic fatty liver disease (NAFLD). The nursing teaching plan should include a. Having genetic testing done. b. Recommending a heart-healthy diet. c. The necessity to reduce weight rapidly. d. Avoiding alcohol until liver enzymes return to normal.

b. Recommending a heart-healthy diet.

Teaching in relation to home management after a laparoscopic cholecystectomy should include a. Keeping the bandages on the puncture sites for 48 hours. b. Reporting any bile-colored drainage or pus from any incision. c. Using over-the-counter antiemetics if nausea and vomiting occur. d. Emptying and measuring the contents of the bile bag from the T tube every day.

b. Reporting any bile-colored drainage or pus from any incision.

A patient who is scheduled for surgery with general anesthesia in 1 hour is observed with a moist, but empty water glass in his hand. Which assessment finding may indicate that the patient drank a glass of water? a. Flat abdomen without movement upon inspection b. Tenderness at left upper quadrant upon palpation c. Easily heard, loud gurgling in the right upper quadrant d. High-pitched, hollow sounds in the left upper quadrant

c. Easily heard, loud gurgling in the right upper quadrant

What does the nurse include when teaching a patient with newly diagnosed peptic ulcer disease? a. Maintain a bland, soft, low-residue diet. b. Use alcohol and caffeine in moderation and always with food. c. Eat as normally as possible, eliminating foods that cause pain or discomfort. d. Avoid milk and milk products because they stimulate gastric acid production.

c. Eat as normally as possible, eliminating foods that cause pain or discomfort. There is no specific diet used for the treatment of peptic ulcers and patients are encouraged to eat as normally as possible, eliminating foods that cause discomfort or pain. Eating six meals a day prevents the stomach from being totally empty and is also recommended. Caffeine and alcohol should be eliminated from the diet because they are known to cause gastric irritation. Milk and milk products do not need to be avoided but they can add fat content to the diet.

A patient with esophageal cancer is scheduled for a partial esophagectomy. Which nursing intervention is likely to be of highest priority preoperatively? a. Practice turning and deep breathing. b. Brush the teeth and mouth well each day. c. Encourage a high-calorie, high-protein diet. d. Teach about postoperative tubes and cares.

c. Encourage a high-calorie, high-protein diet. Eating a high-calorie, high-protein diet, perhaps in liquid form, is the highest priority preoperatively. Because of dysphagia, the patient frequently has poor nutritional status because of the inability to ingest adequate amounts of food before surgery. An esophageal stent may be placed to improve the nutritional status. Turning and deep breathing will be done and the patient will need to know about postoperative care but these are not the priorities. Meticulous oral care is done but it may be done with swabs or gauze pads to prevent the injury and pain brushing may incur.

The nurse determines that a patient has experienced the beneficial effects of therapy with famotidine (Pepcid) when which symptom is relieved? a. Nausea b. Belching c. Epigastric pain d. Difficulty swallowing

c. Epigastric pain

A 20-year-old man is admitted to the emergency department after a motor vehicle crash with suspected abdominal trauma. What assessment finding by the nurse is of highest priority? a. Nausea and vomiting b. Hyperactive bowel sounds c. Firmly distended abdomen d. Abrasions on all extremities

c. Firmly distended abdomen

A patient with stage I colorectal cancer is scheduled for surgery. Patient teaching for this patient would include an explanation that a. Chemotherapy will begin after the patient recovers from the surgery. b. Both chemotherapy and radiation can be used as palliative treatments. c. Follow-up colonoscopies will be needed to ensure that the cancer does not recur. d. A wound, ostomy, and continence nurse will visit the patient to identify an abdominal site for the ostomy.

c. Follow-up colonoscopies will be needed to ensure that the cancer does not recur.

A patient complains of nausea. When administering a dose of metoclopramide (Reglan), the nurse should teach the patient to report which potential adverse effect? a. Tremors b. Constipation c. Double vision d. Numbness in fingers and toes

a. Tremors

The patient is an older woman with cirrhosis who also has anemia. What pathophysiologic changes may contribute to this patient's anemia? (Select all that apply) a. Vitamin B deficiencies b. Stretching of liver capsule c. Vascular congestion of spleen d. Decreased prothrombin production e. Decreased bilirubin conjugation and excretion

a. Vitamin B deficiencies b. Stretching of liver capsule d. Decreased prothrombin production The anemia of cirrhosis is related to overactivity of the enlarged spleen that removes blood cells from circulation. Vitamin B deficiencies from altered intake and metabolism of nutrients and decreased prothrombin production can increase bleeding tendencies. The other options do not contribute to anemia in the patient with cirrhosis.

The nurse has completed initial instruction with a patient regarding a weight loss program. The nurse determines that the teaching has been effective when the patient makes which statement? a. "I plan to lose 4 lb a week until I have lost the 60-pound goal." b. "I will keep a diary of weekly weights to illustrate my weight loss." c. "I will restrict my carbohydrate intake to less than 30 g/day to maximize weight loss." d. "I should not exercise more than my program requires since increased activity increases the appetite."

b. "I will keep a diary of weekly weights to illustrate my weight loss."

The nurse cares for a postoperative patient who has just vomited yellow green liquid and reports nausea. Which action would be an appropriate nursing intervention? a. Offer the patient a herbal supplement such as ginseng. b. Apply a cool washcloth to the forehead and provide mouth care. c. Take the patient for a walk in the hallway to promote peristalsis. d. Discontinue any medications that may cause nausea or vomiting.

b. Apply a cool washcloth to the forehead and provide mouth care.

Inspection of an older patient's mouth reveals the presence of white, curd-like lesions on the patient's tongue. What is the most likely etiology for this abnormal assessment finding? a. Herpesvirus b. Candida albicans c. Vitamin deficiency d. Irritation from ill-fitting dentures

b. Candida albicans

A nurse is completing discharge teaching to a client who has Crohn's disease. Which of the following should be included in the teaching? a. Decrease intake of calorie-dense foods b. Drink canned protein supplements c. Take calcium supplements daily d. Take a bulk-forming laxative daily

b. Drink canned protein supplements

The patient being treated with diuretics for ascites from cirrhosis must be monitored for (select all that apply)? a. GI bleeding b. Hypokalemia c. Renal function d. Body image disturbances e. Increased clotting tendencies

b. Hypokalemia c. Renal function With diuretic therapy, fluid and electrolyte balance must be monitored; serum levels of sodium, potassium, chloride, and bicarbonate must be monitored, especially hypokalemia. Renal function must be monitored with blood urea nitrogen and serum creatinine. Water excess is manifested by muscle cramping, weakness, lethargy, and confusion. GI bleeding, body image disturbances, and bleeding tendencies seen with cirrhosis are not related to diuretic therapy.

The patient with advanced cirrhosis asks why his abdomen is so swollen. The nurse's response is based on the knowledge that a. A lack of clotting factors promotes the collection of blood in the abdominal cavity. b. Portal hypertension and hypoalbuminemia cause a fluid shift into the peritoneal space. c. Decreased peristalsis in the GI tract contributes to gas formation and distention of the bowel. d. Bile salts in the blood irritate the peritoneal membranes, causing edema and pocketing of fluid.

b. Portal hypertension and hypoalbuminemia cause a fluid shift into the peritoneal space.

Which infection or inflammation is found related to systemic disease and cancer chemotherapy? a. Parotitis b. Stomatitis c. Oral candidiasis d. Vincent's infection

b. Stomatitis Stomatitis is inflammation of the mouth related to systemic diseases and cancer chemotherapy medications. There is excessive salivation, halitosis, and a sore mouth. Parotitis is a Staphylococcus infection that may occur with prolonged NPO status and results in decreased saliva and ear pain. Oral candidiasis is seen with prolonged antibiotic or corticosteroid therapy; it has white membranous lesions on the mucosa of the mouth and larynx. Vincent's infection is a bacterial infection predisposed by fatigue, stress, and poor oral hygiene. There are painful, bleeding gums and increased metallic-tasting saliva.

An 85-year-old woman seen in the primary care provider's office for a well check complains of difficulty swallowing. What common effect of aging should the nurse assess for as a possible cause? a. Anosmia b. Xerostomia c. Hypochlorhydria d. Salivary gland tumor

b. Xerostomia

During a routine screening colonoscopy on a 56-year-old patient, a rectosigmoidal polyp was identified and removed. The patient asks the nurse if his risk for colon cancer is increased because of the polyp. What is the best response by the nurse? a. "It is very rare for polyps to become malignant but you should continue to have routine colonoscopies." b. "Individuals with polyps have a 100% lifetime risk of developing colorectal cancer and at an earlier age than those without polyps." c. "All polyps are abnormal and should be removed but the risk for cancer depends on the type and if malignant changes are present." d. "All polyps are premalignant and a source of most colon cancer. You will need to have a colonoscopy every 6 months to check for new polyps."

c. "All polyps are abnormal and should be removed but the risk for cancer depends on the type and if malignant changes are present." Although all polyps are abnormal growths, the most common type of polyp (hyperplastic) is non-neoplastic, as are inflammatory, lipomas, and juvenile polyps. However, adenomatous polyps are characterized by neoplastic changes in the epithelium and most colorectal cancers appear to arise from these polyps. Only patients with a family history of familial adenomatous polyposis (FAP) have close to a 100% lifetime risk of developing colorectal cancer.

A 58-year-old woman is being discharged home today after ostomy surgery for colon cancer. The nurse should assign the patient to which staff member? a. A nursing assistant on the unit who also has hospice experience b. A licensed practical nurse who has worked on the unit for 10 years c. A registered nurse with 6 months of experience on the surgical unit d. A registered nurse who has floated to the surgical unit from pediatrics

c. A registered nurse with 6 months of experience on the surgical unit

When caring for the patient with heart failure, the nurse knows that which gastrointestinal process is most dependent on cardiac output and may affect the patient's nutritional status? a. Ingestion b. Digestion c. Absorption d. Elimination

c. Absorption

A patient with cholelithiasis needs to have the gallbladder removed. Which patient assessment is a contraindication for a cholecystectomy? a. Low-grade fever of 100° F and dehydration b. Abscess in the right upper quadrant of the abdomen c. Activated partial thromboplastin time (aPTT) of 54 seconds d. Multiple obstructions in the cystic and common bile duct

c. Activated partial thromboplastin time (aPTT) of 54 seconds

When assessing a patient's abdomen, what would be most appropriate for the nurse to do? a. Palpate the abdomen before auscultation. b. Percuss the abdomen before auscultation. c. Auscultate the abdomen before palpation. d. Perform deep palpation before light palpation.

c. Auscultate the abdomen before palpation.

The nurse teaching young adults about behaviors that put them at risk for oral cancer includes a. Discouraging use of chewing gum. b. Avoiding use of perfumed lip gloss. c. Avoiding use of smokeless tobacco. d. Discouraging drinking of carbonated beverages.

c. Avoiding use of smokeless tobacco.

A patient returns to the surgical unit with a nasogastric (NG) tube to low intermittent suction, IV fluids, and a Jackson-Pratt drain at the surgical site following an exploratory laparotomy and repair of a bowel perforation. Four hours after admission, the patient experiences nausea and vomiting. What is a priority nursing intervention for the patient? a. Assess the abdomen for distention and bowel sounds. b. Inspect the surgical site and drainage in the Jackson-Pratt. c. Check the amount and character of gastric drainage and the patency of the NG tube. d. Administer prescribed prochlorperazine (Compazine) to control the nausea and vomiting.

c. Check the amount and character of gastric drainage and the patency of the NG tube. An adequately functioning nasogastric (NG) tube should prevent nausea and vomiting because stomach contents are continuously being removed. The first intervention in this case is to check the amount and character of the recent drainage and check the tube for patency. Decreased or absent bowel sounds are expected after a laparotomy and the Jackson-Pratt drains only fluid from the tissue of the surgical site. Antiemetics may be given if the NG tube is patent because anesthetic agents may cause nausea.

The patient tells the nurse she had a history of abdominal pain, so she had a surgery to make an opening into the common bile duct to remove stones. The nurse knows that this surgery is called a a. Colectomy b. Cholecystectomy c. Choledocholithotomy d. Choledochojejunostomy

c. Choledocholithotomy

A nurse is completing an assessment of a client who has a gastric ulcer. Which of the following are expected findings? (Select all that apply) a. Client reports pain relieved by eating b. Client states that pain often occurs at night c. Client reports a sensation of bloating d. Client states that pain occurs 1/2 to 1 hr after a meal e. Client experiences pain upon palpation of the epigastric region

c. Client reports a sensation of bloating d. Client states that pain occurs 1/2 to 1 hr after a meal e. Client experiences pain upon palpation of the epigastric region

The patient has been newly diagnosed with Wilson's disease and D-penicillamine, a chelating agent, has been prescribed. What assessment finding should the nurse expect? a. Pruritus b. Acute kidney injury c. Corneal Fleischer rings d. Elevated serum iron levels

c. Corneal Fleischer rings Corneal Fleischer rings, brownish red rings in the cornea near the limbus, are the hallmark of Wilson's disease. Pruritus (not seen with Wilson's disease) is commonly seen with jaundice or primary biliary cirrhosis. Renal failure associated with hepatorenal syndrome is not seen with Wilson's disease. Elevated serum iron levels are seen with hemochromatosis.

The nurse should teach the patient with chronic constipation that which food has the highest dietary fiber? a. Peach b. Popcorn c. Dried beans d. Shredded wheat

c. Dried beans Of the foods listed, dried beans contain the highest amount of dietary fiber and are an excellent source of soluble fiber. Bran and berries also have large amounts of fiber.

The patient is having a gastroduodenostomy (Billroth I operation) for stomach cancer. What long-term complication is occurring when the patient reports generalized weakness, sweating, palpitations, and dizziness 15 to 30 minutes after eating? a. Malnutrition b. Bile reflux gastritis c. Dumping syndrome d. Postprandial hypoglycemia

c. Dumping syndrome

A female patient has a sliding hiatal hernia. What nursing interventions will prevent the symptoms of heartburn and dyspepsia that she is experiencing? a. Keep the patient NPO. b. Put the bed in the Trendelenberg position. c. Have the patient eat 4 to 6 smaller meals each day. d. Give various antacids to determine which one works for the patient.

c. Have the patient eat 4 to 6 smaller meals each day.

The condition of a patient who has cirrhosis of the liver has deteriorated. Which diagnostic study would help determine if the patient has developed liver cancer? a. Serum α-fetoprotein level b. Ventilation/perfusion scan c. Hepatic structure ultrasound d. Abdominal girth measurement

c. Hepatic structure ultrasound

A nurse is reviewing the health record of a client who has pancreatitis. The physical exam report by the provider indicates the presence of Cullen's sign. Which of the following is an appropriate action by the nurse to identify this finding? a. Tap lightly at the costovertebral margin on the client's back b. Palpate the client's right lower quadrant c. Inspect the skin around the umbilicus d. Auscultate the area below the client's scapula

c. Inspect the skin around the umbilicus

When caring for a patient with autoimmune hepatitis, the nurse understands that what in this patient is different from the patient who has viral hepatitis? a. Does not manifest hepatomegaly or jaundice b. Experiences less liver inflammation and damage c. Is treated with corticosteroids or other immunosuppressive agents d. Is usually an older adult who has used a wide variety of prescription and over-the-counter drugs

c. Is treated with corticosteroids or other immunosuppressive agents Immunosuppressive agents are indicated in hepatitis associated with immune disorders to decrease liver damage caused by autoantibodies. Autoimmune hepatitis is similar to viral hepatitis in presenting signs and symptoms and may become chronic and lead to cirrhosis.

A patient who has undergone an esophagectomy for esophageal cancer develops increasing pain, fever, and dyspnea when a full liquid diet is started postoperatively. The nurse recognizes that these symptoms are most indicative of a. An intolerance to the feedings. b. Extension of the tumor into the aorta. c. Leakage of fluid or foods into the mediastinum. d. Esophageal perforation with fistula formation into the lung.

c. Leakage of fluid or foods into the mediastinum.

A 74-year-old female patient with gastroesophageal reflux disease (GERD) takes over-the-counter medications. For which medication, if taken long-term, should the nurse teach about an increased risk of fractures? a. Sucralfate (Carafate) b. Cimetidine (Tagamet) c. Omeprazole (Prilosec) d. Metoclopramide (Reglan)

c. Omeprazole (Prilosec)

How should the nurse teach the patient with a hiatal hernia or GERD to control symptoms? a. Drink 10 to 12 oz of water with each meal. b. Space six small meals a day between breakfast and bedtime. c. Sleep with the head of the bed elevated on 4- to 6-inch blocks d. Perform daily exercises of toe-touching, sit-ups, and weight lifting.

c. Sleep with the head of the bed elevated on 4- to 6-inch blocks The use of blocks to elevate the head of the bed facilitates gastric emptying by gravity and is strongly recommended to prevent nighttime reflux. Liquids should be taken between meals to prevent gastric distention with meals. Small meals should be eaten frequently but patients should not eat at bedtime or lie down for 2 to 3 hours after eating. Activities that involve increasing intraabdominal pressure, such as bending over, lifting, or wearing tight clothing, should be avoided.

A nurse is completing discharge teaching with a client who is 3 days postoperative for a transverse colostomy. Which of the following should be included in the teaching? a. Mucus will be present in stool for 5 to 7 days after surgery b. Expect 500 to 1,000 mL of semi-liquid stool after 2 weeks c. Stoma should be moist and pink d. Change the ostomy bag when it is 3/4 full

c. Stoma should be moist and pink

A patient with a history of peptic ulcer disease is hospitalized with symptoms of a perforation. During the initial assessment, what should the nurse expect the patient to report? a. Vomiting of bright-red blood b. Projectile vomiting of undigested food c. Sudden, severe upper abdominal pain and back pain d. Hyperactive stomach sounds and upper abdominal swelling

c. Sudden, severe upper abdominal pain and back pain Perforation of an ulcer causes sudden, severe abdominal pain that is often referred to the back, accompanied by a rigid, boardlike abdomen and other signs of peritonitis. Vomiting of blood indicates hemorrhage of an ulcer and gastric outlet obstruction is characterized by projectile vomiting of undigested food, hyperactive stomach sounds, and upper abdominal swelling.

Serologic findings in viral hepatitis include both the presence of viral antigens and antibodies produced in response to the viruses. What laboratory result indicates that the nurse is immune to HBV after vaccination? a. Anti-HBcIgG b. Surface antigen HBsAg c. Surface antibody Anti-HBs d. Core antigen Anti-HBcIgM

c. Surface antibody Anti-HBs Immunization to HBV after vaccination is identified with the hepatitis B surface antibody Anti-HBs. Anti-HBcIgG indicates previous or ongoing HBV infection. Surface antigen HBsAg is present in acute and chronic infection. Core antigen Anti-HBcIgM indicates acute infection and does not appear after vaccination.

A patient is scheduled for biopsy of a painful tongue ulcer. Based on knowledge of risk factors for oral cancer, what should the nurse specifically ask the patient about during a history? a. Excessive exposure to sunlight b. Recurrent herpes simplex infections c. Use of any type of tobacco products d. Difficulty swallowing and pain in the ear

c. Use of any type of tobacco products A positive history of use of tobacco and alcohol is the most significant etiologic factor in oral cancer. Excessive exposure to ultraviolet radiation from the sun is a factor in the development of cancer of the lip. Herpes simplex infections have not been associated with oral cancer. Difficulty swallowing and ear pain are symptoms of advanced oral cancer, not risk factors.

When teaching the patient with acute hepatitis C (HCV), the patient demonstrates understanding when the patient makes which statement? a. "I will use care when kissing my wife to prevent giving it to her." b. "I will need to take adofevir (Hepsera) to prevent chronic HCV." c. "Now that I have had HCV, I will have immunity and not get it again." d. "I will need to be checked for chronic HCV and other liver problems."

d. "I will need to be checked for chronic HCV and other liver problems."

The nurse is caring for a postoperative patient with a colostomy. The nurse is preparing to administer a dose of famotidine (Pepcid) when the patient asks why the medication was ordered since the patient does not have a history of heartburn or gastroesophageal reflux disease (GERD). What response by the nurse would be the most appropriate? a. "This will prevent air from accumulating in the stomach, causing gas pains." b. "This will prevent the heartburn that occurs as a side effect of general anesthesia." c. "The stress of surgery is likely to cause stomach bleeding if you do not receive it." d. "This will reduce the amount of HCl in the stomach until the nasogastric tube is removed and you can eat a regular diet again."

d. "This will reduce the amount of HCl in the stomach until the nasogastric tube is removed and you can eat a regular diet again."

A patient with cirrhosis that is refractory to other treatments for esophageal varices undergoes a portacaval shunt. As a result of this procedure, what should the nurse expect the patient to experience? a. An improved survival rate b. Decreased serum ammonia levels c. Improved metabolism of nutrients d. Improved hemodynamic function and renal perfusion

d. Improved hemodynamic function and renal perfusion By shunting fluid sequestered in the peritoneum into the venous system, pressure on esophageal veins is decreased and more volume is returned to the circulation, improving cardiac output and renal perfusion. However, because ammonia is diverted past the liver, hepatic encephalopathy continues. These procedures do not prolong life or promote liver function.

A nurse is caring for a client who had a paracentesis. Which of the following findings indicate the bowel was perforated during the procedure? a. Client report of upper chest pain b. Decreased urine output c. Pallor d. Temperature elevation

d. Temperature elevation

What physiologically occurs with vomiting? a. The acid-base imbalance most commonly associated with persistent vomiting is metabolic acidosis caused by loss of bicarbonate. b. Stimulation of the vomiting center by the chemoreceptor trigger zone (CTZ) is commonly caused by stretch and distention of hollow organs. c. Vomiting requires the coordination of activities of structures including the glottis, respiratory expiration, relaxation of the pylorus, and closure of the lower esophageal sphincter. d. Immediately before the act of vomiting, activation of the parasympathetic nervous system causes increased salivation, increased gastric motility, and relaxation of the lower esophageal sphincter.

d. The parasympathetic nervous system causes increased salivation and gastric mobility as well as relaxation of the lower esophageal sphincter. The acid-base imbalance that occurs with vomiting is metabolic alkalosis from the loss of hydrochloric acid. The vomiting center in the chemoreceptor trigger zone (CTZ) can be caused by chemical stimuli of drugs, toxins, and labyrinthine stimulation. Vomiting requires the coordination of closing the glottis, deep inspiration with contraction of the diaphragm in the inspiratory position, closure of the pylorus, relaxation of the stomach and lower esophageal sphincter, and contraction of abdominal muscles.

The nurse explains to the patient with Vincent's infection that treatment will include a. Smallpox vaccinations. b. Viscous lidocaine rinses. c. Amphotericin B suspension. d. Topical application of antibiotics.

d. Topical application of antibiotics.

What should a patient be taught after a hemorrhoidectomy? a. Take mineral oil before bedtime. b. Eat a low-fiber diet to rest the colon. c. Administer oil-retention enema to empty the colon. d. Use prescribed pain medication before a bowel movement.

d. Use prescribed pain medication before a bowel movement.

A nurse is completing nutrition teaching for a client who has pancreatitis. Which of the following statements by the client requires further teaching? a. "I plan to eat small, frequent meals" b. "I will eat easy-to-digest foods with limited spice" c. " I will use skim milk when cooking" d. "I plan to drink regular cola"

d. "I plan to drink regular cola"

The nurse is teaching a group of high school students about the prevention of food poisoning. Which comment by the student shows understanding of foodborne illness protection? a. "We like to mix up the ingredients so the flavors will melt before we cook our beef stew." b. "For a snack, I like to eat raw cookie dough from the package instead of baking the cookies." c. "We only have one cutting board, so we cut up our chicken and salad vegetables at the same time." d. "When they gave me a pink hamburger I sent it back and asked for a new bun and clean plate."

d. "When they gave me a pink hamburger I sent it back and asked for a new bun and clean plate."

Which patients would be at highest risk for developing oral candidiasis? a. A 74-year-old patient who has vitamin B and C deficiencies b. A 22-year-old patient who smokes 2 packs of cigarettes per day c. A 58-year-old patient who is receiving amphotericin B for 2 days d. A 32-year-old patient who is receiving ciprofloxacin (Cipro) for 3 weeks

d. A 32-year-old patient who is receiving ciprofloxacin (Cipro) for 3 weeks

When a patient returns to the clinical unit after an abdominal-perineal resection (APR), what should the nurse expect? a. An abdominal dressing b. An abdominal wound and drains c. A temporary colostomy and drains d. A perineal wound, drains, and a stoma

d. A perineal wound, drains, and a stoma With an abdominal perineal-resection (APR), an abdominal incision is made and the proximal sigmoid colon is brought through the abdominal wall and formed into a permanent colostomy. The patient is repositioned, a perineal incision is made, and the distal sigmoid colon, rectum, and anus are removed through the perineal incision, which may be left open, packed, and have drains.

The nurse is admitting a 68-year-old man with severe dehydration and frequent watery diarrhea. He just completed a 10-day outpatient course of antibiotic therapy for bacterial pneumonia. It is most important for the nurse to take which action? a. Wear a mask to prevent transmission of infection. b. Wipe equipment with ammonia-based disinfectant. c. Instruct visitors to use the alcohol-based hand sanitizer. d. Don gloves and gown before entering the patient's room.

d. Don gloves and gown before entering the patient's room.

The nursing management of the patient with cholecystitis associated with cholelithiasis is based on the knowledge that a. Shock-wave therapy should be tried initially. b. Once gallstones are removed, they tend not to recur. c. The disorder can be successfully treated with oral bile salts that dissolve gallstones. d. Laparoscopic cholecystectomy is the treatment of choice in most patients who are symptomatic.

d. Laparoscopic cholecystectomy is the treatment of choice in most patients who are symptomatic.

Which complication of acute pancreatitis requires prompt surgical drainage to prevent sepsis? a. Tetany b. Pseudocyst c. Pleural effusion d. Pancreatic abscess

d. Pancreatic abscess A pancreatic abscess is a collection of pus that must be drained to prevent infection of adjacent organs and sepsis. Tetany from hypocalcemia is treated with IV calcium gluconate (10%). Although pseudocysts usually resolve spontaneously, they may be treated with surgical, percutaneous catheter, or endoscopic drainage to prevent perforation. Pleural effusion is treated by treating the cause (pancreatitis) and monitoring for respiratory distress and oxygen saturation.

A patient reports having a dry mouth and asks for something to drink. The nurse recognizes that this symptom can most likely be attributed to a common adverse effect of which medication that the patient is taking? a. Digoxin (Lanoxin) b. Cefotetan (Cefotan) c. Famotidine (Pepcid) d. Promethazine (Phenergan)

d. Promethazine (Phenergan)

When assessing a patient with acute pancreatitis, the nurse would expect to find a. Hyperactive bowel sounds. b. Hypertension and tachycardia. c. A temperature greater than 102°F (38.9°C). d. Severe midepigastric or left upper quadrant (LUQ) pain.

d. Severe midepigastric or left upper quadrant (LUQ) pain. The predominant symptom of acute pancreatitis is severe, deep abdominal pain that is usually located in the left upper quadrant (LUQ) but may be in the midepigastrium. Bowel sounds are decreased or absent, temperature is elevated only slightly, and the patient has hypovolemia and may manifest symptoms of shock.

The client two (2) hours postoperative laparoscopic cholecystectomy is complaining of severe pain in the right shoulder. Which nursing intervention should the nurse implement? a. Apply a heating pad to the abdomen for 15 to 20 minutes. b. Administer morphine sulfate intravenously after diluting with saline. c. Contact the surgeon for an order to x-ray the right shoulder. d. Apply a sling to the right arm that was injured in surgery.

**a. A heating pad should be applied for 15 to 20 minutes to assist the migration of the O2 used to insufflate the abdomen. b. Morphine sulfate would not affect the etiology of the pain. c. The surgeon would not order an x-ray for this condition. d. There is no indication that an injury occurred during surgery. A sling would not benefit the migration of the O2.

Which sign/symptom should the nurse expect to find in a client diagnosed with ulcerative colitis? a. Twenty bloody stools a day b. Oral temp of 102 F c. Hard, rigid abdomen d. Urinary stress incontinence

**a. The colon is ulcerated and unable to absorb water, resulting in bloody diarrhea. Ten (10) to twenty bloody diarrhea stools is the most common symptom of ulcerative colitis. b. Although there is an inflammation of the colon, there is usually not an elevated temperature, and if it is elevated, it is a low-grade fever. c. A hard, rigid abdomen indicates peritonitis, which is a complication of ulcerative colitis but not an expected symptom. d. Stress incontinence is not a symptom of colitis.

The client diagnosed with acute diverticulitis is complaining of severe abdominal pain. On assessment, the nurse finds a hard, rigid abdomen and T 102F. Which intervention should the nurse implement? a. Notify the health-care provider. b. Prepare to administer a Fleet's enema. c. Administer an antipyretic suppository. d. Continue to monitor the client closely.

**a. These are signs of peritonitis, which is life threatening. The health-care provider should be notified immediately. b. A Fleet's enema will not help a life-threatening complication of diverticulitis. c. A medication administered to help decrease the client's temperature will not help a life-threatening complication. d. These are signs/symptoms that indicate a possible life-threatening situation and require immediate intervention.

The nurse cares for a 34-year-old woman after bariatric surgery. The nurse determines that discharge teaching related to diet is successful if the patient makes which statement? a. "A high protein diet that is low in carbohydrates and fat will prevent diarrhea." b. "Food should be high in fiber to prevent constipation from the pain medication." c. "Three meals a day with no snacks between meals will provide optimal nutrition." d. "Fluid intake should be at least 2000 mL per day with meals to avoid dehydration."

a. "A high protein diet that is low in carbohydrates and fat will prevent diarrhea."

Which type of precaution should the nurse implement to protect from being exposed to any of the hepatitis viruses? a. Airborne precautions. b. Standard precautions. c. Droplet precautions. d. Exposure precautions.

a. Airborne precautions are required for trans- mission that occurs by dissemination of either airborne droplet nuclei or dust particles containing the infectious agent. **b. Standard Precautions apply to blood, all body fluids, secretions, and excretions, except sweat, regardless of whether they contain visible blood. c. Droplet transmission involves contact of the conjunctivae of the eyes or mucous membranes of the nose or mouth with large-particle droplets generated during coughing, sneezing, talking, or suctioning. d. There is no such precaution known as expo- sure precautions.

A community health nurse is conducting an initial assessment of a new patient. Which assessments should the nurse include when screening the patient for metabolic syndrome? Select all that apply a. Blood pressure b. Resting heart rate c. Physical endurance d. Waist circumference e. Fasting blood glucose

a. Blood pressure d. Waist circumference e. Fasting blood glucose

A 35-year-old man with a family history of adenomatous polyposis had a colonoscopy with removal of multiple polyps. Which signs and symptoms should the nurse teach the patient to report immediately? a. Fever and abdominal pain b. Flatulence and liquid stool c. Loudly audible bowel sounds d. Sleepiness and abdominal cramps

a. Fever and abdominal pain

The nurse is completing the shift assessment on the client recovering from abdominal surgery who has a PCA pump. The client has shallow respirations and refuses to deep breathe. Which intervention should the nurse implement? a. Insist that the client take deep breaths. b. Notify the surgeon to request a chest x-ray. c. Determine the last time the client used the PCA pump. d. Administer oxygen 2 L/min via nasal cannula.

a. The nurse cannot force the client to do anything; this would be considered assault. b. There are no data that support the need for a chest x-ray. **c. Shallow respirations and refusal to deep breathe could be the result of abdominal pain. The nurse should assess the client for pain and determine the last time the PCA pump was used. d. Based on the information given, the client does not need oxygen.

Which assessment data indicate that the client recovering from an open cholecystectomy requires pain medication? a. The client's pulse is 65 beats per minute. b. The client has shallow respirations. c. The client's bowel sounds are 20 per minute. d. The client uses a pillow to splint when coughing.

a. The nurse would expect an increased pulse in the client who is in acute pain. **b. Clients having abdominal pain frequently have shallow respirations. When assessing clients for pain, the nurse should discuss pain medication with any client who has shallow respirations. c. These are normal data and would not require further action. d. Splinting the abdomen allows the client to increase the strength of the cough by increasing comfort and would not indicate a need for pain medication.

The client diagnosed with liver problems asks the nurse, "Why are my stools clay- colored?" On which scientific rationale should the nurse base the response? a. There is an increase in serum ammonia level. b. The liver is unable to excrete bilirubin. c. The liver is unable to metabolize fatty foods. d. A damaged liver cannot detoxify vitamins.

a. The serum ammonia level is increased in liver failure, but it is not the cause of clay-colored stools. **b. Bilirubin, the byproduct of red blood cell destruction, is metabolized in the liver and excreted via the feces, which is what gives the feces the dark color. If the liver is damaged, the bilirubin is excreted via the urine and skin. c. The liver excretes bile into the gallbladder and the body uses the bile to digest fat, but it does not affect the feces. d. Vitamin deficiency, resulting from the liver's inability to detoxify vitamins, may cause steatorrhea, but it does not cause clay-colored stool.

Which intervention should the nurse include when discussing ways to help prevent potential episodes of gastroenteritis from Clostridium botulism? a. Make sure that all hamburger meat is well cooked. b. Ensure that all dairy products are refrigerated. c. Discuss that campers should drink only bottled water. d. Discard all canned goods that are damaged.

a. This will help prevent gastroenteritis secondary to staphylococcal food poisoning. b. This will help prevent gastroenteritis secondary to foods kept at room temperature, causing staphylococcal food poisoning. c. This will help prevent gastroenteritis secondary to Escherichia coli and contaminated water. **d. Any food that is discolored or comes from a can or jar that has been damaged or does not have a tight seal should be destroyed without tasting or touching it.

The nurse is teaching a class on diverticulosis. Which interventions should the nurse discuss when teaching ways to prevent an acute exacerbation of diverticulosis? Select all that apply. a. Eat a high-fiber diet. b. Increase fluid intake. c. Elevate the HOB after eating. d. Walk 30 minutes a day. e. Take an antacid every two (2) hours.

**a. A high-fiber diet will help to prevent constipation, which is the primary reason for diverticulitis. **b. Increased fluids will help keep the stool soft and prevent constipation. c. This will not do anything to help prevent diverticulitis. **d. Exercise will help prevent constipation. e. There are no medications used to help prevent an acute exacerbation of diverticulitis. Antacids are used to neutralize hydrochloric acid in the stomach.

The 85-year-old male client diagnosed with cancer of the colon asks the nurse, "Why did I get this cancer?" Which statement is the nurse's best response? a. "Research shows a lack of fiber in the diet can cause colon cancer" 2. "It is not common to get colon cancer at your age; it is usually in young people" 3. "No one knows why anyone gets cancer, it just happens to certain people" 4. "Women usually get colon cancer more often than men but not always"

**a. A long history of low-fiber, high-fat, high protein diets results in a prolonged transit time. This allows the carcinogenic agents in the waste products to have a greater exposure to the lumen of the colon. b. The older the client, the greater the risk of developing cancer of the colon. c. Risk factors for cancer of the colon include increasing age; family history of colon cancer or polyps; history of IBD; genital or breast cancer; and eating a high-fat, high-protein, low-fiber diet. d. Males have a slightly higher incidence of colon cancers than do females.

Which physical exam should the nurse implement first when assessing the client diagnosed with peptic ulcer disease? a. Auscultate the client's bowel sounds in all 4 quadrants b. Palpate the abdominal area for tenderness c. Percuss the abdominal borders to identify organs d. Assess the tender area progressing to nontender

**a. Auscultation should be used prior to palpation or percussion when assessing the abdomen. If the nurse manipulates the abdomen, the bowel sounds can be altered and give false information. b. Palpation gives appropriate information that the nurse needs to collect, but if done prior to auscultation, the sounds will be altered. c. Percussion of the abdomen would not give specific information about peptic ulcer disease. d. Tender areas should be assessed last to prevent guarding and altering the assessment. This would include palpation, which should be done after auscultation.

The client has end-stage liver failure secondary to alcoholic cirrhosis. Which complication indicates the client is at risk for developing hepatic encephalopathy? a. Gastrointestinal bleeding. b. Hypoalbuminemia. c. Splenomegaly. d. Hyperaldosteronism

**a. Blood in the intestinal tract is digested as a protein, which increases serum ammonia levels and increases the risk of developing hepatic encephalopathy. b. Decreased albumin would cause the client to develop ascites. c. An enlarged spleen increases the rate at which RBCs, WBCs, and platelets are destroyed, which causes the client to develop anemia, leukopenia, and thrombocytopenia, but not hepatic encephalopathy. d. An increase in aldosterone causes sodium and water retention that, in turn, causes the development of ascites and generalized edema.

Which signs and symptoms should the nurse report to the health-care provider for the client recovering from an open cholecystectomy? (Select all that apply) a. Clay-colored stools b. Yellow-tinted sclera c. Dark-colored urine d. Feverish chills e. Abdominal pain

**a. Clay-colored stools are caused by recurring stricture of the common bile duct, which is a sign of post-cholecystectomy syndrome. **b. Yellow-tinted sclera and skin indicate residual effects of stricture of the common bile duct, which is a sign of post-cholecystectomy syndrome. **c. Dark yellow urine indicates a residual effect of a stricture of the common bile duct, which is a sign of post-cholecystectomy syndrome. **d. Fever and chills indicate residual or recurring calculi, inflammation, or stricture of common bile duct, which is a sign of postcholecystectomy syndrome. **e. Abdominal pain indicates a residual effect of a stricture of common bile duct, inflammation, or calculi, which is a sign of postcholecystectomy syndrome.

The nurse is planning the care of a client who has had an abdominal perineal resection for cancer of the colon. Which interventions should the nurse implement? Select all that apply. a. Provide meticulous skin care to stoma. b. Assess the flank incision. c. Maintain the indwelling catheter. d. Irrigate the J-P drains every shift. e. Position the client semi-recumbent.

**a. Colostomy stomas are portions of the large intestines pulled through the abdominal wall through which feces exits the body. Feces can be irritating to the abdominal skin, so careful and thorough skin care is needed. b. There are midline and perineal incisions, not flank incisions. **c. Because of the perineal wound, the client will have an indwelling catheter to keep urine out of the incision. d. Jackson Pratt drains are emptied every shift, but they are not irrigated. **e. The client should not sit upright because this would cause pressure on the perineum.

The client has a large abdominal wound that has eviscerated. Which intervention should the nurse implement? a. Apply sterile normal saline dressing. b. Use sterile gloves to replace protruding parts. c. Place the client in the reverse Trendelenburg position. d. Administer intravenous antibiotic stat.

**a. Evisceration is a life-threatening condition in which the abdominal contents have protruded through the ruptured incision. The nurse must protect the bowel from the environment by placing a sterile normal saline dressing on it. The saline prevents the intestines from drying out and necrosing. b. The nurse should not attempt to replace the protruding bowel. c. This position places the client with the head of the bed elevated, which will make the situation worse. d. Antibiotics will not protect the protruding bowels, which must be priority. Antibiotics will be administered at a later time to prevent infection, but this is not urgent.

Which nursing problem is priority for the 76-year-old client diagnosed with gastroenteritis from staphylococcal food poisoning? a. Fluid volume deficit. b. Nausea. c. Risk for aspiration. d. Impaired urinary elimination.

**a. Fluid volume deficit secondary to diarrhea is the priority because of the potential for metabolic acidosis and hypokalemia, which are both life threatening, especially in the elderly. b. Nausea may occur, but it is not priority. However, excessive vomiting could lead to potential complications. c. Risk for aspiration could result from vomiting; however, vomiting does not usually occur in food poisoning, but it may be secondary to botulism. d. Impaired urinary elimination is not a priority. The client has diarrhea, not urine output problems.

The client with acute diverticulitis has a nasogastric tube draining green liquid bile. Which action should the nurse implement? a. Document the findings as normal. b. Assess the client's bowel sounds. c. Determine the client's last bowel movement. d. Insert the NG tube at least 2 more inches.

**a. Green bile contains hydrochloric acid and should be draining from the N/G tube; therefore the nurse should take no action and should document the findings. b. There is no reason for the nurse to take further action because this is normal. c. The client's last bowel movement would not affect the N/G drainage. d. Bile draining from the N/G tube indicates that the tube is in the stomach and there is no need to advance the tube further.

The public health nurse is discussing hepatitis B with a group in the community. Which health promotion activities should the nurse discuss with the group? Select all that apply. a. Do not share needles or equipment. b. Use barrier protection during sex. c. Get the hepatitis B vaccines. d. Obtain immune globulin injections. e. Avoid any type of hepatotoxic medications

**a. Hepatitis B can be transmitted by sharing any type of needles, especially those used by drug abusers. **b. Hepatitis B can be transmitted through sexual activity; therefore the nurse should recommend abstinence, mutual monog- amy, or barrier protection. **c. Three doses of hepatitis B vaccine provide immunity in 90% of healthy adults. d. Immune globulin injections are administered as post-exposure prophylaxis (after being exposed to hepatitis B), but encouraging these injections is not a health promotion activity. e. Hepatotoxic medications should be avoided in clients who have hepatitis or who have had hepatitis. The health-care provider prescribes medications, and the person in the community does not know which medications are hepato- toxic.

The client diagnosed with liver failure is experiencing pruritus secondary to severe jaundice. Which action by the unlicensed assistant warrants intervention by the primary nurse? a. Assisting the client to take a hot soapy shower. b. Applying an emollient to the client's legs and back. c. Putting mittens on both hands of the client. d. Patting the client's skin dry with a clean towel.

**a. Hot water increases pruritus, and soap will cause dry skin, which increases pruritus; therefore, the nurse should discuss this with the assistant. b. This will help prevent dry skin, which will help decrease pruritus; therefore this would not require any intervention by the primary nurse. c. Mittens will help prevent the client from scratching the skin and causing skin breakdown. d. The skin should be patted dry, not rubbed, because rubbing the skin will cause increased irritation.

The client diagnosed with AIDS is experiencing voluminous diarrhea. Which interventions should the nurse implement? Select all that apply. a. Monitor diarrhea, charting amount, character, and consistency. b. Assess the client's tissue turgor every day. c. Encourage the client to drink carbonated soft drinks. d. Weigh the client daily in the same clothes and at the same time. e. Assist the client with a warm sitz bath PRN.

**a. It is important to keep track of the amounts, color, and other characteristics of all body fluids lost. b. Skin turgor should be assessed at least every six (6) to eight (8) hours, not daily. c. Carbonated soft drinks increase flatus in the GI tract, and the increased sugar will act as an osmotic laxative and increase the diarrhea. **d. Daily weights are the best method of determining fluid loss and gain. **e. Sitz baths will assist in keeping the client's perianal area clean without having to rub. The warm water is soothing, providing comfort.

The nurse is caring for clients on a medical unit. Which client information should be brought to the attention of the HCP immediately? a. A serum sodium of 128 mEq/L in a client diagnosed with obstipation b. The client diagnosed with fecal impaction who had 2 hard, formed stools c. A serum potassium level of 3.8 mEq/L in a client diagnosed with diarrhea d. The client with diarrhea who had 2 semiliquid stools totaling 300 mL

**a. Normal serum sodium levels are 135 to 152 mEq/L, so the client's 128 mEq/L value requires intervention b. The client diagnosed with a fecal impaction is beginning to move the stool; this indicates an improvement c. Normal potassium levels are 3.5 to 5.5 mEq/L. A level of 3.8 mEq/L is within normal limits and does not require intervention d. This client has been having diarrhea and now is having semiliquid stools, so this client is getting better

The nurse is caring for an adult client diagnosed with GERD. Which condition is the most common comorbid disease associated with GERD? a. Adult onset asthma b. Pancreatitis c. Peptic ulcer disease d. Increased gastric emptying

**a. Of adult-onset asthma cases, 80%-90% are caused by gastroesophageal reflux disease (GERD). b. Pancreatitis is not related to GERD. c. Peptic ulcer disease is related to H. pylori bacterial infections and can lead to increased levels of gastric acid, but it is not related to reflux. d. GERD is not related to increased gastric emptying. Increased gastric emptying would be a benefit to a client with decreased functioning of the lower esophageal sphincter.

The nurse is performing an admission assessment on a client diagnosed with GERD. Which sign and symptoms would indicate GERD? a. Pyrosis, water brash, and flatulence b. Weight loss, dysarthria, and diarrhea c. Decreased abdominal fat, proteinuria, and constipation d. Midepigastric pain, positive H pylori test, and melena

**a. Pyrosis is heartburn, water brash is the feeling of saliva secretion as a result of reflux, and flatulence is gas—all symptoms of GERD. b. Gastroesophageal reflux disease does not cause weight loss. c. There is no change in abdominal fat, no proteinuria (the result of a filtration problem in the kidney), and no alteration in bowel elimination for the client diagnosed with GERD. d. Mid-epigastric pain, a positive H. pylori test, and melena are associated with gastric ulcer disease.

The nurse writes the client problem "imbalanced nutrition: less than body requirements" for the client diagnosed with hepatitis. Which intervention should the nurse include in the plan of care? a. Provide a high-calorie intake diet. b. Discuss total parenteral nutrition (TPN). c. Instruct the client to decrease salt intake. d. Encourage the client to increase water intake.

**a. Sufficient energy is required for healing. Adequate carbohydrate intake can spare protein. The client should eat approximately 16 carbohydrate kilocalories for each kilogram of ideal body weight daily. b. TPN is not routinely prescribed for the client with hepatitis; the client would have to have lost a large of amount of weight and be unable to eat anything for TPN to be ordered. c. Salt intake does not affect the healing of the liver. d. Water intake does not affect healing of the liver, and the client should not drink so much water as to decrease caloric food intake.

The client is diagnosed with salmonellosis secondary to eating some slightly cooked hamburger meat. Which clinical manifestations would the nurse expect the client to report? a. Abdominal cramping, nausea, and vomiting. b. Neuromuscular paralysis and dysphagia. c. Gross amounts of explosive bloody diarrhea. d. Frequent "rice water stool" with no fecal odor.

**a. Symptoms develop 8-48 hours after ingesting the Salmonella bacteria and include diarrhea, abdominal cramping, nausea, and vomiting, along with low-grade fever, chills, and weakness. b. This occurs with botulism, a severe life-threatening form of food poisoning caused by Clostridium botulinum. c. This is a clinical manifestation of hemorrhagic colitis caused by Escherichia coli. d. This gray-cloudy diarrhea that has no fecal odor, blood, or pus is caused by cholera, which is endemic in parts of Asia, the Middle East, and Africa.

The client diagnosed with IBD is prescribed total parental nutrition. Which intervention should the nurse implement? a. Check the client's glucose level b. Administer an oral hypoglycemic c. Assess the peripheral IV site d. Monitor the clients oral food intake

**a. TPN is high in dextrose, which is glucose; therefore the client's blood glucose level must be monitored closely. b. The client may be on sliding-scale regular insulin coverage for the high glucose level. c. The TPN must be administered via a subclavian line because of the high glucose level. d. The client would be NPO to put the bowel at rest, which is the rationale for administering the TPN.

The nurse caring for a client diagnosed with GERD writes the client problem of "behavior modification." Which intervention should be included for this problem? a. Teach the client to sleep with a foam wedge under the head b. Encourage the client to decrease the amount of smoking c. Instruct the client to take OTC medication for the relief of pain d. Discuss the need to attend AA meetings to stop drinking

**a. The client should elevate the head of the bed on blocks or use a foam wedge to use gravity to help keep the gastric acid in the stomach and prevent reflux into the esophagus. Behavior modification is changing one's behavior. b. The client should be encouraged to quit smoking altogether. Referral to support groups for smoking cessation should be made. c. The nurse would be prescribing medication; this is not in the nurse's scope of practice. d. The client should be instructed to discontinue using alcohol, but the stem does not indicate the client is an alcoholic.

Which type of hepatitis is transmitted by the fecal-oral route via contaminated food, water, or direct contact with an infected person? a. Hepatitis A. b. Hepatitis B. c. Hepatitis C. d. Hepatitis D.

**a. The hepatitis A virus is in the stool of infected people up to two (2) weeks before symptoms develop. b. Hepatitis B virus is spread through contact with infected blood and body fluids. c. Hepatitis C virus is transmitted through infected blood and body fluids. d. Hepatitis D virus only causes infection in people who are also infected with hepatitis B or C.

The nurse caring for a client one (1) day postoperative sigmoid resection notes a moderate amount of dark reddish brown drainage on the midline abdominal incision. Which intervention should the nurse implement first? a. Mark the drainage on the dressing with the time and date. b. Change the dressing immediately using sterile technique. c. Notify the health-care provider immediately. d. Reinforce the dressing with a sterile gauze pad.

**a. The nurse should mark the drainage on the dressing to determine if active bleeding is occurring because dark reddish-brown drainage indicates old blood. This allows the nurse to assess what is actually happening. b. Surgical dressings are initially changed by the surgeon; the nurse should not remove the dressing until the surgeon orders the dressing change to be done by the nurse. c. The nurse should assess the situation before notifying the HCP. d. The nurse may need to reinforce the dressing if the dressing becomes too saturated, but this would be after a thorough assessment is completed.

The client is diagnosed with Crohn's disease, also known as regional enteritis. Which statement by the client supports this diagnosis? a. "My pain goes away when I have a bowel movement" b. "I have bright red blood in my stool all the time" c. "I have episodes of diarrhea and constipation" d. "My abdomen is hard and rigid and I have a fever"

**a. The terminal ileum is the most common site for regional enteritis and causes right lower quadrant pain that is relieved by defecation. b. Stools are liquid or semi-formed and usually do not contain blood. c. Episodes of diarrhea and constipation may be a sign/symptom of colon cancer, not Crohn's disease. d. A fever and hard rigid abdomen are signs/symptoms of peritonitis, a complication of Crohn's disease.

The client presents to the emergency department experiencing frequent watery, bloody stools after eating some undercooked meat at a fast food restaurant. Which intervention should be implemented first? a. Provide the client with a specimen collection hat to collect a stool sample. b. Initiate antibiotic therapy intravenously. c. Have the laboratory draw a complete blood count. d. Administer the antidiarrheal medication Lomotil.

**a. This client may have developed an infection from the undercooked meat. The nurse should try to get a specimen for the laboratory to analyze and for the nurse to be able to assess. The client's complaint of "bloody diarrhea" needs to be investigated by the nurse, who should observe the amount, color, and characteristics of the stool. b. Antibiotic therapy is initiated in only the most serious cases of infectious diarrhea; the diarrhea must be assessed first. A specimen for culture should be obtained, if possible, before beginning medication. c. A complete blood count will provide an estimate of blood loss, but it is not the first intervention. d. An antidiarrheal medication would be administered after the specimen collection.

The client has had a stool that is dark, watery, and shiny in appearance. Which intervention should be the nurse's first action? a. Check for a fecal impaction. b. Encourage the client to drink fluids. c. Check the chart for sodium and potassium levels. d. Apply a protective barrier cream to the perianal area.

**a. This is a symptom of diarrhea moving around an impaction higher up in the colon. The nurse should assess for an impaction when observing this finding. b. Encouraging the client to drink fluids should be done, but this is not the first intervention. c. The sodium level is usually not a problem for clients experiencing diarrhea, but the potassium level may be checked. However, again, this is not the first intervention. d. A protective cream can be applied to an excoriated perineum, but first the nurse should assess the situation.

Which laboratory value would the nurse expect to find indicating a chronic inflammation in the client with cholecystitis? a. An elevated white blood cell (WBC) count. b. A decreased lactate dehydrogenase (LDH) c. An elevated alkaline phosphatase. d. A decreased direct bilirubin level.

**a. This value would be elevated in clients with chronic inflammation. b. This value would indicate liver abnormalities. c. This value would indicate liver abnormalities. d. This value would indicate an obstructive process.

A nurse is caring for a client who has a new diagnosis of gastroesophageal reflux disease (GERD). The nurse should anticipate prescriptions by the provider for which of the following medications? (Select all that apply) a. Antacids b. Histamine2 receptor antagonists c. Opioid analgesics d. Fiber laxatives e. Proton pump inhibitors

a. Antacids b. Histamine2 receptor antagonists e. Proton pump inhibitors

A nurse is providing discharge teaching to a client who has a new prescription for aluminum hydroxide (Amphojel). The nurse should advise the client to a. Take the medication with food b. Monitor for diarrhea c. Wait 1 to 2 hr before taking other oral medications d. Maintain a low-fiber diet

c. Wait 1 to 2 hr before taking other oral medications

Which statement by a patient with dumping syndrome should lead the nurse to determine that further dietary teaching is needed? a. "I should eat bread and jam with every meal." b. "I should avoid drinking fluids with my meals." c. "I should eat smaller meals about six times a day." d. "I need to lie down for 30 to 60 minutes after my meals."

a. "I should eat bread and jam with every meal." Dietary control of dumping syndrome includes small, frequent meals with low carbohydrate content and elimination of fluids with meals. The patient should also lie down for 30 to 60 minutes after meals. These measures help to delay stomach emptying, preventing the rapid movement of a high-carbohydrate food bolus into the small intestine.

When evaluating the patient's understanding about the care of the ileostomy, what statement by the patient indicates the patient needs more teaching? a. "I will be able to regulate when I have stools." b. "I will be able to wear the pouch until it leaks." c. "Dried fruit and popcorn must be chewed very well." d. "The drainage from my stoma can damage my skin."

a. "I will be able to regulate when I have stools."

The nurse is caring for a patient who is 5'6" tall and weighs 186 lb. The nurse has discussed reasonable weight loss goals and a low-calorie diet with the patient. Which statement made by the patient indicates a need for further teaching? a. "I will limit intake to 500 calories a day." b. "I will try to eat very slowly during mealtimes." c. "I'll try to pick foods from all of the basic food groups." d. "It's important for me to begin a regular exercise program."

a. "I will limit intake to 500 calories a day."

A nurse in a clinic is teaching a client who has ulcerative colitis. Which of the following statements by the client indicates understanding of the teaching? a. "I will plan to limit fiber in my diet" b. "I will eat my meals and plan fluid intake between meals" c. "I will switch to black tea instead of drinking coffee" d. "I will try to eat three moderate to large meals a day"

a. "I will plan to limit fiber in my diet"

The patient with cirrhosis is being taught self-care. Which statement indicates the patient needs more teaching? a. "If I notice a fast heart rate or irregular beats, this is normal for cirrhosis." b. "I need to take good care of my belly and ankle skin where it is swollen." c. "A scrotal support may be more comfortable when I have scrotal edema." d. "I can use pillows to support my head to help me breathe when I am in bed."

a. "If I notice a fast heart rate or irregular beats, this is normal for cirrhosis."

The nurse teaches senior citizens at a community center how to prevent food poisoning at their informal social events. The nurse determines that teaching is successful if a community member makes which statement? a. "Pasteurized juices and milk are safe to drink." b. "Alfalfa sprouts are safe if rinsed before eating." c. "Fresh fruits do not need to be washed before eating." d. "Ground beef is safe to eat if cooked until it is brown."

a. "Pasteurized juices and milk are safe to drink."

The nurse instructs an obese 22-year-old man with a sedentary job about the health benefits of an exercise program. The nurse evaluates that teaching is effective when the patient makes which statement? a. "The goal is to walk at least 10,000 steps every day of the week." b. "Weekend aerobics for 2 hours is better than exercising every day." c. "Aerobic exercise will increase my appetite and result in weight gain." d. "Exercise causes weight loss by decreasing my resting metabolic rate."

a. "The goal is to walk at least 10,000 steps every day of the week."

The nurse is preparing to insert a nasogastric (NG) tube into a 68-year-old female patient who is nauseated and vomiting. She has an abdominal mass and suspected small intestinal obstruction. The patient asks the nurse why this procedure is necessary. What response by the nurse is most appropriate? a. "The tube will help to drain the stomach contents and prevent further vomiting." b. "The tube will push past the area that is blocked and thus help to stop the vomiting." c. "The tube is just a standard procedure before many types of surgery to the abdomen." d. "The tube will let us measure your stomach contents so that we can plan what type of IV fluid replacement would be best."

a. "The tube will help to drain the stomach contents and prevent further vomiting."

A nurse on a medical-surgical unit is admitting a client who has hepatitis B with ascites. Which of the following actions should the nurse include in the plan of care? a. Initiate contact precautions b. Weigh client weekly c. Measure abdominal girth 7.5 cm (3 in) above the umbilicus d. Provide a high-calorie, high-carbohydrate diet

d. Provide a high-calorie, high-carbohydrate diet

Which patient is at highest risk for having a gastric ulcer? a. 55-year-old female, smoker, with nausea and vomiting b. 45-year-old female admitted for illicit drug detoxification c. 37-year-old male, smoker, who fell while looking for a job d. 27-year old male who is being divorced and has back pain

a. 55-year-old female, smoker, with nausea and vomiting The 55-year-old female smoker experiencing nausea and vomiting is more likely to have a gastric ulcer. The other patients are not in the highest-risk age range or do not have enough risk factors. Although lower socioeconomic status, smoking, and drug use do increase the risk of gastric ulcers, these patients are more likely to experience duodenal ulcers but further assessment is needed.

The nurse is caring for a woman recently diagnosed with viral hepatitis A. Which individual should the nurse refer for an immunoglobin (IG) injection? a. A caregiver who lives in the same household with the patient b. A friend who delivers meals to the patient and family each week c. A relative with a history of hepatitis A who visits the patient daily d. A child living in the home who received the hepatitis A vaccine 3 months ago

a. A caregiver who lives in the same household with the patient

The nurse is caring for the following clients on a surgical unit. Which client would the nurse assess first? a. The client who had an inguinal hernia repair and has not voided in four (4) hours. b. The client who was admitted with abdominal pain who suddenly has no pain. c. The client four (4) hours postoperative abdominal surgery with no bowel sounds. d. The client who is one (1) day postoperative appendectomy who is being discharged.

a. A client who has not voided within four (4) hours after any surgery would not be priority. This is an acceptable occurrence, but if the client hasn't voided for eight (8) hours, then the nurse would assess further. **b. This could indicate a ruptured appendix, which could lead to peritonitis, a life-threatening complication; therefore, the nurse should assess this client first. c. Bowel sounds should return within 24 hours after abdominal surgery. Absent bowel sounds at four (4) hours postoperative would not be of great concern to the nurse. d. The client being discharged would be stable and not a priority for the nurse.

The nurse is caring for clients in an outpatient clinic. Which information should the nurse teach regarding the American Cancer Society's recommendations for the early detection of colon cancer? a. Beginning at age 60, a digital rectal exam should be done yearly. b. After the client reaches middle age, a yearly fecal occult test should be done. c. Have a colonoscopy at age 50 and then once every 5 to 10 years d. A flexible sigmoidoscopy should be done yearly after age 40.

a. A digital rectal exam is done to detect rectal cancer and should be started at age 40 years. b. "Middle age" is a relative term; specific ages are used for recommendation. **c. The American Cancer Society recommends a colonoscopy at age 50 and every five (5) to ten (10) years thereafter and a flexible sigmoidoscopy and barium enema every five (5) years. d. A flexible sigmoidoscopy should be done at five (5)-year intervals between the colonoscopy

Which disease is the client diagnosed with GERD at greater risk for developing? a. Hiatal hernia b. Gastroenteritis c. Esophageal cancer d. Gastric cancer

a. A hiatal hernia places the client at risk for GERD; GERD does not predispose the client for developing a hiatal hernia. b. Gastroenteritis is an inflammation of the stomach and intestine, usually caused by a virus. **c. Barrett's esophagitis results from long-term erosion of the esophagus as a result of reflux of stomach contents secondary to GERD. This is a precursor to esophageal cancer. d. The problems associated with GERD result from the reflux of acidic stomach contents into the esophagus, which is not a precursor to gastric cancer.

Which specific data should the nurse obtain from the client who is suspected of having peptic ulcer disease? a. History of side effects experienced from all medications b. Use of any NSAIDS c. Any known allergies to drugs and environmental factors d. Medical histories of at least 3 generations

a. A history of problems the client has experienced with medications is taken during the admission interview. This information does not specifically address peptic ulcer disease. **b. Use of NSAIDs places the client at risk for peptic ulcer disease and hemorrhage. Any client suspected of having peptic ulcer disease should be questioned specifically about the use of NSAIDs. c. In taking the history of clients, allergies are included for safety, but this is not specific for peptic ulcer disease. d. Information needs to be collected about past generations so that the nurse can analyze any potential health problems, but this is not specific for peptic ulcer disease.

When providing discharge teaching for the patient after a laparoscopic cholecystectomy, what information should the nurse include? a. A lower-fat diet may be better tolerated for several weeks. b. Do not return to work or normal activities for 3 weeks. c. Bile-colored drainage will probably drain from the incision. d. Keep the bandages on and the puncture site dry until it heals.

a. A lower-fat diet may be better tolerated for several weeks.

The nurse is caring for a client diagnosed with hemorrhaging duodenal ulcer. Which collaborative interventions should the nurse implement? (Select all that apply.) a. Perform a complete pain assessment b. Assess the client's vitals frequently c. Administer a proton pump inhibitor d. Obtain permission and administer blood products e. Monitor the intake of a soft bland diet

a. A pain assessment is an independent intervention that the nurse should implement frequently. b. Evaluating blood pressure is an independent intervention that the nurse should implement. If the client is able, B/Ps should be taken lying, sitting, and standing to assess for orthostatic hypotension **c. This is a collaborative intervention that the nurse should implement. It requires an order from the HCP. **d. Administering blood products is collaborative, requiring an order from the HCP. e. The diet needs an order by the health-care provider, but it would not be given to a client with a bleeding ulcer. These clients are allowed nothing by mouth until the bleeding stops.

The nurse is preparing to administer an aminoglycoside antibiotic to the client just admitted with a diagnosis of acute diverticulitis. Which intervention should the nurse implement? a. Obtain a serum trough level. b. Ask about drug allergies. c. Monitor the peak level. d. Assess the vital signs.

a. A peak and trough is drawn after the client has received at least three (3) to four (4) doses of medication, not on the initial dose because the client has just been admitted. **b. The nurse should always ask about allergies to medication when administering medications, but especially when administering antibiotics, which are notorious for allergic reactions. c. The peak and trough would not be drawn prior to the first dose; it is ordered after multiple doses. d. The nurse should question when to administer the medication, but there is no vital sign that would prevent the nurse from administering this medication.

The client diagnosed with ulcerative colitis has an ileostomy. Which statement indicates the client needs more teaching concerning the ileostomy? a. "My stoma should be pink and moist" b. "I will irrigate my ileostomy every am" c. "If I get a red, bumpy, itchy rash I will call my HCP" d. "I will change my pouch if it starts leaking"

a. A pink and moist stoma indicates viable tissue and adequate circulation. A purple stoma indicates necrosis. **b. An ileostomy will drain liquid all the time and should not routinely be irrigated; only specially trained nurses are allowed to irrigate an ileostomy. A sigmoid colostomy may need daily irrigation to evacuate feces. c. A red, bumpy, itchy rash indicates infection with the yeast Candida albicans, which should be treated with medication. d. The ileostomy drainage has enzymes and bile salts that are irritating and harsh to the skin; therefore, the pouch should be changed if any leakage occurs.

Which client would be most likely to have the diagnosis of diverticulosis? a. A 60-year-old male with a sedentary lifestyle. b. A 72-year-old female with multiple childbirths. c. A 63-year-old female with hemorrhoids. d. A 40-year-old male with a family history of diverticulosis.

a. A sedentary lifestyle may lead to obesity and contribute to hypertension or heart disease but usually not to diverticulosis. b. Multiple childbirths are not a risk factor for developing diverticulosis. **c. Hemorrhoids would indicate the client has chronic constipation, which is a strong risk factor for diverticulosis. Constipation increases the intraluminal pressure in the sigmoid colon, leading to weakness in the intestinal lining, which, in turn, causes outpouchings, or diverticula. d. A family history is not a risk factor. Having daily bowel movements and preventing constipation will decrease the chance of developing diverticulosis.

The nurse explains to the patient undergoing ostomy surgery that the procedure that maintains the most normal functioning of the bowel is a. A sigmoid colostomy. b. A transverse colostomy. c. A descending colostomy. d. An ascending colostomy.

a. A sigmoid colostomy.

The male client has had abdominal surgery and is now diagnosed with peritonitis. Which assessment data support the client's diagnosis of peritonitis? a. Absent bowel sounds and potassium level of 3.9 mEq/L. b. Abdominal cramping and hemoglobin of 14 gm/dL. c. Profuse diarrhea and stool specimen shows Campylobacter. d. Hard, rigid abdomen and white blood cell count 22,000 mm.

a. Absent bowel sounds would indicate a paralytic ileus, not peritonitis, and this is a normal potassium level (3.5 to 5.5 mEq/L). b. Abdominal cramping would not make the nurse suspect peritonitis, and the hemoglobin is normal (13-17 g/dL) c. Campylobacter is a cause of profuse diarrhea, but it does not support a diagnosis of peritonitis. **d. A hard, rigid abdomen indicates an inflamed peritoneum (abdominal wall cavity) resulting from an infection, which results in an elevated WBC level.

Regardless of the precipitating factor, what causes the injury to mucosal cells in peptic ulcers? a. Acid back diffusion into the mucosa b. The release of histamine from GI cells c. Ammonia formation in the mucosal wall d. Breakdown of the gastric mucosal barrier

a. Acid back diffusion into the mucosa The ultimate damage to the tissues of the stomach and duodenum, precipitating ulceration, is acid back diffusion into the mucosa. The gastric mucosal barrier is protective of the mucosa but without the acid environment and damage, ulceration does not occur. Ammonia formation by H. pylori and release of histamine impair the barrier but are not directly responsible for tissue injury.

Which type of gastritis is most likely to occur in a college student who has an isolated drinking binge? a. Acute gastritis b. Chronic gastritis c. Helicobacter pylori gastritis d. Autoimmune metaplastic atrophic gastritis

a. Acute gastritis Acute gastritis is most likely to occur with an isolated drinking binge. Chronic gastritis is usually caused by Heliobacter pylori or viral and fungal infections. Autoimmune gastritis is an inherited condition

The client is four (4) hours postoperative open cholecystectomy. Which data would warrant immediate intervention by the nurse? a. Absent bowel sounds in all four (4) quadrants. b. The T-tube with 60 mL of green drainage. c. Urine output of 100 mL in the past three (3) hours. d. Refusal to turn, deep breathe, and cough.

a. After abdominal surgery, it is not uncommon for bowel sounds to be absent. b. This is a normal amount and color of drainage. c. The minimum urine output is 30 mL/hr. **d. Refusing to turn, deep breathe, and cough puts the client at risk for pneumonia. This client needs immediate intervention to prevent complications.

The nurse is planning to teach the patient with gastroesophageal reflux disease (GERD) about foods or beverages that decrease lower esophageal sphincter (LES) pressure. What should be included in this list? (Select all that apply) a. Alcohol b. Root beer c. Chocolate d. Citrus fruits e. Fatty foods f. Cola sodas

a. Alcohol c. Chocolate e. Fatty foods f. Cola sodas Alcohol, chocolate, fatty foods, and cola sodas (caffeine) as well as peppermint and spearmint will decrease lower esophageal sphincter (LES) pressure. Root beer and herbal tea do not have caffeine. Citrus fruits will not affect LES pressure.

A postoperative patient has a nursing diagnosis of pain related to effects of medication and decreased GI motility as evidenced by abdominal pain and distention and inability to pass flatus. Which nursing intervention is most appropriate for this patient? a. Ambulate the patient more frequently. b. Assess the abdomen for bowel sounds. c. Place the patient in high Fowler's position. d. Withhold opioids because they decrease bowel motility.

a. Ambulate the patient more frequently. The abdominal pain and distention that occur from the decreased motility of the bowel should be treated with increased ambulation and frequent position changes to increase peristalsis. If the pain is severe, cholinergic drugs, rectal tubes, or application of heat to the abdomen may be prescribed. Assessment of bowel sounds is not an intervention to relieve the pain and a high Fowler's position is not indicated. Opioids may still be necessary for pain control and motility can be increased by other means.

The client being admitted from the emergency department is diagnosed with a fecal impaction. Which nursing intervention should be implemented? a. Administer an antidiarrheal medication, every day and PRN. b. Perform bowel training every two (2) hours. c. Administer oil retention enemas. d. Prepare for an upper gastrointestinal (UGI) series x-ray.

a. An antidiarrheal medication would slow down the peristalsis in the colon, worsening the problem. b. The client has an immediate need to evacuate the bowel, not bowel training. **c. Oil retention enemas will help to soften the feces and evacuate the stool. d. A UGI series would add barium to the already hardened stool in the colon. Barium enemas x-ray the colon; a UGI x-rays the stomach and jejunum.

Which medication would the nurse expect the health-care provider to order to treat the client diagnosed with botulism secondary to eating contaminated canned goods? a. An antidiarrheal medication. b. An aminoglycoside antibiotic. c. An antitoxin medication. d. An ACE inhibitor medication.

a. Antidiarrheal medications are contraindicated with botulism because the toxin needs to be expelled from the body. A laxative may be ordered to promote removal of the toxin from the bowel. b. Aminoglycoside antibiotics will not be ordered because there is no bacterium with botulism; it is a toxin. **c. A botulism antitoxin neutralizes the circulating toxin and is prescribed for a client with botulism. d. An ACE inhibitor is prescribed for a client diagnosed with cardiovascular disease

The client diagnosed with IBD is prescribed sulfasalazine (Asulfidine), a sulfonamide antibiotic. Which statement best describes the rationale for administering this medication? a. It is administered rectally to help decrease colon inflammation b. This medication slows GI motility and reduced diarrhea c. This medication kills the bacteria causing the exacerbation d. It acts topically on the colon mucosa to decrease inflammation

a. Asulfidine cannot be administered rectally. Corticosteroids may be administered by enema for the local effect of decreasing inflammation but minimizing the systemic effects. b. Antidiarrheal agents slow the gastrointestinal motility and reduce diarrhea. c. IBD is not caused by bacteria. **d. This antibiotic is poorly absorbed from the gastrointestinal tract and acts topically on the colonic mucosa to inhibit the inflammatory process.

The nurse is instructing a patient with chronic pancreatitis on measures to prevent further attacks. What information should be provided (select all that apply)? a. Avoid nicotine. b. Eat bland foods. c. Observe stools for steatorrhea. d. Eat high-fat, low-protein, high-carbohydrate meals. e. Take prescribed pancreatic enzymes immediately following meals.

a. Avoid nicotine. b. Eat bland foods. c. Observe stools for steatorrhea. Measures to prevent attacks of pancreatitis are those that decrease the stimulation of the pancreas. Lower fat intake and foods that are less stimulating and irritating (bland) should be encouraged. Higher carbohydrates are less stimulating. Avoid alcohol and nicotine, since both stimulate the pancreas. Monitor for steatorrhea to determine effectiveness of the enzymes and because it may indicate worsening pancreatic function. Pancreatic enzymes should be taken with, not after, meals.

What information should be included in the dietary teaching for the patient following a Roux-en-Y gastric bypass? a. Avoid sugary foods and limit fluids to prevent dumping syndrome b. Gradually increase the amount of food ingested to preoperative levels c. Maintain a long-term liquid diet to prevent damage to the surgical site d. Consume foods high in complex carbohydrates, protein, and fiber to add bulk to contents

a. Avoid sugary foods and limit fluids to prevent dumping syndrome Fluids and foods high in carbohydrates tend to promote diarrhea and symptoms of gastric syndrome in patients with gastric bypass surgery. The diet generally should be high in protein and low in carbohydrates, fat, and roughage and consist of six small feedings a day because of the small stomach size. Liquid diets are likely to be used longer for the patient with a gastroplasty.

Which statement made by the client admitted with electrolyte imbalance from frequent cathartic use demonstrates an understanding of the discharge teaching? a. "In the future I will eat a banana every time I take the medication." b. "I don't have to have a bowel movement every day." c. "I should limit the fluids I drink with my meals." d. "If I feel sluggish, I will eat a lot of cheese and dairy products."

a. Bananas are encouraged for clients with potassium loss from diuretics; a banana is not needed for harsh laxative (cathartic) use. Harsh laxatives should be discouraged because they cause laxative dependence and a narrowing of the colon with long-term use. **b. It is not necessary to have a bowel movement every day to have normal bowel functioning. c. Limiting fluids will increase the problem; the client should be encouraged to increase the fluids in the diet. d. If the client were feeling "sluggish" from not being able to have a bowel movement, these foods would increase constipation because they are low in residue (fiber).

A nurse is reviewing nutrition teaching for a client who has cholecystitis. Which of the following food choices can trigger cholecystitis? a. Brownie with nuts b. Bowl of mixed fruit c. Grilled turkey d. Baked potato

a. Brownie with nuts

A patient with a gastric outlet obstruction has been treated with NG decompression. After the first 24 hours, the patient develops nausea and increased upper abdominal bowel sounds. What is the best action by the nurse? a. Check the patency of the NG tube. b. Place the patient in a recumbent position. c. Assess the patient's vital signs and circulatory status. d. Encourage the patient to deep breathe and consciously relax.

a. Check the patency of the NG tube. If symptoms of gastric outlet obstruction, such as nausea, vomiting, and stomach distention, occur while the patient is on NPO status or has an NG tube, the patency of the NG tube should be assessed. A recumbent position should not be used in a patient with a gastric outlet obstruction because it increases abdominal pressure on the stomach and vital signs and circulatory status assessment are important if hemorrhage or perforation is suspected. Deep breathing and relaxation may help some patients with nausea but not when stomach contents are obstructed from flowing into the small intestine.

Nursing management of the patient with acute pancreatitis includes (select all that apply) a. Checking for signs of hypocalcemia. b. Providing a diet low in carbohydrates. c. Giving insulin based on a sliding scale. d. Observing stools for signs of steatorrhea. e. Monitoring for infection, particularly respiratory tract infection.

a. Checking for signs of hypocalcemia. e. Monitoring for infection, particularly respiratory tract infection.

The dietician and nurse in a long-term care facility are planning the menu for the day. Which foods would be recommended for the immobile clients for whom swallowing is not an issue? a. Cheeseburger and milk shake. b. Canned peaches and a sandwich on whole-wheat bread. c. Mashed potatoes and mechanically ground red meat. d. Biscuits and gravy with bacon.

a. Cheeseburgers and milk shakes are low residue foods that can make constipation worse. **b. Canned peaches are soft and can be chewed and swallowed easily while providing some fiber, and whole-wheat bread is higher in fiber than white bread. These foods will be helpful for clients whose gastric motility is slowed as a result of lack of exercise or immobility. c. These foods do not provide the needed fiber. d. These are refined flour foods or processed meat (fat). These will not help clients to prevent constipation

The client is in the preicteric phase of hepatitis. Which signs/symptoms would the nurse expect the client to exhibit during this phase? a. Clay-colored stools and jaundice. b. Normal appetite and pruritus. c. Being afebrile and left upper quadrant pain. d. Complaints of fatigue and diarrhea.

a. Clay-colored stools and jaundice occur in the icteric phase of hepatitis. b. These signs/symptoms occur in the icteric phase of hepatitis. c. Fever subsides in the icteric phase, and the pain is in the right upper quadrant. **d. "Flu-like" symptoms are the first complaints of the client in the preicteric phase of hepatitis, which is the initial phase and may begin abruptly or insidiously.

The male client tells the nurse he has been experiencing "heartburn" at night that awakens him. Which assessment question should the nurse ask? a. "How much weight have you gained recently?" b. "What have you done to alleviate the heartburn?" c. "Do you consume many milk and dairy products?" d. "Have you been around anyone with a stomach virus?"

a. Clients with heartburn are frequently diagnosed as having gastroesophageal reflux disease (GERD). GERD can occasionally cause weight loss, but not weight gain. **b. Most clients with GERD have been self-medicating with over-the-counter medications prior to seeking advice from a health-care provider. It is important to know what the client has been using to treat the problem. c. Milk and dairy products contain lactose, which would be important if considering lactose intolerance, but it is not important for "heartburn." d. Heartburn is not a symptom of a viral illness

The nurse teaches a 50-year-old woman who has a body mass index (BMI) of 39 kg/m2 about weight loss. Which dietary change would be appropriate for the nurse to recommend to this patient? a. Decrease fat intake and control portion size b. Increase vegetables and decrease fluid intake c. Increase protein intake and avoid carbohydrates d. Decrease complex carbohydrates and limit fiber

a. Decrease fat intake and control portion size

The nurse has administered an antibiotic, a proton pump inhibitor, and Pepto-Bismol for peptic ulcer disease secondary to H. pylori. Which data would indicate to the nurse that the medications are effective? a. A decrease in alcohol intake. b. Maintaining a bland diet. c. A return to previous activities. d. A decrease in gastric distress.

a. Decreasing the alcohol intake indicates that the client is making some lifestyle changes. b. Maintaining a bland diet would indicate that dietary restrictions are being followed. c. The return to previous activities would indicate that the client has not adapted to the lifestyle changes and has returned to the previous behaviors that precipitated the peptic ulcer disease. **d. Antibiotics, proton pump inhibitors, and Pepto-Bismol are administered to decrease the irritation of the ulcerative area and cure the ulcer. A decrease in gastric distress indicates the medication is effective.

A nurse is caring for a client who has cirrhosis. Which of the following medications can the nurse expect to administer to this client? (Select all that apply) a. Diuretic b. Beta-blocking agent c. Opioid analgesic d. Lactulose (Cephulac) e. Sedative

a. Diuretic b. Beta-blocking agent d. Lactulose (Cephulac)

The school nurse is discussing ways to prevent an outbreak of hepatitis A with a group of high school teachers. Which action is the most important intervention that the school nurse must explain to the school teachers? a. Do not allow students to eat or drink after each other. b. Drink bottled water as much as possible. c. Encourage protected sexual activity. d. Thoroughly wash hands.

a. Eating after each other should be discouraged, but it is not the most important intervention. b. Only bottled water should be consumed in Third World countries, but that precaution is not necessary in American high schools. c. Hepatitis B and C, not hepatitis A, are trans- mitted by sexual activity. **d. Hepatitis A is transmitted via the fecal-oral route. Good hand washing helps to prevent its spread.

A nurse is caring for a client who has a small bowel obstruction from adhesions. Which of the following findings are consistent with this diagnosis? (Select all that apply) a. Emesis greater than 500 mL with a fecal odor b. Report of spasmodic abdominal pain c. Pain relieved with vomiting d. Abdomen flat with rebound tenderness to palpation e. Laboratory findings indicate metabolic acidosis

a. Emesis greater than 500 mL with a fecal odor b. Report of spasmodic abdominal pain c. Pain relieved with vomiting

The nurse plans teaching for the patient with a colostomy but the patient refuses to look at the nurse or the stoma, stating, "I just can't see myself with this thing." What is the best nursing intervention for this patient? a. Encourage the patient to share concerns and ask questions. b. Refer the patient to a chaplain to help cope with this situation. c. Explain that there is nothing the patient can do about it and must take care of it. d. Tell the patient that learning about it will prevent stool leaking and the sounds of flatus.

a. Encourage the patient to share concerns and ask questions. Encouraging the patient to share concerns and ask questions will help the patient to begin to adapt to living with the colostomy. The other options do not support the patient and do not portray the nurse's focus on helping the patient or treating the patient as an individual.

When preparing a patient for a capsule endoscopy study, what should the nurse do? a. Ensure the patient understands the required bowel preparation. b. Have the patient return to the procedure room for removal of the capsule. c. Teach the patient to maintain a clear liquid diet throughout the procedure. d. Explain to the patient that conscious sedation will be used during placement of the capsule.

a. Ensure the patient understands the required bowel preparation.

The nurse has received the A.M. shift report. Which client should the nurse assess first? a. The 44-year-old client diagnosed with peptic ulcer disease who is complaining of acute epigastric pain. b. The 74-year-old client diagnosed with acute gastroenteritis who has had four (4) diarrhea stools during the night. c. The 65-year-old client diagnosed with inflammatory bowel disease who has a hard, rigid abdomen and elevated temperature. d. The 15-year-old client diagnosed with food poisoning who has vomited several times during the night shift.

a. Epigastric pain is expected in a client diagnosed with peptic ulcer disease. b. Four (4) diarrheal stools would not be unusual in a client diagnosed with gastroenteritis. **c. A hard, rigid abdomen and an elevated temperature are abnormal in any circumstance and the nurse should assess this client first. These are clinical manifestations of peritonitis, a potentially life-threatening condition. d. Vomiting is expected in a client diagnosed with food poisoning.

Which medications are used to decrease gastric or hydrochloric acid secretion (select all that apply)? a. Famotidine (Pepcid) b. Sucralfate (Carafate) c. Omeprazole (Prilosec) d. Misoprostol (Cytotec) e.Amoxicillin/clarithromycin/omeprazole

a. Famotidine (Pepcid) c. Omeprazole (Prilosec) d. Misoprostol (Cytotec) e. Amoxicillin/clarithromycin/omeprazole Famotidine (Pepcid) reduces HCl secretion by blocking histamine and omeprazole (Prilosec) decreases gastric acid secretion by blocking adenosine triphosphatase (ATPase) enzyme. Sucralfate (Carafate) coats the ulcer to protect it from acid erosion. Misoprostol (Cytotec) mixture has antisecretory effects. Amoxicillin/ clarithromycin/omeprazole are used in patients with verified H. pylori.

Acalculous cholecystitis is diagnosed in an older, critically ill patient. Which factors may be associated with this condition (select all that apply)? a. Fasting b. Hypothyroidism c. Parenteral nutrition d. Prolonged immobility e. Streptococcus pneumoniae f. Absence of bile in the intestine

a. Fasting c. Parenteral nutrition d. Prolonged immobility Acalculous cholecystitis is associated with prolonged immobility, fasting, prolonged parenteral nutrition, and diabetes mellitus. Hypothyroidism, Streptococcus pneumoniae, and absence of bile in the intestine are unrelated to this condition.

In planning care for a patient with metastatic liver cancer, the nurse should include interventions that a. Focus primarily on symptomatic and comfort measures. b. Reassure the patient that chemotherapy offers a good prognosis. c. Promote the patient's confidence that surgical excision of the tumor will be successful. d. Provide information necessary for the patient to make decisions regarding liver transplantation.

a. Focus primarily on symptomatic and comfort measures.

What are characteristics of gingivitis? a. Formation of abscesses with loosening of teeth b. Caused by upper respiratory tract viral infection c. Shallow, painful vesicular ulcerations of lips and mouth d. Infectious ulcers of mouth and lips as a result of systemic disease

a. Formation of abscesses with loosening of teeth When gingivitis is untreated, abscesses form and teeth are loosened with periodontitis. Herpes simplex is the viral infection related to the upper respiratory system and has shallow, painful vesicular ulcerations of lips and mouth. Aphthous stomatitis has infectious ulcers of the mouth and lips with a defined erythematous base occurring as a result of systemic disease.

The nurse is discussing the therapeutic diet for the client diagnosed with diverticulosis. Which meal indicates the client understands the discharge teaching? a. Fried fish, mashed potatoes, and iced tea. b. Ham sandwich, applesauce, and whole milk. c. Chicken salad on whole-wheat bread and water. d. Lettuce, tomato, and cucumber salad and coffee.

a. Fried foods increase cholesterol. Mashed potatoes do not have the peel, which is needed for increased fiber. b. Applesauce does not have the peel, which is needed for increased fiber, and the option does not identify which type of bread; whole milk is high in fat. **c. Chicken and whole-wheat bread are high in fiber, which is the therapeutic diet prescribed for clients with diverticulosis. An adequate intake of water helps prevent constipation. d. Tomatoes and cucumbers both have seeds, and many health-care providers recommend that people with diverticulosis avoid seeds because of the possibility of the seeds entering the diverticulum and becoming trapped, leading to peritonitis.

When a 35-year-old female patient is admitted to the emergency department with acute abdominal pain, which possible diagnosis should you consider that may be the cause of her pain (select all that apply)? a. Gastroenteritis b. Ectopic pregnancy c. Gastrointestinal bleeding d. Irritable bowel syndrome e. Inflammatory bowel disease

a. Gastroenteritis b. Ectopic pregnancy c. Gastrointestinal bleeding d. Irritable bowel syndrome e. Inflammatory bowel disease

The client diagnosed with gastroenteritis is being discharged from the emergency department. Which intervention should the nurse include in the discharge teaching? a. If diarrhea persists for more than 96 hours, contact the physician. b. Instruct the client to wash hands thoroughly before handling any type of food. c. Explain the importance of decreasing steroids gradually as instructed. d. Discuss how to collect all stool samples for the next 24 hours.

a. If the diarrhea persists more than 48 hours, notify the physician. Diarrhea for more than 96 hours could lead to metabolic acidosis, hypokalemia, and possible death. **b. This should be done by the client at all times, but especially when the client has gastroenteritis. The bacteria in feces may be transferred to other people via food if hands are not washed properly c. Steroids are not used in the treatment of gastroenteritis; antidiarrheal medication is usually prescribed. d. The client may be asked to provide a stool specimen for culture, ova, parasites, and fecal leukocytes, but the client would not be asked for a 24-hour stool collection.

The client with a new colostomy is being discharged. Which statement made by the client indicates the need for further teaching? a. "If I notice any skin breakdown I will call the HCP." b. "I should drink only liquids until the colostomy starts to work." c. "I should not take a tub bath until the HCP okays it." d. "I should not drive or lift more than five (5) pounds."

a. If the tissue around the stoma becomes excoriated, the client will be unable to pouch the stoma adequately, resulting in discomfort and leakage. The client understands the teaching. **b. The client should be on a regular diet, and the colostomy will have been working for several days prior to discharge. The client's statement indicates the need for further teaching. c. Until the incision is completely healed the client should not sit in bath water because of the potential contamination of the wound by the bath water. The client understands the teaching. d. The client has had major surgery and should limit lifting to minimal weight. The client understands the teaching.

The appropriate collaborative therapy for the patient with acute diarrhea caused by a viral infection is to a. Increase fluid intake. b. Administer an antibiotic. c. Administer antimotility drugs. d. Quarantine the patient to prevent spread of the virus.

a. Increase fluid intake.

A patient is admitted to the emergency department with acute abdominal pain. What nursing intervention should the nurse implement first? a. Measurement of vital signs b. Administration of prescribed analgesics c. Assessment of the onset, location, intensity, duration, and character of the pain d. Physical assessment of the abdomen for distention, bowel sounds, and pigmentation changes

a. Measurement of vital signs The patient with an acute abdomen may have significant fluid or blood loss into the abdomen and evaluation of blood pressure (BP) and heart rate (HR) should be the first intervention, followed by assessment of the abdomen and the nature of the pain. Analgesics should be used cautiously until a diagnosis can be determined so that symptoms are not masked.

The nurse writes a psychosocial problem of "risk for altered sexual functioning related to new colostomy." Which intervention should the nurse implement? a. Tell the client that there should be no intimacy for at least three (3) months. b. Ensure that the client and significant other are able to change the ostomy pouch. c. Demonstrate with charts possible sexual positions for the client to assume. d. Teach the client to protect the pouch from becoming dislodged during sex

a. Intimacy involves more than sexual intercourse. The client can be sexually active whenever the wounds are healed sufficiently to not cause pain. b. This is an appropriate nursing intervention for home care, but it has nothing to do with sexual activity. c. The nurse is not a sexual counselor who would have these types of charts. The nurse should address sexuality with the client but would not be considered an expert capable of explaining the advantages and disadvantages of sexual positioning. **d. A pouch that becomes dislodged during the sexual act would cause embarrassment for the client whose body image has already been dealt a blow.

The patient with suspected gallbladder disease is scheduled for an ultrasound of the gallbladder. What should the nurse explain to the patient about this test? a. It is noninvasive and is a very reliable method of detecting gallstones. b. It is used only when other tests cannot be used because of allergy to contrast media. c. It will outline the gallbladder and the ductal system to enable visualization of stones. d. It is an adjunct to liver function tests to determine whether the gallbladder is inflamed.

a. It is noninvasive and is a very reliable method of detecting gallstones. Ultrasonography is 90% to 95% accurate in detecting gallstones and is a noninvasive procedure. An IV cholangiogram uses radiopaque dye to outline the gallbladder and the ducts. Liver function tests will be elevated if liver damage has occurred but do not indicate gallbladder disease.

Which assessment question would be priority for the nurse to ask the client diagnosed with end-stage liver failure secondary to alcoholic cirrhosis? a. "How many years have you been drinking alcohol?" b. "Have you completed an advanced directive?" c. "When did you have your last alcoholic drink?" d. "What foods did you eat at your last meal?"

a. It really doesn't matter how long the client has been drinking alcohol. The diagnosis of alcoholic cirrhosis indicates the client has probably been drinking for many years. b. An advance directive is important for the client who is terminally ill, but it is not the priority question. **c. The nurse must know when the client had the last alcoholic drink to be able to determine when and if the client will experience delirium tremens, the physical withdrawal from alcohol. d. This is not a typical question asked by the nurse unless the client is malnourished, which is not information given in the stem.

What must the nurse do to care for a T-tube in a patient following a cholecystectomy? a. Keep the tube supported and free of kinks. b. Attach the tube to low, continuous suction. c. Clamp the tube when ambulating the patient. d. Irrigate the tube with 10-mL sterile saline every 2 to 4 hours.

a. Keep the tube supported and free of kinks. The T-tube drains bile from the common bile duct until swelling from trauma has subsided and bile can freely enter the duodenum. The tube is placed to gravity drainage and should be kept open and free from kinks to prevent bile from backing up into the liver. The tube is not normally irrigated.

A nurse is teaching a client who has hepatitis B about home care. Which of the following should the nurse include in teaching? (Select all that apply) a. Limit physical activity b. Avoid alcohol c. Take acetaminophen for comfort d. Wear a mask when in public places e. Eat small frequent meals

a. Limit physical activity b. Avoid alcohol e. Eat small frequent meals

The nurse is conducting discharge teaching for a patient with metastatic lung cancer who was admitted with a bowel impaction. Which instructions would be most helpful to prevent further episodes of constipation? a. Maintain a high intake of fluid and fiber in the diet. b. Reduce intake of medications causing constipation. c. Eat several small meals per day to maintain bowel motility. d. Sit upright during meals to increase bowel motility by gravity.

a. Maintain a high intake of fluid and fiber in the diet.

The client with a history of peptic ulcer disease has been admitted into the hospital intensive care unit with frank gastric bleeding. Which priority intervention should the nurse implement? a. Maintain a strict record of intake and output. b. Insert a nasogastric tube and begin saline lavage. c. Assist the client with keeping a detailed calorie count. d. Provide a quiet environment to promote rest.

a. Maintaining a strict record of intake and output is important to evaluate the progression of the client's condition, but it is not the most important intervention. **b. Inserting a nasogastric tube and lavaging the stomach with saline is the most important intervention because this directly stops the bleeding. c. A calorie count is important information that can assist in the prevention and treatment of a nutritional deficit, but this intervention does not address the client's immediate and life-threatening problem. d. Promoting a quiet environment aids in the reduction of stress, which can cause further bleeding, but this will not stop the bleeding.

A patient with ulcerative colitis has a total proctocolectomy with formation of a terminal ileum stoma. What is the most important nursing intervention for this patient postoperatively? a. Measure the ileostomy output to determine the status of the patient's fluid balance. b. Change the ileostomy appliance every 3 to 4 hours to prevent leakage of drainage onto the skin. c. Emphasize that the ostomy is temporary and the ileum will be reconnected when the large bowel heals. d. Teach the patient about the high-fiber, low-carbohydrate diet required to maintain normal ileostomy drainage.

a. Measure the ileostomy output to determine the status of the patient's fluid balance. Initial output from a newly formed ileostomy may be as high as 1500 to 2000 mL daily and intake and output must be accurately monitored for fluid and electrolyte imbalance. Ileostomy bags may need to be emptied every 3 to 4 hours but the appliance should not be changed for several days unless there is leakage onto the skin. A terminal ileum stoma is permanent and the entire colon has been removed. A return to a normal, presurgical diet is the goal for the patient with an ileostomy, with restrictions based only on the patient's individual tolerances.

Duodenal and gastric ulcers have similar as well as differentiating features. What are characteristics unique to duodenal ulcers? Select all that apply. a. Pain is relieved with eating food. b. They have a high recurrence rate. c. Increased gastric secretion occurs. d. Associated with Helicobacter pylori infection. e. Hemorrhage, perforation, and obstruction may result. f. There is burning and cramping in the midepigastric area.

a. Pain is relieved with eating food. c. Increased gastric secretion occurs. f. There is burning and cramping in the midepigastric area.

The post-anesthesia care nurse is caring for a client who has had abdominal surgery. The client is complaining of nausea. Which intervention should the nurse implement first? a. Medicate the client with a narcotic analgesic IVP. b. Assess the nasogastric tube for patency. c. Check the temperature for elevation. d. Hyperextend the neck to prevent stridor.

a. Medicating the client with an analgesic could increase the client's nausea unless the nausea is caused by pain. The nurse should assess the etiology to determine the interventions. **b. A client who has had abdominal surgery usually has a nasogastric tube (NGT) in place. If the NGT is not patent, this will cause nausea. Irrigating the NGT may relieve nausea. c. Checking the temperature will not treat the nausea. d. Hyperextending the neck will assist the client to breathe but will not treat nausea.

What type of bleeding will a patient with peptic ulcer disease with a slow upper GI source of bleeding have? a. Melena b. Occult blood c. Coffee-ground emesis d. Profuse bright-red hematemesis

a. Melena Melena is black, tarry stools from slow bleeding from an upper GI source when blood passes through the GI tract and is digested. Occult blood is the presence of guaiac-positive stools or gastric aspirate. Coffee-ground emesis is blood that has been in the stomach for some time and has reacted with gastric secretions. Profuse bright-red hematemesis is arterial blood that has not been in contact with gastric secretions, as in esophageal or oral bleeding.

An 82-year-old man is admitted with an acute attack of diverticulitis. What should the nurse include in his care? a. Monitor for signs of peritonitis. b. Treat with daily medicated enemas. c. Prepare for surgery to resect the involved colon. d. Provide a heating pad to apply to the left lower quadrant.

a. Monitor for signs of peritonitis. The inflammation and infection of diverticula cause small perforations with spread of the inflammation to the surrounding area in the intestines. Abscesses may form or complete perforation with peritonitis may occur. Systemic antibiotic therapy is often used but medicated enemas would increase intestinal motility and increase the possibility of perforation, as would the application of heat. Surgery is only necessary to drain abscesses or to resect an obstructing inflammatory mass.

For the patient hospitalized with inflammatory bowel disease (IBD), which treatments would be used to rest the bowel (select all that apply)? a. NPO b. IV fluids c. Bed rest d. Sedatives e. Nasogastric suction f. Parenteral nutrition

a. NPO b. IV fluids e. Nasogastric suction f. Parenteral nutrition With an acute exacerbation of inflammatory bowel disease (IBD), to rest the bowel the patient will be NPO, receive IV fluids and parenteral nutrition, and have nasogastric suction. Sedatives would be used to alleviate stress. Enteral nutrition will be used as soon as possible

What treatment for acute cholecystitis will prevent further stimulation of the gallbladder? a. NPO with NG suction b. Incisional cholecystectomy c. Administration of antiemetics d. Administration of anticholinergics

a. NPO with NG suction NPO and nasogastric (NG) suction prevent gallbladder stimulation by food or fluids moving into the duodenum. Incisional choleycystectomy removes the gallbladder, not its stimulation. Administration of antiemetics decreases nausea and vomiting but does not decrease gallbladder stimulation. Anticholinergics counteract the smooth muscle spasms of the bile ducts to decrease pain.

Of the following characteristics, identify those that are most commonly associated with cholelithiasis (select all that apply). a. Obesity b. Age over 40 c. Multiparous female d. History of excessive alcohol intake e. Family history of gallbladder disease f. Use of estrogen or oral contraceptives

a. Obesity b. Age over 40 c. Multiparous female e. Family history of gallbladder disease f. Use of estrogen or oral contraceptives Incidence of cholelithiasis is higher in women, multiparous women, persons over 40 years of age, and those with family history and obesity. Postmenopausal women taking estrogen therapy have a higher incidence than women taking oral contraceptives. Alcohol intake and diet do not increase the incidence of cholelithiasis.

A nurse is planning care for a client who has a new prescription for total parenteral nutrition (TPN). Which of the following interventions should be included in the plan of care? (Select all that apply) a. Obtain a capillary blood glucose four times daily b. Administer prescribed medications through a secondary port on the TPN IV tubing c. Monitor vital signs three times during the 12-hr shift d. Change the TPN IV tubing every 24 hours e. Ensure a daily aPTT is obtained

a. Obtain a capillary blood glucose four times daily c. Monitor vital signs three times during the 12-hr shift d. Change the TPN IV tubing every 24 hours

Malnutrition can be a big problem for patients with cirrhosis. Which nursing intervention can help to improve nutrient intake? a. Oral hygiene before meals and snacks b. Provide all foods the patient likes to eat c. Improve oral intake by feeding the patient d. Limit snack offers to when the patient is hungry

a. Oral hygiene before meals and snacks Oral hygiene may improve the patient's taste sensation. Food preferences are important but some foods may be restricted if the patient is on a low-sodium diet. The patient will feel more independent with self-feeding and will be more likely to increase intake by having someone sit with the patient while the patient eats. Snacks and supplements should be available whenever the patient desires them but should not be forced on the patient.

A 50-year-old African American woman has a BMI of 35 kg/m2, type 2 diabetes mellitus, hypercholesterolemia, and irritable bowel syndrome (IBS). She is seeking assistance in losing weight, because, "I have trouble stopping eating when I should, but I do not want to have bariatric surgery." Which drug therapy should the nurse question if it is prescribed for this patient? a. Orlistat (Xenical) b. Locaserin (Belviq) c. Phentermine (Adipex-P) d. Phentermine and topiramate (Qsymia)

a. Orlistat (Xenical)

Which statement made by the client indicates to the nurse the client may be experiencing GERD? a. "My chest hurts when I walk up the stairs in my home" b. "I take antacid tablets with me wherever I go" c. "My spouse tells me I snore very loudly at night" d. "I drink 6 to 7 soft drinks a day"

a. Pain in the chest when walking up stairs would indicate angina. **b. Frequent use of antacids indicates an acid reflux problem. c. Snoring loudly could indicate sleep apnea, but not GERD. d. Carbonated beverages increase stomach pressure. Six (6) to seven (7) soft drinks a day would not be tolerated by a client with GERD.

A patient with inflammatory bowel disease has a nursing diagnosis of imbalanced nutrition: less than body requirements related to decreased nutritional intake and decreased intestinal absorption. Which assessment data support this nursing diagnosis? a. Pallor and hair loss b. Frequent diarrhea stools c. Anorectal excoriation and pain d. Hypotension and urine output below 30 mL/hr

a. Pallor and hair loss Signs of malnutrition include pallor from anemia, hair loss, bleeding, cracked gingivae, and muscle weakness, which support a nursing diagnosis that identifies impaired nutrition. Diarrhea may contribute to malnutrition but is not a defining characteristic. Anorectal excoriation and pain relate to problems with skin integrity. Hypotension relates to problems with fluid deficit.

The nurse is administering morning medications at 0730. Which medication should have priority? a. A proton pump inhibitor b. A nonnarcotic analgesic c. A histamine receptor antagonist d. A mucosal barrier agent

a. Proton pump inhibitors can be administered at routine dosing times, usually 0900 or after breakfast. b. Pain medication is important, but a nonnarcotic medication, such as Tylenol, can be administered after a medication that must be timed. c. A histamine receptor antagonist can be administered at routine dosing times. **d. A mucosal barrier agent must be administered on an empty stomach for the medication to coat the stomach.

A patient with a history of peptic ulcer disease has presented to the emergency department reporting severe abdominal pain and has a rigid, boardlike abdomen that prompts the health care team to suspect a perforated ulcer. What intervention should the nurse anticipate? a. Providing IV fluids and inserting a nasogastric (NG) tube b. Administering oral bicarbonate and testing the patient's gastric pH level c. Performing a fecal occult blood test and administering IV calcium gluconate d. Starting parenteral nutrition and placing the patient in a high-Fowler's position

a. Providing IV fluids and inserting a nasogastric (NG) tube

What is one of the most challenging nursing interventions to promote healing in the patient with viral hepatitis? a. Providing adequate nutritional intake b. Promoting strict bed rest during the icteric phase c. Providing pain relief without using liver-metabolized drugs d. Providing quiet diversional activities during periods of fatigue

a. Providing adequate nutritional intake Adequate nutrition is especially important in promoting regeneration of liver cells but the anorexia of viral hepatitis is often severe, requiring creative and innovative nursing interventions. Strict bed rest is not usually required, and the patient usually has only minor discomfort with hepatitis. Diversional activities may be required to promote psychologic rest but not during periods of fatigue.

A patient with hepatitis A is in the acute phase. The nurse plans care for the patient based on the knowledge that a. Pruritus is a common problem with jaundice in this phase. b. The patient is most likely to transmit the disease during this phase. c. Gastrointestinal symptoms are not as severe in hepatitis A as they are in hepatitis B. d. Extrahepatic manifestations of glomerulonephritis and polyarteritis are common in this phase.

a. Pruritus is a common problem with jaundice in this phase.

The charge nurse is making assignments. Staffing includes a registered nurse with 5 years of medical-surgical experience, a newly graduated registered nurse, and 2 unlicensed assistive personnel (UAPs). Which client should be assigned to the most experienced nurse? a. The 39-year-old client diagnosed with lower esophageal dysfunction who is complaining of pyrosis b. The 54-year-old client diagnosed with Barrett's esophagus who is scheduled to have an endoscopy this morning c. The 46-year-old client diagnosed with gastroesophageal reflux disease who has wheezes in all five lobes d. The 68-year-old client who is 3 days postoperative for hiatal hernia and needs to be ambulated 4 times today

a. Pyrosis is heartburn and is expected in a client diagnosed with GERD. The new graduate can care for this client. b. Barrett's esophagitis is a complication of GERD; new graduates can prepare a client for a diagnostic procedure. **c. This client is exhibiting symptoms of asthma, a complication of GERD; therefore, the client should be assigned to the most experienced nurse. d. This client can be cared for by the new graduate, and ambulating can be delegated to the unlicensed nursing assistant.

A nurse is caring for a client who is receiving TPN solution. It has been 24 hr since the current bag of solution was hung, and 400 mL remains to infuse. Which of the following is the appropriate action for the nurse to take? a. Remove the current bag and hang a new bag b. Infuse the remaining solution at the current rate and then hang a new bag c. Increase the infusion rate so the remaining solution is administered within the hour and hang a new bag d. Remove the current bag and hang a bag of lactated Ringer's solution

a. Remove the current bag and hang a new bag

A nurse in the emergency department is completing an assessment of a client who has suspected stomach perforation due to a peptic ulcer. Which of the following are expected findings? (Select all that apply) a. Rigid abdomen b. Tachycardia c. Elevated blood pressure d. Circumoral cyanosis e. Rebound tenderness

a. Rigid abdomen b. Tachycardia e. Rebound tenderness

The nurse determines that the goals of dietary teaching have been met when the patient with celiac disease selects from the menu a. Scrambled eggs and sausage. b. Buckwheat pancakes with syrup. c. Oatmeal, skim milk, and orange juice. d. Yogurt, strawberries, and rye toast with butter.

a. Scrambled eggs and sausage.

The client diagnosed with end-stage liver failure is admitted with hepatic encephalopathy. Which dietary restriction should be implemented by the nurse to address this complication? a. Restrict sodium intake to 2 g/day. b. Limit oral fluids to 1500 mL/day. c. Decrease the daily fat intake. d. Reduce protein intake to 60 to 80 g/day.

a. Sodium is restricted to reduce ascites and generalized edema, not for hepatic encephalopathy. b. Fluids are calculated based on diuretic therapy, urine output, and serum electrolyte values; fluids do not affect hepatic encephalopathy. c. A diet high in calories and moderate in fat intake is recommended to promote healing. **d. Ammonia is a byproduct of protein metabolism and contributes to hepatic encephalopathy. Reducing protein intake should decrease ammonia levels.

The nurse is caring for the immediate postoperative client who had a laparoscopic cholecystectomy. Which task could the nurse delegate to the unlicensed nursing assistant? a. Check the abdominal dressings for bleeding. b. Increase the IV fluid if the blood pressure is low. c. Document the patient's input and output d. Listen to the breath sounds in all lobes.

a. This is assessment and cannot be delegated. b. This intervention would require analysis. **c. This intervention would be appropriate for the nursing assistant to implement. d. This would require assessment and cannot be delegated.

The occupational health nurse is preparing a presentation to a group of factory workers about preventing colon cancer. Which information should be included in the presentation? a. Wear a high filtration mask when around chemicals. b. Eat several servings of cruciferous vegetables daily. c. Take a multiple vitamin every day. d. Do not engage in high-risk sexual behaviors.

a. Some cancers have a higher risk of development when the client is exposed to occupationally exposed chemicals, but cancer of the colon is not one of them. **b. Cruciferous vegetables, such as broccoli, cauliflower, and cabbage, are high in fiber. One of the risks for cancer of the colon is a high-fat, low-fiber, and high-protein diet. The longer the transit time (the time from ingestion of the food to the elimination of the waste products) the greater the chance of developing cancer of the colon c. A multiple vitamin may improve immune system function, but it does not prevent colon cancer. d. High-risk sexual behavior places the client at risk for sexually transmitted diseases. A history of multiple sexual partners and initial sexual experience at an early age does increase the risk for the development of cancer of the cervix in females.

Which assessment data would indicate to the nurse that the client's gastric ulcer has perforated? a. Complaints of sudden, sharp, substernal pain. b. Rigid, boardlike abdomen with rebound tenderness. c. Frequent, clay-colored, liquid stool. d. Complaints of vague abdominal pain in the right upper quadrant.

a. Sudden sharp pain felt in the substernal area is indicative of angina or myocardial infarction. **b. A rigid boardlike abdomen with rebound tenderness is the classic sign and symptom of peritonitis, which is a complication of a perforated gastric ulcer. c. Clay-colored stools indicate liver disorders, such as hepatitis. d. Clients with gallbladder disease report vague to sharp abdominal pain in the right upper quadrant.

A nurse is teaching a client who has a duodenal ulcer and a new prescription for esomeprazole (Nexium). Which of the following should be included in the teaching? (Select all that apply) a. Take the medication 1 hr before a meal b. Limit NSAIDs when taking this medication c. Expect skin flushing when taking this medication d. Increase fiber intake when taking this medication e. Chew the medication thoroughly before swallowing

a. Take the medication 1 hr before a meal b. Limit NSAIDs when taking this medication

A patient with a gunshot wound to the abdomen complains of increasing abdominal pain several hours after surgery to repair the bowel. What action should the nurse take first? a. Take the patient's vital signs. b. Notify the health care provider. c. Position the patient with the knees flexed. d. Determine the patient's IV intake since the end of surgery.

a. Take the patient's vital signs. It is likely that the patient could be developing a peritonitis, which could be life-threatening, and assessment of vital signs for hypovolemic shock should be done to report to the health care provider. If an IV line is not in place, it should be inserted and pain may be eased by flexing the knees.

The client diagnosed with inflammatory bowel disease has a serum potassium level of 3.4. Which action should the nurse implement first? a. Notify the HCP b. Assess the client for leg cramps/muscle weakness c. Request telemetry for the client d. Prepare to administer IV potassium

a. The HCP needs to be notified so that potassium supplements can be ordered, but this is not the first intervention. **b. Leg cramps are a sign of hypokalemia; hypokalemia can lead to cardiac dysrhythmias and can be life threatening. Assessment is priority for a potassium level that is just below normal level, which is 3.5 to 5.5 mEq/L. c. Hypokalemia can lead to cardiac dysrhythmias; therefore, requesting telemetry is appropriate, but it is not the first intervention. d. The client will need potassium to correct the hypokalemia, but it is not the first intervention.

The 79-year-old client diagnosed with acute gastroenteritis is admitted to the medical unit. Which nursing task would be most appropriate for the nurse to delegate to the unlicensed nursing assistant? a. Evaluate the client's intake and output. b. Take the client's vital signs. c. Change the client's intravenous solution. d. Assess the client's perianal area.

a. The assistant can calculate the client's intake and output, but the nurse must evaluate the data to determine if it is normal for the elderly client diagnosed with acute gastroenteritis. **b. The assistant can take the vital signs for a client who is stable; the nurse must interpret and evaluate the vital signs. c. The assistant cannot administer medications, and IV solutions are considered to be medications. d. The nurse cannot delegate assessment. The client may have an excoriated perianal area secondary to diarrhea; therefore, the nurse should assess the client.

During the initial postoperative period following bariatric surgery, the nurse recognizes the importance of monitoring obese patients for respiratory insufficiency based on what knowledge? a. The body stores anesthetics in adipose tissue. b. Postoperative pain may cause a decreased respiratory rate. c. Intubation may be difficult because of extra chin skinfolds. d. The patient's head must remain flat for a minimum of 2 hours postprocedure.

a. The body stores anesthetics in adipose tissue.

The client is one (1) day postoperative major abdominal surgery. Which client problem is priority? a. Impaired skin integrity. b. Fluid and electrolyte imbalance. c. Altered bowel elimination. d. Altered body image.

a. The client has a surgical incision, which impairs the skin integrity, but it is not the priority because it is sutured under sterile conditions. **b. After abdominal surgery, the body distributes fluids to the affected area as part of the healing process. These fluids are shifted from the intravascular compartment to the interstitial space, which causes potential fluid and electrolyte imbalance. c. Bowel elimination is a problem, but after general anesthesia wears off, the bowel sounds will return and this is not a life-threatening problem. d. Psychosocial problems are not priority over actual physiological problems

The nurse is preparing a client diagnosed with GERD for discharge following an EGD. Which statement indicated the client understands the discharge instructions? a. "I should not eat for at least 1 day following this procedure" b. "I can lie down whenever I want after a meal. It won't make a difference" c. "The stomach contents won't bother my esophagus but will make me nauseous" d. "I should avoid orange juice and eating tomatoes until my esophagus heals"

a. The client is allowed to eat as soon as the gag reflex has returned. b. An esophagogastroduodenoscopy is a diagnostic procedure, not a cure. Therefore the client still has GERD and should be instructed to stay in an upright position for two (2) to three (3) hours after eating. c. Stomach contents are acidic and will erode the esophageal lining. **d. Orange and tomato juices are acidic, and the client diagnosed with GERD should avoid acidic foods until the esophagus has had a chance to heal.

The nurse is caring for a client diagnosed with GERD. Which nursing interventions should be implemented? a. Place the client prone in bed and administer nonsteroidal anti-inflammatory medications b. Have the client remain upright at all times and walk for 30 minutes 3 times a week c. Instruct the client to maintain a right lateral side-lying position and take antacids before meals d. Elevate the head of the bed 30 degrees and discuss lifestyle modifications with the client

a. The client is encouraged to lie with the head of the bed elevated, but this is difficult to achieve when on the stomach. NSAIDs inhibit prostaglandin synthesis in the stomach, which places the client at risk for developing gastric ulcers. The client is already experiencing gastric acid difficulty. b. The client will need to lie down at some time, and walking will not help with GERD. c. If lying on the side, the left side-lying position, not the right side, will allow less chance of reflux into the esophagus. Antacids are taken one (1) and three (3) hours after a meal. **d. The head of the bed should be elevated to allow gravity to help in preventing reflux. Lifestyle modifications of losing weight, making dietary modifications, attempting smoking cessation, discontinuing the use of alcohol, and not stooping or bending at the waist all help to decrease reflux.

The nurse is planning the care of a client diagnosed with lower esophageal sphincter dysfunction. Which dietary modifications should be included in the plan of care? a. Allow any of the clients favorite foods as long as the amount is limited b. Have the client perform eructation exercises several times a day c. Eat 4 to 6 small meals a day and limit fluids during meal times d. Encourage the client to consume a glass of red wine with one meal a day

a. The client is instructed to avoid spicy and acidic foods and any food that produces symptoms. b. Eructation means belching, which is a symptom of GERD. **c. Clients should eat small, frequent meals and limit fluids with the meals to prevent reflux into the esophagus from a distended stomach. d. Clients are encouraged to forgo all alcoholic beverages because alcohol relaxes the lower esophageal sphincter and increases the risk of reflux.

The client is placed on percutaneous gastrostomy (PEG) tube feedings. Which occurrence would warrant immediate intervention by the nurse? a. The client tolerates the feedings being infused at 50 mL/hour. b. The client pulls the nasogastric feeding tube out. c. The client complains of being thirsty. d. The client has green, watery stool.

a. The client is tolerating the feeding change so there is no need for an immediate action. b. The client has a PEG tube; therefore, the nasogastric feeding tube can be removed. c. This should be addressed; the client may require some ice chips in the mouth or some oral care, but this must wait until an assessment of the client's ability to swallow has been completed. **d. This client needs to be cleaned immediately; the abdomen must be assessed; and a determination must be made regarding the type of feeding and the additives and medications being administered and skin damage occurring. This client is priority.

Which instruction should the nurse discuss with the client who is in the icteric phase of hepatitis C? a. Decrease alcohol intake. b. Encourage rest periods. c. Eat a large evening meal. d. Drink diet drinks and juices.

a. The client must avoid alcohol altogether, not decrease intake, to prevent further liver damage and promote healing. **b. Adequate rest is needed for maintaining optimal immune function. c. Clients are more often anorexic and nauseated in the afternoon and evening; therefore the main meal should be in the morning. d. Diet drinks and juices provide few calories, and the client needs an increased caloric diet for healing.

The client is diagnosed with peritonitis. Which assessment data indicate the client's condition is improving? a. The client is using more pain medication on a daily basis. b. The client's nasogastric tube is draining coffee-ground material. c. The client has a decrease in temperature and a soft abdomen. d. The client has had two (2) soft, formed bowel movements.

a. The client needing more pain medication indicates the client's condition is getting worse. b. Coffee-ground material indicates old blood. **c. Because the signs of peritonitis are elevated temperature and rigid abdomen, a reversal of these signs would indicate the client is getting better. d. Two soft-formed bowel movements are good, but this does not have anything to do with peritonitis.

Which statement by the client diagnosed with hepatitis would warrant immediate intervention by the clinic nurse? a. "I will not drink any type of beer or mixed drink." b. "I will get adequate rest so that I don't get exhausted." c. "I had a big hearty breakfast this morning." d. "I took some cough syrup for this nasty head cold."

a. The client should avoid alcohol to prevent further liver damage and promote healing b. Rest is needed for healing of the liver and to promote optimum immune function. c. Clients with hepatitis need increased caloric intake, so this is a good statement. **d. The client needs to understand that some types of cough syrup have alcohol and all alcohol must be avoided to prevent further injury to the liver; therefore this statement requires intervention.

The nurse is teaching the client diagnosed with diverticulosis. Which instruction should the nurse include in the teaching session? a. Discuss the importance of drinking 1000 mL of water daily. b. Instruct the client to exercise at least three (3) times a week. c. Teach the client about a eating a low-residue diet. d. Explain the need to have daily bowel movements.

a. The client should drink at least 3000 mL of water daily to help prevent constipation. b. The client should exercise daily to help prevent constipation. c. The client should eat a high-fiber diet to help prevent constipation. **d. The client should have regular bowel movements, preferably daily. Constipation may cause diverticulitis, which is a potentially life-threatening complication of diverticulosis.

The client has had a liver biopsy. Which post-procedure intervention should the nurse implement? a. Instruct the client to void immediately. b. Keep the client NPO for eight (8) hours. c. Place the client on the right side. d. Monitor blood urea nitrogen (BUN) and creatinine level.

a. The client should empty the bladder immediately prior to the liver biopsy, not after the procedure. b. Foods and fluids are usually withheld two (2) hours after the biopsy, after which the client can resume the usual diet. **c. Direct pressure is applied to the site, and then the client is placed on the right side to maintain site pressure. d. BUN and creatinine levels are monitored for kidney function, not liver function, and the renal system is not affected with the liver biopsy.

A 61-year-old patient with suspected bowel obstruction had a nasogastric tube inserted at 4:00 AM. The nurse shares in the morning report that the day shift staff should check the tube for patency at what times? a. 7:00 AM, 10:00 AM, and 1:00 PM b. 8:00 AM, 12:00 PM, and 4:00 PM c. 9:00 AM and 3:00 PM d. 9:00 AM, 12:00 PM, and 3:00 PM

b. 8:00 AM, 12:00 PM, and 4:00 PM

The client is admitted to the medical unit with a diagnosis of acute diverticulitis. Which health-care provider's order should the nurse question? a. Insert a nasogastric tube. b. Start IV D5W at 125 mL/hr. c. Put client on a clear liquid diet. d. Place client on bed rest with bathroom privileges.

a. The client will have a nasogastric tube because the client will be NPO, which will decompress the bowel and help remove hydrochloric acid. b. Preventing dehydration is a priority with the client who is NPO. **c. The nurse should question a clear liquid diet because the bowel must be put on total rest, which means NPO. d. The client is in severe pain and should be on bed rest, which will help rest the bowel.

The client presents with a complete blockage of the large intestine from a large tumor. Which health-care provider's order would the nurse question? a. Obtain consent for a colonoscopy and biopsy. b. Start an IV of 0.9% saline at 125 mL/hour. c. Administer 3 liters of Go-LYTELY. d. Give tap water enemas until it is clear.

a. The client will need to have diagnostic tests so this is an appropriate intervention. b. The client who has an intestinal blockage will need to be hydrated. **c. This client has an intestinal blockage from a solid tumor blocking the colon. Although the client needs to be cleaned out for the colonoscopy, this would cause severe cramping without a reasonable benefit to the client and could cause a medical emergency. d. Tap water enemas until it is clear would be instilling water from below the tumor to try and rid the colon of any feces in that portion of the colon and the client can expel this water.

The client has developed a paralytic ileus after abdominal surgery. Which intervention should the nurse include in the plan of care? a. Administer a laxative of choice. b. Encourage client to increase oral fluids. c. Encourage the client to take deep breaths. d. Maintain a patent nasogastric tube.

a. The client would be NPO; therefore no medication would be administered. b. The client would be NPO; therefore no food or fluids would be administered. c. Deep breathing will help prevent pulmonary complications, but it will not address the client's paralytic ileus. **d. A paralytic ileus is the absence of peristalsis; therefore the bowel will be unable to process any oral intake. A nasogastric tube is inserted to decompress the bowel until there is surgical intervention or bowel sounds return spontaneously.

The female client came to the clinic complaining of abdominal cramping and has had at least 10 episodes of diarrhea every day for the last 2 days. The client reported that she had been in Mexico on a mission trip and just returned yesterday. Which intervention should the nurse implement? a. Instruct the client to take a cathartic laxative daily. b. Encourage the client to drink lots of Gatorade. c. Discuss the need to increase protein in the diet. d. Explain that the client should weigh herself daily.

a. The client would be taking antidiarrheal medication, not medications to stimulate bowel movements. **b. The client probably has traveler's diarrhea, and oral rehydration is the preferred choice for replacing fluids lost as a result of diarrhea. An oral glucose electrolyte solution, such as Gatorade, All-Sport, or Pedialyte, is recommended. c. The client should be encouraged to stay on liquids and eat bland foods of all three (3) food groups—carbohydrates, proteins, and fats. d. There is no need for the client to weigh herself daily. Symptoms usually resolve within two (2) to three (3) days without complications.

Which assessment data would the nurse expect to find in the client diagnosed with acute gastroenteritis? a. Decreased gurgling sounds on auscultation of the abdominal wall. b. A hard, firm edematous abdomen on palpation. c. Frequent, small melena-type liquid bowel movements. d. Bowel assessment reveals loud, rushing bowel sounds.

a. The client would have increased gurgling sounds revealing hyperactive bowel movements. b. A hard, firm, edematous abdomen would not be expected in a client with gastroenteritis; this would indicate a possible complication and require further assessment. c. The client would have increased liquid bowel movements, diarrhea, but would not have blood in the stool, which is the definition of melena. **4. Borborygmi, or loud, rushing bowel sounds, indicates increased peristalsis, which occurs in clients with diarrhea and is the primary clinical manifestation in a client diagnosed with acute gastroenteritis

The client is two (2) hours post-colonoscopy. Which assessment data would warrant intermediate intervention by the nurse? a. The client has a soft, nontender abdomen. b. The client has a loose, watery stool. c. The client has hyperactive bowel sounds. d. The client's pulse is 104 and BP is 98/60.

a. The client's abdomen should be soft and nontender; therefore, this finding would not require immediate intervention. b. The client had to clean the bowel prior to the colonoscopy; therefore, watery stool would be expected. c. The client was NPO and received bowel preparation prior to the colonoscopy; therefore, hyperactive bowel sounds might occur and would not warrant immediate intervention. **d. Bowel perforation is a potential complication of a colonoscopy. Therefore signs of hypotension—decreased BP and increased pulse—would warrant immediate intervention from the nurse.

The client is diagnosed with an acute exacerbation of ulcerative colitis. Which intervention should the nurse implement? a. Provide a low-residue diet b. Rest the client's bowel c. Assess vital signs daily d. Administer antacids orally

a. The client's bowel should be placed on rest and no foods or fluids should be introduced into the bowel. **b. Whenever a client has an acute exacerbation of a GI disorder the first intervention is to place the bowel on rest. The client should be NPO with IV fluids to prevent dehydration c. The vital signs must be taken more often than daily in a client who is having an acute exacerbation of ulcerative colitis. d. The client will receive anti-inflammatory and antidiarrheal medications, not antacids, which are used for gastroenteritis.

The client diagnosed with end-stage liver failure is admitted to the medical unit diagnosed with esophageal bleeding. The HCP inserts and inflates a triple-lumen nasogastric tube (Sengstaken-Blakemore). Which nursing action should the nurse implement for this treatment? a. Assess the gag reflex every shift. b. Stay with the client at all times. c. Administer the laxative lactulose (Chronulac). d. Monitor the client's ammonia level.

a. The client's throat is not anesthetized during the insertion of a nasogastric tube, so the gag reflex does not need to be assessed. **b. While the balloons are inflated, the client must not be left unattended in case they become dislodged and occlude the airway. This is a safety issue. c. This laxative is administered to decrease the ammonia level, but the question does not say that the client's ammonia level is elevated d. Esophageal bleeding does not cause the ammonia level to be elevated.

The clinic nurse is talking on the phone to a client who has diarrhea. Which intervention should the nurse discuss with the client? a. Tell the client to measure the amount of stool b. Recommend the client come to the clinic immediately c. Explain the client should follow the BRAT diet d. Discuss taking over the counter histamine-2 blocker

a. The clinic nurse should not ask the client to measure stool at home; this is done in the acute care setting b. Unless the client has had diarrhea for longer than 48 hours, the client does not need to be seen in the clinic **c. The BRAT (bananas, rice, applesauce, and toast) diet is recommended for a client with diarrhea because it is low residue and produces nutrition while not irritating the GI system d. Histamine-2 blockers decreased gastric acid production and would not be prescribed for a client with diarrhea

Which expected outcome would be appropriate for the client scheduled to have a cholecystectomy? a. Decreased pain management. b. Ambulate first day postoperative. c. No break in skin integrity. d. Knowledge of postoperative care.

a. The expected outcome would be increased pain management for both preoperative and postoperative care. b. Postoperative care would include ambulation. c. Prevention of an additional break in skin integrity would be a desired postoperative outcome. The incision would be a break in skin integrity. **d. This would be an expected outcome for the client scheduled for surgery. This indicates that preoperative teaching has been effective.

Which task would be most appropriate for the nurse to delegate to the unlicensed nursing assistant? a. Draw the serum liver function test. b. Evaluate the client's intake and output. c. Assist the client to the bedside commode. d. Help the ward clerk transcribe orders.

a. The laboratory technician draws serum blood studies, not the nursing assistant. b. The nursing assistant can obtain the intake and output, but the nurse must evaluate the data to determine if the results are normal for the client's disease process or condition. **c. The nursing assistant can assist a client to the bedside commode. d. The ward clerk has specific training that allows the transcribing of health-care provider orders.

The 84-year-old client comes to the clinic complaining of right lower abdominal pain. Which question would be most appropriate for the nurse to ask the client? a. "When was your last bowel movement?" b. "Did you have a high-fat meal last night?" c. "How long have you had this pain?" d. "Have you been experiencing any gas?"

a. The last bowel movement would not help identify the cause of the client's right lower abdominal pain. This might be appropriate for a client with left lower abdominal pain. b. Information about a high-fat meal would be asked if the nurse suspected the client had a gallbladder problem. **c. Elderly clients usually display a high tolerance to pain and frequently may have a ruptured appendix with minimal pain, therefore the nurse should assess the characteristic and etiology of the pain. c. The passage of flatus (gas) does not help determine the cause of right lower abdominal pain.

The client is diagnosed with gastroenteritis. Which laboratory data would warrant immediate intervention by the nurse? a. A serum sodium level of 137 mEq/L. b. An arterial blood gas of pH 7.37, PaO2 95, PaCO2 43, HCO3 24. c. A serum potassium level of 3.3 mEq/L. d. A stool sample that is positive for fecal leukocytes.

a. The normal serum sodium level is 135-145 mEq/L; therefore an intervention by the nurse is not needed. b. These are normal arterial blood gas results; therefore, the nurse would not need to intervene. **c. In gastroenteritis, diarrhea often results in metabolic acidosis and loss of potassium. The normal serum potassium level is 3.5-5.5 mEq/L; therefore a 3.3 mEq/L would require immediate intervention. Hypokalemia (a low potassium level) can lead to life-threatening cardiac dysrhythmias. d. A stool specimen showing fecal leukocytes would support the diagnosis of gastroenteritis and not warrant immediate intervention by the nurse.

The client has had abdominal surgery and tells the nurse, "I felt as something just gave way in my stomach." Which action should the nurse implement first? a. Notify the surgeon immediately. b. Instruct the client to splint the incision. c. Assess the abdominal wound incision d. Administer pain medication intravenously.

a. The nurse may notify the surgeon if warranted, but that is not the first intervention. b. The nurse should instruct the client to splint the incision when coughing but then take further action. **c. Assessing the surgical incision is the first intervention because this may indicate the client has wound dehiscence. d. The nurse should never administer pain medication without assessing for potential complications.

The female nurse sticks herself with a dirty needle. Which action should the nurse implement first? a. Notify the infection control nurse. b. Cleanse the area with soap and water. c. Request post-exposure prophylaxis. d. Check the hepatitis status of the client.

a. The nurse must notify the infection control nurse as soon as possible so that treatment can start if needed, but this is not the first intervention. **b. The nurse should first clean the needle stick with soap and water to help remove any virus that is on the skin. c. Post-exposure prophylaxis may be needed, but this is not the first action. d. The infection control nurse will check the status of the client that the needle was used on before the nurse stuck herself.

Which nursing interventions should be included in the care plan for the 84-year-old client diagnosed with acute gastroenteritis? Select all that apply. a. Assess the skin turgor on the back of the client's hands. b. Monitor the client for orthostatic hypotension. c. Record the frequency and characteristics of sputum. d. Use standard precautions when caring for the client. e. Institute safety precautions when ambulating the client.

a. The nurse should assess the elderly skin turgor over the sternum because loss of subcutaneous fat associated with aging makes skin turgor assessment on the arms less reliable. **b. Orthostatic hypotension indicates fluid volume deficit, which can occur in an elderly client who is having many episodes of diarrhea, which occurs with acute gastroenteritis. c. The nurse should record frequency and characteristics of stool, not sputum, in the client diagnosed with gastroenteritis. **d. Standard precautions, including wearing gloves and hand washing, help prevent the spread of the infection to others. **e. The elderly client is at risk for orthostatic hypotension; therefore safety precautions should be instituted to ensure the client doesn't fall as a result of a decrease in blood pressure.

The nurse is caring for a patient treated with IV fluid therapy for severe vomiting. As the patient recovers and begins to tolerate oral intake, which food choice does the nurse understand would be most appropriate? a. Iced tea b. Dry toast c. Hot coffee d. Plain hamburger

b. Dry toast

The client complains to the nurse of unhappiness with the health-care provider. Which intervention should the nurse implement next? a. Call the HCP and suggest he or she talk with the client. b. Determine what about the HCP is bothering the client. c. Notify the nursing supervisor to arrange a new HCP to take over. d. Explain to the client that until discharge, the client will have to keep the HCP

a. The nurse should first inform the HCP of the client's concerns and then allow the HCP and client to discuss the situation. **b. The nurse should determine what is concerning the client. It could be a misunderstanding or a real situation where the client's care is unsafe or inadequate. c. If a new HCP is to be arranged, it is the HCP's responsibility to arrange for another HCP to take over the care of the client. d. The choice of HCP is ultimately the client's. If the HCP cannot arrange for another HCP, the client may have to be discharged and obtain a new health-care provider.

The client diagnosed with Crohn's disease is crying and tells the nurse "I can't take it anymore. I never know when I will get sick and end up here in the hospital." Which statement is the nurse's best response? a. "I understand how frustrating this must be for you" b. "You must keep thinking about the good things in your life" c. "I can see you are very upset. Ill sit down and we can talk" d. "Are you thinking about doing anything like committing suicide?"

a. The nurse should never tell a client that they understand what they are going through. b. This is not addressing the client's feelings. **c. The client is crying and is expressing feelings of powerlessness; therefore the nurse should allow the client to talk. d. The client is crying and states "I can't take it anymore," but this is not a suicidal comment or situation.

The nurse, a licensed practical nurse, and an unlicensed nursing assistant are caring for clients on a medical floor. Which nursing task would be most appropriate to assign to the licensed practical nurse? a. Assist the unlicensed nursing assistant to learn to perform blood glucose checks. b. Monitor the potassium levels of a client with diarrhea. c. Administer a bulk laxative to a client diagnosed with constipation. d. Assess the abdomen of a client who has had complaints of pain.

a. The nurse will be responsible for signing off on the unlicensed nursing assistant as to being competent to perform the blood glucose. The nurse should do this to determine the competency of the assistant. b. The lab values may require the nurse to interpret and act on the results. The nurse cannot delegate anything that requires professional judgment. **c. The licensed practical nurse could administer a laxative. d. The nurse cannot delegate assessment.

The wound, ostomy, and continence (WOC) nurse selects the site where the ostomy will be placed. What should be included in the consideration for the site? a. The patient must be able to see the site. b. Outside the rectus muscle area is the best site. c. It is easier to seal the drainage bag to a protruding area. d. The ostomy will need irrigation, so area should not be tender.

a. The patient must be able to see the site.

Which assessment data supports the client's diagnosis of gastric ulcer? a. Presence of blood in the clients stool for the past month b. Reports of a burning sensation moving like a wave c. Sharp pain in the upper abdomen after eating a heavy meal d. Complaints of epigastric pain 30-60 minutes after ingesting food.

a. The presence of blood does not specifically indicate diagnosis of an ulcer. The client could have hemorrhoids or cancer that would result in the presence of blood. b. A wavelike burning sensation is a symptom of gastroesophageal reflux. c. Sharp pain in the upper abdomen after eating a heavy meal is a symptom of gallbladder disease. **d. In a client diagnosed with a gastric ulcer, pain usually occurs 30-60 minutes after eating, but not at night. In contrast, a client with a duodenal ulcer has pain during the night that is often relieved by eating food. Pain occurs 1-3 hours after meals.

The client diagnosed with end-stage renal failure with ascites is scheduled for a paracentesis. Which client teaching should the nurse discuss with the client? a. Explain that the procedure will be done in the operating room. b. Instruct the client that a Foley catheter will have to be inserted. c. Tell the client that vital signs will be taken frequently after the procedure. d. Provide instructions on holding the breath when the HCP inserts the catheter

a. The procedure is done in the client's room, with the client either seated on the side of the bed or in a chair. b. The client should empty the bladder prior to the procedure to avoid bladder puncture, but there is no need for a Foley catheter to be inserted. **c. The client is at risk for hypovolemia; therefore, vital signs will be assessed frequently to monitor for signs of hemorrhaging. d. The client does not have to hold the breath when the catheter is inserted into the peritoneum; this is done when obtaining a liver biopsy.

The nurse has been assigned to care for a client diagnosed with peptic ulcer disease. When the nurse is evaluating care, which assessment data require further intervention? a. Bowel sounds auscultated fifteen (15) times in one (1) minute. b. Belching after eating a heavy and fatty meal late at night. c. A decrease in systolic BP of 20 mm Hg from lying to sitting. d. A decreased frequency of distress located in the epigastric region.

a. The range for normoactive bowel sounds is from five (5) to thirty-five (35) times per minute. This would require no intervention. b. Belching after a heavy, fatty meal is a symptom of gallbladder disease. Eating late at night may cause symptoms of esophageal disorders. **c. A decrease of 20 mm Hg in blood pressure after changing position from lying, to sitting, to standing is orthostatic hypotension. This could indicate that the client is bleeding. d. A decrease in the quality and quantity of discomfort shows an improvement in the client's condition. This would not require further intervention.

The client who has had an abdominal perineal resection is being discharged. Which discharge information should the nurse teach? a. The stoma should be a white, blue, or purple color. b. Limit ambulation to prevent the pouch from coming off. c. Take pain medication when the pain level is at an "8." d. Empty the pouch when it is one-third to one-half full.

a. The stoma should be light to a medium pink, the color of the intestines. A blue or purple color would indicate a lack of circulation to the stoma and is a medical emergency. b. The stoma should be pouched securely for the client to be able to participate in normal daily activities. The client should be encouraged to ambulate to aid in recovery. c. Pain medication should be taken before the pain level reaches a five (5). Delaying taking medication will delay the onset of pain relief and the client will not get a full benefit from the medication. **d. The pouch should be emptied when it is one-third to one-half full to prevent the contents from becoming too heavy for the seal to hold and to prevent leakage from occurring.

The patient with cirrhosis has an increased abdominal girth from ascites. The nurse should know that this fluid gathers in the abdomen for which reasons? Select all that apply. a. There is decreased colloid oncotic pressure from the liver's inability to synthesize albumin. b. Hyperaldosteronism related to damaged hepatocytes increases sodium and fluid retention. c. Portal hypertension pushes proteins from the blood vessels, causing leaking into the peritoneal cavity. d. Osmoreceptors in the hypothalamus stimulate thirst, which causes the stimulation to take in fluids orally. e. Overactivity of the enlarged spleen results in increased removal of blood cells from the circulation, which decreases the vascular pressure.

a. There is decreased colloid oncotic pressure from the liver's inability to synthesize albumin. b. Hyperaldosteronism related to damaged hepatocytes increases sodium and fluid retention. c. Portal hypertension pushes proteins from the blood vessels, causing leaking into the peritoneal cavity.

Which problems should the nurse include in the plan of care for the client diagnosed with peptic ulcer disease to observe for physiological complications? a. Alteration in bowel elimination patterns b. Knowledge deficit in the causes of ulcers c. Inability to cope with changing family roles d. Potential for alteration in gastric emptying

a. There is no indication from the question that there is a problem or potential problem with bowel elimination. b. Knowledge deficit does not address physiological complications. c. This client may have problems from changing roles within the family, but the question asks for potential physiological complications, not psychosocial problems. **d. Potential for alteration in gastric emptying is caused by edema or scarring associated with peptic ulcer disease, which may cause a feeling of "fullness," vomiting of undigested food, or abdominal distention.

The client admitted to the medical unit with diverticulitis is complaining of severe pain in the left lower quadrant and has an oral temperature of 100.6F. Which action should the nurse implement first? a. Notify the health-care provider. b. Document the findings in the chart. c. Administer an oral antipyretic. d. Assess the client's abdomen.

a. These are classic signs/symptoms of diverticulitis; therefore the HCP would not need to be notified. b. These are normal findings for a client diagnosed with diverticulitis, but on admission the nurse should assess the client and document the findings in the client's chart. c. The nurse should not administer any food or medications. **d. The nurse should assess the client to determine if the abdomen is soft and nontender. A rigid tender abdomen may indicate peritonitis.

The charge nurse has completed report. Which client should be seen first? a. The client diagnosed with Crohn's disease who had two (2) semi-formed stools on the previous shift. b. The elderly client admitted from another facility who is complaining of constipation. c. The client diagnosed with AIDS who had a 200-mL diarrhea stool and has elastic skin tissue turgor. d. The client diagnosed with hemorrhoids who had some spotting of bright red blood on the toilet tissue.

a. This client is improving; semi-formed stools are better than diarrhea. **b. This client has just arrived so the nurse does not know if the complaint is valid and needs intervention unless this client is seen and assessed. The elderly have difficulty with constipation as a result of decreased gastric motility, medications, poor diet, and immobility. c. The client has diarrhea, but only 200 mL, and has elastic tissue turgor that lets the nurse know the client is not dehydrated. d. This is not normal, but it is expected for a client with hemorrhoids.

The client is one (1) hour post-endoscopic retrograde cholangiopancreatogram (ERCP). Which intervention should the nurse include in the plan of care? a. Instruct the client to cough forcefully. b. Encourage early ambulation. c. Assess for return of a gag reflex. d. Administer held medications.

a. This intervention may irritate the client's throat. b. This would not enhance safety. **c. The endoscopic retrograde cholangiopancreatogram (ERCP) requires that an anesthetic spray be used prior to insertion of the endoscope. If medications, food, or fluid is given orally prior to the return of the gag reflex, the client may aspirate, causing pneumonia that could be fatal. d. Medications would not be given until the gag reflex has returned.

The nurse assesses a large amount of red drainage on the dressing of a client who is 6 hours postoperative open cholecystectomy. Which intervention should the nurse implement? a. Measure the abdominal girth. b. Palpate the lower abdomen for a mass. c. Turn client onto side to assess for further drainage. d. Remove the dressing to determine the source.

a. This intervention would help further assess internal bleeding, not external bleeding. b. This would assess the bladder, not bleeding. **c. Turning the client to the side to assess the amount of drainage and possible bleeding is important prior to contacting the surgeon. d. The first dressing change is usually done by the surgeon; the nurse would reinforce the dressing.

The client with hepatitis asks the nurse, "I went to an herbalist, who recommended I take milk thistle. What do you think about that?" Which statement is the nurse's best response? a. "You are concerned about taking an herb." b. "The herb has been used to treat liver disease." c. "I would not take anything that is not prescribed." d. "Why would you want to take any herbs?"

a. This is a therapeutic response and the nurse should provide factual information. **b. Milk thistle has an active ingredient, silymarin, which has been used to treat liver disease for more than 2000 years. It is a powerful oxidant and promotes liver cell growth. c. The nurse should not discourage complemen- tary therapies. d. This is a judgmental statement and the nurse should encourage the client to ask questions.

The client is diagnosed with end-stage liver failure. The client asks the nurse, "Why is my doctor decreasing the doses of my medications?" Which statement is the nurse's best response? a. "You are worried that your doctor has decreased the dosage." b. "You really should ask your doctor. I am sure there is a good reason." c. "You may have an overdose of the medication because your liver is damaged." d. "The half-life is altered because the liver is damaged."

a. This is a therapeutic response and used to encourage the client to verbalize feelings, but it does not provide factual information. b. This is passing the buck; the nurse should be able to answer this question. **c. This is the main reason the HCP decreases the client's medication dose, and it is an explanation appropriate for the client. d. This is the medical explanation as to why the medication dose is decreased, but it should not be used to explain to a layperson.

Which nursing diagnosis would be highest priority for the client who had an open cholecystectomy surgery? a. Alteration in nutrition. b. Alteration in skin integrity. c. Alteration in urinary pattern. d. Alteration in comfort.

a. This may be an appropriate client problem, but it is not priority. b. This may be an appropriate client problem, but is not priority. c. This may be an appropriate client problem, but is not priority. **d. Acute pain management is the highest priority client problem after surgery because pain may indicate a life-threatening problem.

The client with type 2 diabetes is prescribed prednisone, a steroid, for an acute exacerbation of inflammatory bowel disease. Which intervention should the nurse discuss with the client? a. Take the medication on an empty stomach b. Notify the HCP if experiencing a moon face c. Take the steroid medication as prescribed d. Notify the HCP if the blood glucose is over 160

a. This medication can cause erosion of the stomach and should be taken with food. b. A moon face is an expected side effect of prednisone. **c. This medication must be tapered off to prevent adrenal insufficiency; therefore, the client must take this medication as prescribed. d. There is no need to take this medication only in the morning. It is usually taken three (3) to four (4) times a day.

The nurse is teaching a client recovering from a laparoscopic cholecystectomy. Which statement indicates the discharge teaching was effective? a. "I will take my lipid-lowering medicine at the same time each night." b. "I may experience some discomfort when I eat a high-fat meal." c. "I need someone to stay with me for about a week after surgery." d. "I should not splint my incision when I deep breathe and cough."

a. This surgery does not require lipid-lowering medications, but eating high-fat meals may cause discomfort. **b. After removal of the gallbladder, some clients experience abdominal discomfort when eating fatty foods. c. Laparoscopic cholecystectomy surgeries are performed in day surgery, and clients usually do not need assistance for a week. d. Using a pillow to splint the abdomen provides support for the incision and should be continued after discharge.

The client is admitted to the medical floor with acute diverticulitis. Which collaborative intervention would the nurse anticipate the health-care provider ordering? a. Administer total parenteral nutrition. b. Maintain NPO and nasogastric tube. c. Maintain on a high-fiber diet and increase fluids. d. Obtain consent for abdominal surgery.

a. Total parenteral nutrition is not an expected order for this client. **b. The bowel must be put at rest. Therefore, the nurse should anticipate orders for maintaining NPO and a nasogastric tube. c. These orders would be instituted when the client is getting better and the bowel is not inflamed. d. Surgery is not the first consideration when the client is admitted into the hospital.

The nurse identifies the client problem as "excess fluid volume" for the client in liver failure. Which short-term goal would be most appropriate for this problem? a. The client will not gain more that two (2) kg a day. b. The client will have no increase in abdominal girth. c. The client's vital signs will remain within normal limits (WNL). d. The client will receive a low-sodium diet.

a. Two (2) kg is more than four (4) pounds, which indicates severe fluid retention and is not an appropriate goal. **b. Excess fluid volume could be secondary to portal hypertension. Therefore, no increase in abdominal girth would be an appropriate short-term goal, indicating no excess of fluid volume. c. Vital signs are appropriate to monitor, but they do not yield specific information about fluid volume status. d. Having the client receive a low-sodium diet does not ensure that the client will comply with the diet. The short-term goal must evaluate if the fluid volume is within normal limits

The client is admitted with end-stage liver failure and is prescribed the laxative lactulose (Chronulac). Which statement indicates the client needs more teaching concerning this medication? a. "I should have two to three soft stools a day." b. "I must check my ammonia level daily." c. "If I have diarrhea, I will call my doctor." d. "I should check my stool for any blood."

a. Two to soft three stools a day indicates the medication is effective. **b. There is no instrument that can be used at home to test daily ammonia levels. The ammonia level is a serum level that requires venipuncture and laboratory diagnostic equipment. c. Diarrhea indicates an overdosage, possibly requiring that the dosage be decreased. The HCP would need to make this change in dosage, so the client is correct. d. The client should check the stool for bright red blood as well as dark, tarry stool.

When planning the care for a client diagnosed with peptic ulcer disease, which expected outcome should the nurse include? a. The client's pain is controlled with the use of NSAIDs. b. The client maintains lifestyle modifications. c. The client has no signs and symptoms of hemoptysis. d. The client takes antacids with each meal.

a. Use of NSAIDs increases and causes problems associated with peptic ulcer disease. **b. Maintaining lifestyle changes such as following an appropriate diet and reducing stress indicates that the client is complying with the medical teachings. Such compliance is the goal of treatment to prevent complications. c. Hemoptysis is coughing up blood, which is not a sign or symptom of peptic ulcer disease, so not coughing up blood would not be an expected outcome for a client with peptic ulcer disease. d. Antacids should be taken one (1) to three (3) hours after meals, not with each meal.

When caring for a patient with liver disease, the nurse recognizes the need to prevent bleeding resulting from altered clotting factors and rupture of varices. Which nursing interventions would be appropriate to achieve this outcome? Select all that apply. a. Use smallest gauge needle possible when giving injections or drawing blood. b. Teach patient to avoid straining at stool, vigorous blowing of nose, and coughing. c. Advise patient to use soft-bristle toothbrush and avoid ingestion of irritating food. d. Apply gentle pressure for the shortest possible time period after performing venipuncture. e. Instruct patient to avoid aspirin and NSAIDs to prevent hemorrhage when varices are present.

a. Use smallest gauge needle possible when giving injections or drawing blood. b. Teach patient to avoid straining at stool, vigorous blowing of nose, and coughing c. Advise patient to use soft-bristle toothbrush and avoid ingestion of irritating food. e. Instruct patient to avoid aspirin and NSAIDs to prevent hemorrhage when varices are present.

Combined with clinical manifestations, what is the laboratory finding that is most commonly used to diagnose acute pancreatitis? a. Increased serum calcium b. Increased serum amylase c. Increased urinary amylase d. Decreased serum glucose

b. Increased serum amylase Although serum lipase levels and urinary amylase levels are increased, an increased serum amylase level is the criterion most commonly used to diagnose acute pancreatitis in the first 24 to 72 hours. Serum calcium levels are decreased.

The patient calls the clinic and describes a bump at the site of a previous incision that disappears when he lies down. The nurse suspects that this is which type of hernia (select all that apply)? a. Ventral b. Inguinal c. Femoral d. Reducible e. Incarcerated f. Strangulated

a. Ventral d. Reducible The ventral or incisional hernia is due to a weakness of the abdominal wall at the site of a previous incision. It is reducible when it returns to the abdominal cavity. Inguinal hernias are at the weak area of the abdominal wall where the spermatic cord in men or the round ligament in women emerges. A femoral hernia is a protrusion through the femoral ring into the femoral canal. Incarcerated hernias do not reduce.

When caring for a patient with a biliary obstruction, the nurse will anticipate administering which vitamin supplements? Select all that apply a. Vitamin A b. Vitamin D c. Vitamin E d. Vitamin K e. Vitamin B

a. Vitamin A b. Vitamin D c. Vitamin E d. Vitamin K

A patient treated for vomiting is to begin oral intake when the symptoms have subsided. To promote rehydration, the nurse plans to administer which fluid first? a. Water b. Hot tea c. Gatorade d. Warm broth

a. Water Water is the fluid of choice for rehydration by mouth. Very hot or cold liquids are not usually well tolerated and although broth and Gatorade have been used for the patient with severe vomiting, these substances are high in sodium and should be administered with caution.

The client is diagnosed with an acute exacerbation of IBD. Which priority intervention should the nurse implement first? a. Weight the client daily and document in the client's chart b. Teach coping strategies such as dietary modifications c. Record the frequency, amount, and color of stools d. Monitor the clients oral fluid intake every shift

a. Weighing the client daily will help identify if the client is experiencing malnutrition, but it is not the priority intervention during an acute exacerbation. b. Coping strategies help develop healthy ways to deal with this chronic disease that has remissions and exacerbations, but it is not the priority intervention. **c. The severity of the diarrhea helps determine the need for fluid replacement. The liquid stool should be measured as part of the total output. d. The client will be NPO when there is an acute exacerbation of IBD to allow the bowel to rest.

A patient was diagnosed with nonalcoholic fatty liver disease (NAFLD). What treatment measures should the nurse plan to teach the patient about (select all that apply)? a. Weight loss b. Diabetes management c. Ulcerative colitis dietary changes d. Dietary management of hyperlipidemia e. Maintaining blood pressure with increased sodium and fluid intake

a. Weight loss b. Diabetes management d. Dietary management of hyperlipidemia There is no treatment for nonalcoholic fatty liver disease (NAFLD) except to control the other diseases that are frequently diagnosed in these individuals. These measures include weight loss for obesity, control of blood glucose for diabetes, control of hyperlipidemia, and treating hypertension if it is present. Ulcerative colitis is unrelated to NAFLD.

A 20-year old patient with a history of Crohn's disease comes to the clinic with persistent diarrhea. What are characteristics of Crohn's disease (select all that apply)? a. Weight loss b. Rectal bleeding c. Abdominal pain d. Toxic megacolon e. Has segmented distribution f. Involves the entire thickness of the bowel wall

a. Weight loss c. Abdominal pain e. Has segmented distribution f. Involves the entire thickness of the bowel wall Crohn's disease may have severe weight loss, segmented distribution through the entire wall of the bowel, and crampy abdominal pain. Rectal bleeding and toxic megacolon are more often seen with ulcerative colitis.

A 90-year-old healthy man is suffering from dysphagia. The nurse explains what age-related change of the GI tract is the most likely cause of his difficulty? a. Xerostomia b. Esophageal cancer c. Decreased taste buds d. Thinner abdominal wall

a. Xerostomia

The nurse identifies a need for further teaching when the patient with hepatitis B makes which statement? a. "I should avoid alcohol completely for as long as a year." b. "I must avoid all physical contact with my family until the jaundice is gone." c. "I should use a condom to prevent spread of the disease to my sexual partner." d. "I will need to rest several times a day, gradually increasing my activity as I tolerate it."

b. "I must avoid all physical contact with my family until the jaundice is gone." The patient with hepatitis B is infectious for 4 to 6 months and precautions to prevent transmission through percutaneous and sexual contact should be maintained until tests for HbsAg or Anti-HBcIgM are negative. Close contact does not have to be avoided but close contacts of the patient should be vaccinated. Alcohol should not be used for at least a year and rest with increasing activity during convalescence is recommended.

The results of a patient's recent endoscopy indicate the presence of peptic ulcer disease (PUD). Which teaching point should the nurse provide to the patient based on this new diagnosis? a. "You'll need to drink at least two to three glasses of milk daily." b. "It would likely be beneficial for you to eliminate drinking alcohol." c. "Many people find that a minced or pureed diet eases their symptoms of PUD." d. "Your medications should allow you to maintain your present diet while minimizing symptoms."

b. "It would likely be beneficial for you to eliminate drinking alcohol."

A 62-year-old woman patient is scheduled for a percutaneous transhepatic cholangiography to restore biliary drainage. The nurse discusses the patient's health history and is most concerned if the patient makes which statement? a. "I am allergic to bee stings." b. "My tongue swells when I eat shrimp." c. "I have had epigastric pain for 2 months." d. "I have a pacemaker because my heart rate was slow."

b. "My tongue swells when I eat shrimp."

A nurse is completing preoperative teaching for a client who will undergo a laparoscopic cholecystectomy. Which of the following should be included in the teaching? a. "The scope will be passed through your rectum" b. "You may have shoulder pain after surgery" c. "The T-tube will remain in place for 1 to 2 weeks" d. "You should limit how often you walk for 1 to 2 weeks"

b. "You may have shoulder pain after surgery"

A patient who is given a bisacodyl (Dulcolax) suppository asks the nurse how long it will take to work. The nurse replies that the patient will probably need to use the bedpan or commode within which time frame after administration? a. 2-5 minutes b. 15-60 minutes c. 2-4 hours d. 6-8 hours

b. 15-60 minutes

On examining a patient 8 hours after having surgery to create a colostomy, what should the nurse expect to find? a. Hyperactive, high-pitched bowel sounds b. A brick-red, puffy stoma that oozes blood c. A purplish stoma, shiny and moist with mucus d. A small amount of liquid fecal drainage from the stoma

b. A brick-red, puffy stoma that oozes blood A normal new colostomy stoma should appear bright red, have mild to moderate edema, and have a small amount of bleeding or oozing of blood when touched. A purplish stoma indicates inadequate blood supply and should be reported. The colostomy will not have any fecal drainage for 2 to 4 days but there may be some earlier mucus or serosanguineous drainage. Bowel sounds after extensive bowel surgery will be diminished or absent

A patient with advanced cirrhosis has a nursing diagnosis of imbalanced nutrition: less than body requirements related to anorexia and inadequate food intake. What would be an appropriate midday snack for the patient? a. Peanut butter and salt-free crackers b. A fresh tomato sandwich with salt-free butter c. Popcorn with salt-free butter and herbal seasoning d. Canned chicken noodle soup with low-protein bread

b. A fresh tomato sandwich with salt-free butter The patient with advanced, complicated cirrhosis requires a high-calorie, high-carbohydrate diet with moderate to low fat. Patients with cirrhosis are at risk for edema and ascites and their sodium intake may be limited. The tomato sandwich with salt-free butter best meets these requirements. Rough foods, such as popcorn, may irritate the esophagus and stomach and lead to bleeding. Peanut butter is high in sodium and fat and canned chicken noodle soup is very high in sodium

A patient with ulcerative colitis undergoes the first phase of a total proctocolectomy with ileal pouch and anal anastomosis. On postoperative assessment of the patient, what should the nurse expect to find? a. A rectal tube set to low continuous suction b. A loop ileostomy with a plastic rod to hold it in place c. A colostomy stoma with an NG tube in place to provide pouch irrigations d. A permanent ileostomy stoma in the right lower quadrant of the abdomen

b. A loop ileostomy with a plastic rod to hold it in place The initial procedure for a total proctocolectomy with ileal pouch and anal anastomosis includes a colectomy, rectal mucosectomy, ileal reservoir construction, ileoanal anastomosis, and a temporary ileostomy. A loop ileostomy is the most common temporary ileostomy and it may be held in place with a plastic rod for the first week. A rectal tube to suction is not indicated in any of the surgical procedures for ulcerative colitis. A colostomy is not used and an NG tube would not be used to irrigate the pouch. A permanent ileostomy stoma would be expected following a total proctocolectomy with a permanent ileostomy.

A nurse is teaching a client who has a new diagnosis of pernicious anemia due to chronic gastritis. Which of the following should be included in the teaching? a. Cells producing gastric acid have been damaged b. A monthly injection of medication is required c. Vitamin K supplements will be administered d. Increased production of intrinsic factor is occurring

b. A monthly injection of medication is required

A patient with an obstruction of the common bile duct has clay-colored fatty stools, among other manifestations. What is the pathophysiologic change that causes this clinical manifestation? a. Soluble bilirubin in the blood excreted into the urine b. Absence of bile salts in the intestine and duodenum, preventing fat emulsion and digestion c. Contraction of the inflamed gallbladder and obstructed ducts, stimulated by cholecystokinin when fats enter the duodenum d. Obstruction of the common duct prevents bile drainage into the duodenum, resulting in congestion of bile in the liver and subsequent absorption into the bloods

b. Absence of bile salts in the intestine and duodenum, preventing fat emulsion and digestion Absence of bile salts in the intestine and duodenum lead to clay-colored stools and steatorrhea. Soluble bilirubin in the blood excreted into the urine leads to dark urine. Contraction of the inflamed gallbladder leads to pain with fatty food intake. Obstruction of the common bile duct prevents bile drainage into the duodenum, with congestion of bile in the liver. Bilirubin absorption in the blood leads to jaundice.

When considering the following causes of acute abdomen pain, the nurse should know that surgery would be indicated for? Select all that apply a. Pancreatitis b. Acute ischemic bowel c. Foreign-body perforation d. Pelvic inflammatory disease e. Ruptured ectopic pregnancy f. Ruptured abdominal aneurysm

b. Acute ischemic bowel c. Foreign-body perforation e. Ruptured ectopic pregnancy f. Ruptured abdominal aneurysm An immediate surgical consult is needed for acute ischemic bowel, foreign-body perforation, ruptured ectopic pregnancy, or ruptured abdominal aneurysm. A diagnostic laparoscopy may be done or a laparotomy may be done to repair a ruptured abdominal aneurysm or remove the appendix. Surgery is not needed for pancreatitis or pelvic inflammatory disease, as these can be diagnosed and treated without surgery.

A colectomy is scheduled for a 38-year-old woman with ulcerative colitis. The nurse should plan to include what prescribed measure in the preoperative preparation of this patient? a. Instruction on irrigating a colostomy b. Administration of a cleansing enema c. A high-fiber diet the day before surgery d. Administration of IV antibiotics for bowel preparation

b. Administration of a cleansing enema

What information should be included when the nurse teaches a patient about colostomy irrigation? a. Infuse 1500 to 2000 mL of warm tap water as irrigation fluid. b. Allow 30 to 45 minutes for the solution and feces to be expelled. c. Insert a firm plastic catheter 3 to 4 inches into the stoma opening. d. Hang the irrigation bag on a hook about 36 inches above the stoma.

b. Allow 30 to 45 minutes for the solution and feces to be expelled. Following infusion of the fluid into the stoma, the solution and feces will take about 30 to 45 minutes to return and the patient can plan to read or perform other quiet activities during the wait time. Between 500 and 1000 mL of warm tap water should be used. A cone tip on the end of the tubing prevents bowel damage that could occur if a stiff plastic catheter is used. Fluid should be elevated about 18 to 24 inches above the stoma, or to about shoulder level, to prevent too rapid infusion of the solution and cramping.

A nurse is providing care to a client who is 1 day postoperative paracentesis. The nurse observes clear, pale-yellow fluid leaking from the operative site. Which of the following is an appropriate nursing intervention? a. Place a clean towel near the drainage site b. Apply a dry, sterile dressing c. Attach an ostomy bag d. Place the client in a supine position

b. Apply a dry, sterile dressing

How is the most common form of malabsorption syndrome treated? a. Administration of antibiotics b. Avoidance of milk and milk products c. Supplementation with pancreatic enzymes d. Avoidance of gluten found in wheat, barley, oats, and rye

b. Avoidance of milk and milk products The most common type of malabsorption syndrome is lactose intolerance and it is managed by restricting the intake of milk and milk products. Antibiotics are used in cases of bacterial infections that cause malabsorption, pancreatic enzyme supplementation is used for pancreatic insufficiency, and restriction of gluten is necessary for control of adult celiac disease (celiac sprue, gluten-induced enteropathy).

The teaching plan for the patient being discharged after an acute episode of upper GI bleeding includes information concerning the importance of (Select all that apply) a. Only taking aspirin with milk or bread products. b. Avoiding taking aspirin and drugs containing aspirin. c. Only taking drugs prescribed by the health care provider. d. Taking all drugs 1 hour before mealtime to prevent further bleeding. e. Reading all OTC drug labels to avoid those containing stearic acid and calcium.

b. Avoiding taking aspirin and drugs containing aspirin. c. Only taking drugs prescribed by the health care provider.

What type of pain does the nurse expect a patient with an ulcer of the posterior portion of the duodenum to experience? a. Pain that occurs after not eating all day b. Back pain that occurs 2 to 4 hours following meals c. Midepigastric pain that is unrelieved with antacids d. High epigastric burning that is relieved with food intake

b. Back pain that occurs 2 to 4 hours following meals Back pain is a common manifestation of ulcers located on the posterior aspect of the duodenum and is important for nurses to keep in mind during assessment of the patient, because the more typical epigastric burning and pain may not be present. Duodenal ulcers are more often relieved by food than are gastric ulcers and when epigastric discomfort occurs, it is lower than that of gastric ulcers. Eating stimulates gastric acid production, increasing discomfort for patients with gastric ulcers, whereas the pain of duodenal ulcers usually occurs several hours after eating.

Which esophageal disorder is described as a precancerous lesion associated with GERD? a. Achalasia b. Barrett's esophagus c. Esophageal strictures d. Esophageal diverticula

b. Barrett's esophagus Barrett's esophagus is an esophageal metaplasia primarily related to gastroesophageal reflux disease (GERD). Achalasia is a rare chronic disorder with the absence of peristalsis of the lower two thirds of the esophagus. Esophageal strictures are narrowing of the esophagus from scarring by many causes. Esophageal diverticula are saclike outpouchings of one or more layers of the esophagus commonly seen above the esophageal sphincter.

A nurse is caring for a client who has advanced cirrhosis with worsening hepatic encephalopathy. Which of the following is an expected assessment finding? (Select all that apply) a. Anorexia b. Change in orientation c. Asterixis d. Ascites e. Fetor hepaticus

b. Change in orientation c. Asterixis e. Fetor hepaticus

What is a risk factor associated with cancer of the pancreas? a. Alcohol intake b. Cigarette smoking c. Exposure to asbestos d. Increased dietary intake of spoiled milk products

b. Cigarette smoking Major risk factors for pancreatic cancer are cigarette smoking, high-fat diet, diabetes, and exposure to benzidine. Pancreatic cancer is not directly associated with alcohol intake, as pancreatitis is. Chronic pancreatitis is a risk factor for pancreatic cancer.

A nurse is providing discharge teaching to a client who is postoperative following open cholecystectomy with T-tube placement. Which of the following instructions should the nurse include in the teaching? (Select all that apply) a. Take baths rather than showers b. Clamp T-tube for 1 to 2 hr before and after meals c. Keep the drainage system above the level of the gallbladder d. Expect to have constipation e. Empty drainage bag every 8 hr

b. Clamp T-tube for 1 to 2 hr before and after meals e. Empty drainage bag every 8 hr

The patient asks the nurse to explain what the physician meant when he said the patient had an anorectal abscess. Which description should the nurse use to explain this to the patient? a. Ulcer in anal wall b. Collection of perianal pus c. Sacrococcygeal hairy tract d. Tunnel leading from the anus or rectum

b. Collection of perianal pus An anorectal abscess is a collection of perianal pus. An ulcer in the anal wall is an anal fissure. Sacrococcygeal hairy tract describes a pilonidal sinus. A tunnel leading from the anus or rectum is an anorectal fistula.

When caring for a patient with irritable bowel syndrome (IBS), what is most important for the nurse to do? a. Recognize that IBS is a psychogenic illness that cannot be definitively diagnosed. b. Develop a trusting relationship with the patient to provide support and symptomatic care. c. Teach the patient that a diet high in fiber will relieve the symptoms of both diarrhea and constipation. d. Inform the patient that new medications for IBS are available and effective for treatment of IBS manifested by either diarrhea or constipation.

b. Develop a trusting relationship with the patient to provide support and symptomatic care. Because there is no definitive treatment for irritable bowel syndrome (IBS) and patients become frustrated and discouraged with uncontrolled symptoms, it is important to develop a trusting relationship that will support the patient as different treatments are implemented and evaluated. Diagnosis of IBS can be established by Rome criteria and by elimination of other problems. Although IBS can be precipitated and aggravated by stress and emotions, it is not a psychogenic illness. High-fiber diets may help but they might also increase the bloating and gas pains of IBS. Medications are available but usually used as a last resort because of side effects.

A patient with oral cancer has a history of heavy smoking, excessive alcohol intake, and personal neglect. During the patient's early postoperative course, what does the nurse anticipate that the patient may need? a. Oral nutritional supplements b. Drug therapy to prevent substance withdrawal symptoms c. Counseling about lifestyle changes to prevent recurrence of the tumor d. Less pain medication because of cross-tolerance with central nervous system (CNS) depressants

b. Drug therapy to prevent substance withdrawal symptoms Measures to assess and treat withdrawal from alcohol should be implemented with patients who have heavy use of this substance because alcohol is a strong risk factor and withdrawal can be life threatening. Tobacco withdrawal also may be uncomfortable for the patient. Nutritional needs may need to be addressed with tube feedings postoperatively and pain medications may need to be increased because of cross-tolerance. Counseling about lifestyle changes is not a priority in the early postoperative course.

What is a nursing intervention that is indicated for a male patient following an inguinal herniorrhaphy? a. Applying heat to the inguinal area b. Elevating the scrotum with a scrotal support c. Applying a truss to support the operative site d. Encouraging the patient to cough and deep breathe

b. Elevating the scrotum with a scrotal support Scrotal edema is a common and painful complication after an inguinal hernia repair and can be relieved in part by application of ice and elevation of the scrotum with a scrotal support. Heat would increase the edema and the discomfort and a truss is used to keep unrepaired hernias from protruding. Coughing is discouraged postoperatively because it increases intraabdominal pressure and stress on the repair site.

M.J. calls to tell the nurse that her 85-year-old mother has been nauseated all day and has vomited twice. Before the nurse hangs up and calls the health care provider, she should instruct M.J. to a. Administer antispasmodic drugs and observe skin turgor. b. Give her mother sips of water and elevate the head of her bed to prevent aspiration. c. Offer her mother a high-protein liquid supplement to drink to maintain her nutritional needs. d. Offer her mother large quantities of Gatorade to drink because older adults are at risk for sodium depletion.

b. Give her mother sips of water and elevate the head of her bed to prevent aspiration.

A large number of children at a public school have developed profuse diarrhea and bloody stools. The school nurse suspects food poisoning related to food from the school cafeteria and requests analysis and culture of which food? a. Chicken b. Ground beef c. Commercially canned fish d. Salads with mayonnaise dressing

b. Ground beef Food poisoning caused by Escherichia coli is characterized by profuse diarrhea, abdominal cramping, and bloody stools and is most often associated with contaminated beef, especially ground beef. Salmonella contamination most often occurs with poultry, staphylococcal infections occur with milk and salad dressings, and botulism occurs with fish and low-acid canned products.

What test will be done before prescribing treatment for the patient with positive testing for HCV? a. Anti-HCV b. HCV genotyping c. HCV RNA quantitation d. Recombinant immunoblot assay (RIBA)

b. HCV genotyping HCV genotyping is done to predict HCV response to drug therapy. Anti-HCV and HCV RNA quantitation are tests completed to diagnose HCV. Recombinant immunoblot assays are used to confirm anti-HCV reactivity.

Which patient is most likely to be diagnosed with short bowel syndrome? a. History of ulcerative colitis b. Had extensive resection of the ileum c. Diagnosed with irritable bowel syndrome d. Had colectomy performed for cancer of the bowel

b. Had extensive resection of the ileum Short bowel syndrome results from extensive resection of portions of the small bowel and would occur if a patient had an extensive resection of the ileum. The other conditions primarily affect the large colon and result in fewer and less severe symptoms.

The patient who is admitted with a diagnosis of diverticulitis and a history of irritable bowel disease and gastroesophageal reflux disease (GERD) has received a dose of Mylanta 30 mL PO. The nurse should evaluate its effectiveness by questioning the patient as to whether which symptom has been resolved? a. Diarrhea b. Heartburn c. Constipation d. Lower abdominal pain

b. Heartburn

Which type of hepatitis is a DNA virus, can be transmitted via exposure to infectious blood or body fluids, is required for HDV to replicate, and increases the risk of the chronic carrier for hepatocellular cancer? a. Hepatitis A (HAV) b. Hepatitis B (HBV) c. Hepatitis C (HCV) d. Hepatitis E (HEV)

b. Hepatitis B (HBV) Hepatitis B virus (HBV) is a DNA virus that is transmitted via infectious blood and body products and is required for hepatitis D virus (HDV) replication, and chronic HBV along with chronic hepatitis C virus (HCV) accounts for 80% of hepatocellular cancer cases. Hepatitis A virus (HAV), HCV, HDV, and HEV are all RNA viruses.

The nurse is planning care for a 68-year-old patient with an abdominal mass and suspected bowel obstruction. Which factor in the patient's history increases the patient's risk for colorectal cancer? a. Osteoarthritis b. History of colorectal polyps c. History of lactose intolerance d. Use of herbs as dietary supplements

b. History of colorectal polyps

The nurse admitting a patient for bariatric surgery obtains the following information from the patient. Which finding should be brought to the surgeon's attention before proceeding with further patient preparation? a. History of hypertension b. History of untreated depression c. History of multiple attempts at weight loss d. History of sleep apnea treated with continuous positive airway pressure (CPAP)

b. History of untreated depression Patients with histories of untreated depression or psychosis are not good candidates for surgery. All other historical information includes medical complications of severe obesity that would help to qualify the patient for the surgery.

What information would have the highest priority to be included in preoperative teaching for a 68-year-old patient scheduled for a colectomy? a. How to care for the wound b. How to deep breathe and cough c. The location and care of drains after surgery d. Which medications will be used during surgery

b. How to deep breathe and cough

A patient who has been vomiting for several days from an unknown cause is admitted to the hospital. What should the nurse anticipate will be included in collaborative care? a. Oral administration of broth and tea b. IV replacement of fluid and electrolytes c. Administration of parenteral antiemetics d. Insertion of a nasogastric (NG) tube for suction

b. IV replacement of fluid and electrolytes The patient with severe or persistent vomiting requires IV replacement of fluids and electrolytes until able to tolerate oral intake to prevent serious dehydration and electrolyte imbalances. Oral fluids are not given until vomiting has been relieved and parenteral antiemetics are often not used until a cause of the vomiting can be established. Nasogastric (NG) intubation may be indicated in some cases but fluid and electrolyte replacement is the first priority.

A nurse is reviewing the laboratory findings of a client who has an acute exacerbation of Crohn's disease. Which of the following laboratory findings is indicative of Crohn's disease? (Select all that apply) a. Increased hematocrit b. Increased erythrocyte sedimentation rate (ESR) c. Increased WBC d. Increased folic acid e. Increased serum albumin

b. Increased erythrocyte sedimentation rate (ESR) c. Increased WBC

The patient with sudden pain in the left upper quadrant radiating to the back and vomiting was diagnosed with acute pancreatitis. What intervention(s) should the nurse expect to include in the patient's plan of care? a. Immediately start enteral feeding to prevent malnutrition. b. Insert an NG and maintain NPO status to allow pancreas to rest. c. Initiate early prophylactic antibiotic therapy to prevent infection. d. Administer acetaminophen (Tylenol) every 4 hours for pain relief.

b. Insert an NG and maintain NPO status to allow pancreas to rest.

A patient with cancer of the stomach at the lesser curvature undergoes a total gastrectomy with an esophagojejunostomy. Postoperatively, what should the nurse teach the patient to expect? a. Rapid healing of the surgical wound b. Lifelong administration of cobalamin c. To be able to return to normal dietary habits d. Close follow-up for development of peptic ulcers in the jejunum

b. Lifelong administration of cobalamin A total gastrectomy removes the parietal cells responsible for secreting intrinsic factor necessary for absorption of cobalamin. Lifelong administration of cobalamin is necessary to prevent the development of pernicious anemia. Wound healing is usually impaired in the patient with a total gastrectomy performed for gastric cancer because of impaired nutritional status before surgery. Following a total gastrectomy, the patient also requires diet modifications as a result of dumping syndrome and postprandial hypoglycemia. Peptic ulcers are not a common finding after total gastrectomy.

Nursing management of the patient with chronic gastritis includes teaching the patient to a. Take antacids before meals to decrease stomach acidity. b. Maintain a nonirritating diet with six small meals a day. c. Eliminate alcohol and caffeine from the diet when symptoms occur. d. Use nonsteroidal antiinflammatory drugs (NSAIDs) instead of aspirin for minor pain relief.

b. Maintain a nonirritating diet with six small meals a day. A nonirritating diet with six small meals a day is recommended to help control the symptoms of gastritis. Nonsteroidal antiinflammatory drugs (NSAIDs) are often as irritating to the stomach as aspirin and should not be used in the patient with gastritis. Antacids are often used for control of symptoms but have the best neutralizing effect if taken after meals. Alcohol and caffeine should be eliminated entirely because they may precipitate gastritis.

Following a patient's esophagogastrostomy for cancer of the esophagus, what is most important for the nurse to do? a. Report any bloody drainage from the NG tube. b. Maintain the patient in semi-Fowler's or Fowler's position. c. Monitor for abdominal distention that may disrupt the surgical site. d. Expect to find decreased breath sounds bilaterally because of the surgical approach.

b. Maintain the patient in semi-Fowler's or Fowler's position. Following esophageal surgery, the patient should be positioned in semi-Fowler's or Fowler's position to prevent reflux and aspiration of gastric sections. NG drainage is expected to be bloody for 8 to 12 hours postoperatively. Abdominal distention is not a major concern following esophageal surgery and even though the thorax may be opened during the surgery, clear breath sounds should be expected in all areas of the lungs.

The patient has persistent and continuous pain at McBurney's point. The nursing assessment reveals rebound tenderness and muscle guarding with the patient preferring to lie still with the right leg flexed. What should the nursing interventions for this patient include? a. Laxatives to move the constipated bowel b. NPO status in preparation for possible appendectomy c. Parenteral fluids and antibiotic therapy for 6 hours before surgery d. NG tube inserted to decompress the stomach and prevent aspiration

b. NPO status in preparation for possible appendectomy The patient's manifestations are characteristic of appendicitis. After laboratory test and CT scan confirmation, the patient will have surgery. Laxatives are not used. The 6 hours of fluids and antibiotics preoperatively would be used only if the appendix was ruptured. The NG tube is more likely to be used with abdominal trauma.

A nurse is preparing to administer pancrelipase (Viokase) to a client who has pancreatitis. Which of the following is an appropriate nursing action? a. Administer medication 30 minutes after a snack b. Offer a glass of water following medication administration c. Administer the medication 30 minutes before meals d. Sprinkle the contents on peanut butter

b. Offer a glass of water following medication administration

In contrast to diverticulitis, the patient with diverticulosis a. Has rectal bleeding. b. Often has no symptoms. c. Has localized cramping pain. d. Frequently develops peritonitis.

b. Often has no symptoms.

The nurse is caring for a 45-year-old woman with a herniated lumbar disc. The patient realizes that weight loss is necessary to lessen back strain. The patient is 5'6" tall and weighs 186 lb (84.5 kg) with a body mass index (BMI) of 28 kg/m2. The nurse explains to the patient that this measurement places her in which of the following weight categories? a. Normal weight b. Overweight c. Obese d. Severely obese

b. Overweight

When caring for a patient following a glossectomy with dissection of the floor of the mouth and a radical neck dissection for cancer of the tongue, what is the nurse's primary concern? a. Relief of pain b. Patent airway c. Positive body image d. Tube feedings to provide nutrition

b. Patent airway Because surgical treatment of oral cancers involves extensive excision, a tracheostomy is usually performed with the radical dissections. The first goal of care is that the patient will have a patent airway. The other goals are appropriate but of lesser priority.

Which statements are characteristic of the uses of antacids for peptic ulcer disease (select all that apply)? a. Used in patients with verified H. pylori b. Patients frequently noncompliant with use c. Prevent conversion of pepsinogen to pepsin d. Cover the ulcer, protecting it from erosion by acids e. High incidence of side effects and contraindications f. High dose and frequency stimulate release of gastrin

b. Patients frequently noncompliant with use c. Prevent conversion of pepsinogen to pepsin f. High dose and frequency stimulate release of gastrin Antacids need a high dose and frequency, which may lead to noncompliance; prevent the conversion of pepsinogen to pepsin; and may stimulate the release of gastrin. Amoxicillin/clarithromycin/omeprazole are used in patients with verified H. pylori. Sucralfate (Carafate) covers the ulcer to protect it from acid erosion.

A patient with acute hepatitis B is being discharged in 2 days. In the discharge teaching plan the nurse should include instructions to a. Avoid alcohol for the first 3 weeks. b. Use a condom during sexual intercourse. c. Have family members get an injection of immunoglobulin. d. Follow a low-protein, moderate-carbohydrate, moderate-fat diet.

b. Use a condom during sexual intercourse.

A patient is admitted to the emergency department with profuse bright-red hematemesis. During the initial care of the patient, what is the nurse's first priority? a. Establish two IV sites with large-gauge catheters b. Perform a focused nursing assessment of the patient's status c. Obtain a thorough health history to assist in determining the cause of the bleeding d. Perform a gastric lavage with cool tap water in preparation for endoscopic examination

b. Perform a focused nursing assessment of the patient's status Although all of the interventions may be indicated when a patient has upper GI bleeding, the first nursing priority with bright-red (arterial) blood is to perform a focused assessment of the patient's condition, with emphasis on blood pressure (BP), pulse, and peripheral perfusion to determine the presence of hypovolemic shock.

A nurse is caring for a client who has a new diagnosis of hepatitis C. Which of the following is an expected laboratory finding? a. Presence of immunoglobulin G antibodies (IgG) b. Presence of enzyme immunoassay (EIA) c. Aspartate aminotransferase (AST) 35 units/L d. Alanine aminotransferase (ALT) 15 IU/L

b. Presence of enzyme immunoassay (EIA)

An important nursing intervention for a patient with a small intestinal obstruction who has an NG tube is to a. Offer ice chips to suck PRN. b. Provide mouth care every 1 to 2 hours. c. Irrigate the tube with normal saline every 8 hours. d. Keep the patient supine with the head of the bed elevated 30 degrees.

b. Provide mouth care every 1 to 2 hours. Mouth care should be done frequently for the patient with a small intestinal obstruction who has an NG tube because of vomiting, fecal taste and odor, and mouth breathing. No ice chips are allowed when a patient is NPO because of a bowel obstruction. The NG tube should be checked for patency and irrigated as ordered. The position of the patient should be one of comfort.

A 40-year-old severely obese female patient with type 2 diabetes wants to lose weight. After learning about the surgical procedures, she thinks a combination of restrictive and malabsorptive surgery would be best. Which procedure should the nurse teach her about? a. Lipectomy b. Roux-en-Y bypass c. Adjustable gastric banding d. Vertical sleeve gastrectomy

b. Roux-en-Y bypass The Roux-en-Y gastric bypass is a common combination of restrictive (limiting the size of the stomach) and malabsorptive (less food is absorbed) surgery. Lipectomy is used to remove unsightly flabby folds of adipose tissue. Adjustable gastric banding is the most common restrictive procedure. Vertical sleeve gastrectomy is a restrictive procedure that preserves stomach function.

What characteristics describe adjustable gastric banding? Select all that apply. a. 85% of the stomach is removed b. Stomach restriction can be reversed c. Eliminates hormones that stimulate hunger d. Malabsorption of fat-soluble vitamins occur e. Inflatable band allows for modification of gastric stoma size f. Stomach with a gastric pouch surgically anastomosed to the jejunum

b. Stomach restriction can be reversed e. Inflatable band allows for modification of gastric stoma size The adjustable gastric banding procedure is reversible and allows a change in gastric stoma size by inflation or deflation of the band around the fundus of the stomach. The vertical sleeve gastrectomy removes 85% of the stomach and eliminates the hormones produced in the stomach that stimulate hunger. The biliopancreatic diversion is a maladaptive surgery that prevents absorption of nutrients, including fat-soluble vitamins. The Roux-en-Y gastric bypass reduces the stomach size with a gastric pouch anastomosed to the small intestine, so it is both restrictive and malabsorptive.

The patient had a car accident and was "scared to death." The patient is now reporting constipation. What affecting the gastrointestinal (GI) tract does the nurse know could be contributing to the constipation? a. The patient is too nervous to eat or drink, so there is no stool. b. The sympathetic nervous system was activated, so the GI tract was slowed. c. The parasympathetic nervous system is now functioning to slow the GI tract. d. The circulation in the GI system has been increased, so less waste is removed.

b. The sympathetic nervous system was activated, so the GI tract was slowed.

What should the nurse emphasize when teaching patients at risk for upper GI bleeding to prevent bleeding episodes? a. All stools and vomitus must be tested for the presence of blood. b. The use of over-the-counter (OTC) medications of any kind should be avoided. c. Antacids should be taken with all prescribed medications to prevent gastric irritation. d. Misoprostol (Cytotec) should be used to protect the gastric mucosa in individuals with peptic ulcers.

b. The use of over-the-counter (OTC) medications of any kind should be avoided. All over-the-counter (OTC) drugs should be avoided because their contents may include drugs that are contraindicated because of the irritating effects on the gastric mucosa. Patients are taught to test suspicious vomitus or stools for occult blood but all stools do not need to be tested. Antacids cannot be taken with all medications because they prevent the absorption of many drugs. Misoprostol is used to protect the gastric mucosa in patients who must take NSAIDs for other conditions because it inhibits acid secretion stimulated by NSAIDs.

What laboratory test results should the nurse expect to find in a patient with cirrhosis? a. Serum albumin: 7.0 g/dL (70 g/L) b. Total bilirubin: 3.2 mg/dL (54.7 mmol/L) c. Serum cholesterol: 260 mg/dL (6.7 mmol/L) d. Aspartate aminotransferase (AST): 6.0 U/L (0.1 mkat/L)

b. Total bilirubin: 3.2 mg/dL (54.7 mmol/L) Serum bilirubin, both direct and indirect, would be expected to be increased in cirrhosis. Serum albumin and cholesterol are decreased and liver enzymes, such as aspartate aminotransferase (AST) and alanine aminotransferase (ALT), are initially elevated but may be normal in end-stage liver disease.

A physician just told a patient that she has a volvulus. When the patient asks the nurse what this is, what is the best description for the nurse to give her? a. Bowel folding on itself b. Twisting of bowel on itself c. Emboli of arterial supply to the bowel d. Protrusion of bowel in weak or abnormal opening

b. Twisting of bowel on itself Volvulus is the bowel twisting on itself. The bowel folding on itself is intussusception. Emboli of arterial blood supply to the bowel is vascular obstruction. Protrusion of bowel in a weak or abnormal opening is a hernia

A nurse is completing an admission assessment of a client who has a small bowel obstruction. Which of the following findings should the nurse report to the provider? (Select all that apply) a. Profuse emesis prior to insertion of the NG tube b. Urine specific gravity 1.040 c. Hematocrit 60% d. Serum potassium 3.0 mEq/L e. WBC 10,000/uL

b. Urine specific gravity 1.040 c. Hematocrit 60% d. Serum potassium 3.0 mEq/L

A patient with chronic cholecystitis asks the nurse whether she will need to continue a low-fat diet after she has a cholecystectomy. What is the best response by the nurse? a. "A low-fat diet will prevent the development of further gallstones and should be continued." b. "Yes; because you will not have a gallbladder to store bile, you will not be able to digest fats adequately." c. "A low-fat diet is recommended for a few weeks after surgery until the intestine adjusts to receiving a continuous flow of bile." d. "Removal of the gallbladder will eliminate the source of your pain associated with fat intake, so you may eat whatever you like."

c. "A low-fat diet is recommended for a few weeks after surgery until the intestine adjusts to receiving a continuous flow of bile." After removal of the gallbladder, bile drains directly from the liver into the duodenum and a low-fat diet is recommended until adjustment to this change occurs. Most patients tolerate a regular diet with moderate fats but should avoid excessive fats, as large volumes of bile previously stored in the gallbladder are not available. Steatorrhea could occur with a large fat intake.

The nurse provides discharge instructions for a 64-year-old woman with ascites and peripheral edema related to cirrhosis. Which statement, if made by the patient, indicates teaching was effective? a. "It is safe to take acetaminophen up to four times a day for pain." b. "Lactulose (Cephulac) should be taken every day to prevent constipation." c. "Herbs and other spices should be used to season my foods instead of salt." d. "I will eat foods high in potassium while taking spironolactone (Aldactone)."

c. "Herbs and other spices should be used to season my foods instead of salt."

A patient is diagnosed with celiac disease following a workup for iron-deficiency anemia and decreased bone density. The nurse identifies that additional teaching about disease management is needed when the patient makes which statement? a. "I should ask my close relatives to be screened for celiac disease." b. "If I do not follow the gluten-free diet, I might develop a lymphoma." c. "I don't need to restrict gluten intake because I don't have diarrhea or bowel symptoms." d. "It is going to be difficult to follow a gluten-free diet because it is found in so many foods."

c. "I don't need to restrict gluten intake because I don't have diarrhea or bowel symptoms." The autoimmune process associated with celiac disease continues as long as the body is exposed to gluten, regardless of the symptoms it produces, and a lifelong gluten-free diet is necessary. The other statements regarding celiac disease are all true

The nurse determines that teaching for the patient with peptic ulcer disease has been effective when the patient makes which statement? a. "I should stop all my medications if I develop any side effects." b. "I should continue my treatment regimen as long as I have pain." c. "I have learned some relaxation strategies that decrease my stress." d. "I can buy whatever antacids are on sale because they all have the same effect."

c. "I have learned some relaxation strategies that decrease my stress." Increased vagal stimulation from emotional stress causes hypersecretion of hydrochloric acid and stress reduction is an important part of the patient's management of peptic ulcers, especially duodenal ulcers. If side effects to medications develop, the patient should notify the health care provider before altering the drug regimen. Although effective treatment will promote pain relief in several days, the treatment regimen should be continued until there is evidence that the ulcer has healed completely. Interchanging brands and preparations of antacids and histamine (H2)- receptor blockers without checking with health care providers may cause harmful side effects and patients should take only prescribed medications.

The nurse identifies a need for additional teaching when a patient with acute infectious diarrhea makes which statement? a. "I can use A&D ointment or Vaseline jelly around the anal area to protect my skin." b. "Gatorade is a good liquid to drink because it replaces the fluid and salts I have lost." c. "I may use over-the-counter Imodium or Parepectolin when I need to control the diarrhea." d. "I must wash my hands after every bowel movement to prevent spreading the diarrhea to my family."

c. "I may use over-the-counter Imodium or Parepectolin when I need to control the diarrhea." Antiperistaltic agents, such as loperamide (Imodium) and paregoric, should not be used in infectious diarrhea because of the potential of prolonging exposure to the infectious agent. Demulcent agents may be used to coat and protect mucous membranes in these cases. The other options are all appropriate measures to use in cases of infectious diarrhea.

The nurse determines that further discharge instruction is needed when the patient with acute pancreatitis makes which statement? a. "I should observe for fat in my stools." b. "I must not use alcohol to prevent future attacks of pancreatitis." c. "I shouldn't eat any salty foods or foods with high amounts of sodium." d. "I will need to continue to monitor my blood glucose levels until my pancreas is healed."

c. "I shouldn't eat any salty foods or foods with high amounts of sodium." Sodium restriction is not indicated for patients recovering from acute pancreatitis but the stools should be observed for steatorrhea, indicating that fat digestion is impaired, and glucose levels may be monitored for indication of impaired β-cell function. Alcohol is a primary cause of pancreatitis and should not be used.

A nurse is completing discharge teaching to a client who is postoperative following fundoplication. Which of the following statements by the client indicates understanding of the teaching? a. "When sitting in my lounge chair after a meal, I will lower the back of it" b. "I will try to eat three large meals a day" c. "I will elevate the head of my bed on blocks" d. "When sleeping, I will lay on my left side"

c. "I will elevate the head of my bed on blocks"

A nurse is completing discharge teaching for a client who has an infection due to H. pylori. Which of the following statements by the client indicates understanding of the teaching? a. "I will continue my prescription for corticosteroids" b. "I will schedule a CT scan to monitor improvement" c. "I will take a combination of medications for treatment" d. "I will have my throat swabbed to recheck for this bacteria"

c. "I will take a combination of medications for treatment"

A nurse is completing teaching to a client who has a new prescription for famotidine (Pepcid). Which of the following statements by the client indicates understanding of the teaching? a. "This medicine coats the lining of my stomach" b. "This should stop the pain right away" c. "I will take my pill at meal time" d. "I will monitor for bleeding from my nose"

c. "I will take my pill at meal time"

A patient diagnosed with chronic hepatitis B asks about drug therapy to treat the disease. What is the most appropriate response by the nurse? a. "Only chronic hepatitis C is treatable and primarily with antiviral agents and interferon." b. "There are no specific drug therapies that are effective for treating acute viral hepatitis." c. "Interferon combined with lamivudine (Epivir) will decrease viral load and prevent complications." d. "There are no drugs used for the treatment of viral hepatitis because of the risk of additional liver damage."

c. "Interferon combined with lamivudine (Epivir) will decrease viral load and prevent complications." Nucleoside and nucleotide analogs (e.g., lamivudine), ribavirin, and pegylated interferon are used to treat chronic hepatitis B or C. Protease inhibitors are also used to treat chronic hepatitis C. No specific drugs are effective in treating acute viral hepatitis, although supportive drugs, such as antiemetics, sedatives, or antipruritics, may be used for symptom control.

A nurse is reinforcing teaching for a client who has a prescription for sulfasalazine (Azulfidine). Which of the following should the nurse include in the teaching? a. "Take the medication 1 or 2 hr after eating" b. "This medication may cause yellowing of the sclera" c. "Notify the provider if you experience a sore throat" d. "This medication may cause your stools to turn black"

c. "Notify the provider if you experience a sore throat"

The nurse instructs a 50-year-old woman about cholestyramine to reduce pruritis caused by gallbladder disease. Which statement by the patient to the nurse indicates she understands the instructions? a. "This medication will help me digest fats and fat-soluble vitamins." b. "I will apply the medicated lotion sparingly to the areas where I itch." c. "The medication is a powder and needs to be mixed with milk or juice." d. "I should take this medication on an empty stomach at the same time each day."

c. "The medication is a powder and needs to be mixed with milk or juice."

The family members of a patient with hepatitis A ask if there is anything that will prevent them from developing the disease. What is the best response by the nurse? a. "No immunization is available for hepatitis A, nor are you likely to get the disease." b. "All family members should receive the hepatitis A vaccine to prevent or modify the infection." c. "Those who have had household or close contact with the patient should receive immune globulin." d. "Only those individuals who have had sexual contact with the patient should receive immunization."

c. "Those who have had household or close contact with the patient should receive immune globulin." Individuals who have been exposed to hepatitis A through household contact or foodborne outbreaks should be given immune globulin within 1 to 2 weeks of exposure to prevent or modify the illness. Hepatitis A vaccine is used to provide preexposure immunity to the virus and is indicated for individuals at high risk for hepatitis A exposure. Although hepatitis A may be spread by sexual contact, the risk is higher for transmission with the oral- fecal route.

In developing a weight reduction program with a 45-year-old female patient who weighs 197 lb, the nurse encourages the patient to set a weight loss goal of how many pounds in 4 weeks? a. 1-2 b. 3-5 c. 4-8 d. 5-10

c. 4-8

The nurse is caring for a group of patients. Which patient is at highest risk for pancreatic cancer? a. A 38-year-old Hispanic female who is obese and has hyperinsulinemia b. A 23-year-old who has cystic fibrosis-related pancreatic enzyme insufficiency c. A 72-year-old African American male who has smoked cigarettes for 50 years d. A 19-year-old who has a 5-year history of uncontrolled type 1 diabetes mellitus

c. A 72-year-old African American male who has smoked cigarettes for 50 years

The pernicious anemia that may accompany gastritis is due to a. Chronic autoimmune destruction of cobalamin stores in the body. b. Progressive gastric atrophy from chronic breakage in the mucosal barrier and blood loss. c. A lack of intrinsic factor normally produced by acid-secreting cells of the gastric mucosa. d. Hyperchlorhydria resulting from an increase in acid-secreting parietal cells and degradation of RBCs.

c. A lack of intrinsic factor normally produced by acid-secreting cells of the gastric mucosa.

In discussing long-term management with the patient with alcoholic cirrhosis, what should the nurse advise the patient? a. A daily exercise regimen is important to increase the blood flow through the liver. b. Cirrhosis can be reversed if the patient follows a regimen of proper rest and nutrition. c. Abstinence from alcohol is the most important factor in improvement of the patient's condition. d. The only over-the-counter analgesic that should be used for minor aches and pains is acetaminophen.

c. Abstinence from alcohol is the most important factor in improvement of the patient's condition. Abstinence from alcohol is very important in alcoholic cirrhosis and may result in improvement if started when liver damage is limited. Although further liver damage may be reduced by rest and nutrition, most changes in the liver cannot be reversed. Exercise does not promote portal circulation and very moderate exercise is recommended. Acetaminophen should not be used by the patient with alcoholic cirrhosis because this liver is more sensitive to the hepatotoxicity of acetaminophen

The nurse receives an order for a parenteral dose of promethazine (Phenergan) and prepares to administer the medication to a 38-year-old male patient with nausea and repeated vomiting. Which action is most important for the nurse to take? a. Administer the medication subcutaneously for fast absorption. b. Administer the medication into an arterial line to prevent extravasation. c. Administer the medication deep into the muscle to prevent tissue damage. d. Administer the medication with 0.5 mL of lidocaine to decrease injection pain.

c. Administer the medication deep into the muscle to prevent tissue damage.

A 54-year-old patient admitted with diabetes mellitus, malnutrition, osteomyelitis, and alcohol abuse has a serum amylase level of 280 U/L and a serum lipase level of 310 U/L. To what diagnosis does the nurse attribute these findings? a. Malnutrition b. Osteomyelitis c. Alcohol abuse d. Diabetes mellitus

c. Alcohol abuse

During the incubation period of viral hepatitis, what should the nurse expect the patient to report? a. Pruritus and malaise b. Dark urine and easy fatigability c. Anorexia and right upper quadrant discomfort d. Constipation or diarrhea with light-colored stools

c. Anorexia and right upper quadrant discomfort Incubation symptoms occur before the onset of jaundice and include a variety of gastrointestinal (GI) symptoms as well as discomfort and heaviness in the upper right quadrant of the abdomen. Pruritus, dark urine, and light-colored stools occur with the onset of jaundice in the acute phase. Easy fatigability and malaise are seen in the convalescent phase as jaundice disappears.

The patient with chronic gastritis is being put on a combination of medications to eradicate H. pylori. Which drugs does the nurse know will probably be used for this patient? a. Antibiotic(s), antacid, and corticosteroid b. Antibiotic(s), aspirin, and antiulcer/protectant c. Antibiotic(s), proton pump inhibitor, and bismuth d. Antibiotic(s) and nonsteroidal antiinflammatory drugs (NSAIDs)

c. Antibiotic(s), proton pump inhibitor, and bismuth

The patient is receiving the following medications. Which one is prescribed to relieve symptoms rather than treat a disease? a. Corticosteroids b. 6-Mercaptopurine c. Antidiarrheal agents d. Sulfasalazine (Azulfidine)

c. Antidiarrheal agents Antidiarrheal agents only relieve symptoms. Corticosteroids, 6-mercaptopurine, and sulfasalazine (Azulfidine) are used to treat and control inflammation with various diseases.

A nurse is planning care for a client who has a small bowel obstruction and a NG tube in place. Which of the following nursing interventions should be included in the plan of care? (Select all that apply) a. Subtract the NG drainage from the client's output b. Irrigate the NG tube every 8 hr c. Assess bowel sounds d. Provide oral hygiene every 2 hr e. Clamp the NG tube during ambulation

c. Assess bowel sounds d. Provide oral hygiene every 2 hr e. Clamp the NG tube during ambulation

The patient has hepatic encephalopathy. What is a priority nursing intervention to keep the patient safe? a. Turn the patient every 3 hours. b. Encourage increasing ambulation. c. Assist the patient to the bathroom. d. Prevent constipation to reduce ammonia production.

c. Assist the patient to the bathroom. The patient may not be oriented or able to walk to the bathroom alone because of hyperreflexia, asterixis, or decreased motor coordination. Turning should be done every 2 hours to prevent skin breakdown. Activity is limited to decrease ammonia as a by-product of protein metabolism. Although constipation will be prevented, it will not keep the patient safe.

Which clinical manifestations of inflammatory bowel disease are common to both patients with ulcerative colitis (UC) and Crohn's disease? Select all that apply? a. Restricted to rectum b. Strictures are common. c. Bloody, diarrhea stools d. Cramping abdominal pain e. Lesions penetrate intestine.

c. Bloody, diarrhea stools d. Cramping abdominal pain

A nurse is assessing a client who has been taking prednisone following an exacerbation of inflammatory bowel disease (IBD). Which of the following assessment findings is the highest priority? a. Client reports difficulty sleeping b. Blood glucose at 0800 is 140 mg/dL c. Client reports having a sore throat d. Client reports gaining 4 lb in last 6 months

c. Client reports having a sore throat

The medications prescribed for the patient with inflammatory bowel disease include cobalamin and iron injections. What is the rationale for using these drugs? a. Alleviate stress b. Combat infection c. Correct malnutrition d. Improve quality of life

c. Correct malnutrition Cobalamin and iron injections will help to correct malnutrition. Correcting malnutrition will also indirectly help to improve quality of life and fight infections.

What laboratory findings are expected in ulcerative colitis as a result of diarrhea and vomiting? a. Increased albumin b. Elevated white blood cells (WBCs) c. Decreased Na+, K+, Mg+, Cl-, and HCO3- d. Decreased hemoglobin (Hgb) and hematocrit (Hct)

c. Decreased Na+, K+, Mg+, Cl-, and HCO3- In the patient with ulcerative colitis, decreased Na+, K+, Mg+, Cl-, and HCO3- are a result of diarrhea and vomiting. Hypoalbuminemia may be present. Elevated WBCs occur with toxic megacolon. Decreased hemoglobin (Hgb) and hematocrit (Hct) occur with bloody diarrhea.

Which manifestations may be seen in the patient with cirrhosis related to esophageal varices? a. Jaundice, peripheral edema, and ascites from increased intrahepatic pressure and dysfunction b. Loss of the small bile ducts and cholestasis and cirrhosis in patients with other autoimmune disorders c. Development of collateral channels of circulation in inelastic, fragile esophageal veins as a result of portal hypertension d. Scarring and nodular changes in the liver lead to compression of the veins and sinusoids, causing resistance of blood flow through the liver from the portal vein

c. Development of collateral channels of circulation in inelastic, fragile esophageal veins as a result of portal hypertension Esophageal varices occur when collateral channels of circulation develop inelastic fragile veins as a result of portal hypertension. Portal hypertension is from scarring and nodular changes in the liver leading to compression of the veins and sinusoids, causing resistance of blood flow through the liver from the portal vein. It contributes to peripheral edema and ascites. Jaundice is from the inability of the liver to conjugate bilirubin. Biliary cirrhosis causes the loss of small bile ducts and ultimate cholestasis in patients with other autoimmune disorders

A nurse in a clinic is reviewing the laboratory reports of a client who has suspected cholelithiasis. Which of the following is an expected finding? a. Serum albumin 4.1 g/dL b. WBC 9,511/uL c. Direct bilirubin 2.1 mg/dL d. Serum cholesterol 171 mg/dL

c. Direct bilirubin 2.1 mg/dL

Older patients may have cardiac or renal insufficiency and may be more susceptible to problems from vomiting and antiemetic drug side effects. What nursing intervention is most important to implement with these patients? a. Keep the patient flat in bed to decrease dizziness. b. Keep the patient NPO until nausea and vomiting has stopped. c. Do hourly visual checks or use a sitter to keep the patient safe. d. Administer IV fluids as rapidly as possible to prevent dehydration.

c. Do hourly visual checks or use a sitter to keep the patient safe. Implementing safety precautions (placement close to the nurses' station, call bell in reach, hourly visual checks, use of sitters) is the priority. The patient would not be kept in a flat position because of the potential for aspiration of vomitus. Because the older patient is more likely to have cardiac or renal insufficiency, the patient's fluid and electrolyte status are monitored more closely (laboratory, intake and output). Monitor vital signs along with breath sounds. Assess mucous membranes, skin turgor, and color to assess for dehydration. Assess level of consciousness closely. Check dosing of antiemetics. Assess for weakness and fatigue.

The nurse is assessing a 50-year-old woman admitted with a possible bowel obstruction. Which assessment finding would be expected in this patient? a. Tympany to abdominal percussion b. Aortic pulsation visible in epigastric region c. High-pitched sounds on abdominal auscultation d. Liver border palpable 1 cm below the right costal margin

c. High-pitched sounds on abdominal auscultation

A 60-year-old African American patient is afraid she might have anal cancer. What assessment finding puts her at high risk for anal cancer? a. Alcohol use b. Only one sexual partner c. Human papillomavirus (HPV) d. Use of a condom with sexual intercourse

c. Human papillomavirus (HPV) Human papillomavirus (HPV) is associated with about 80% of anal cancer cases. Other risk factors include multiple sexual partners, smoking, receptive anal sex, and HIV infection, as well as being female, age 60, and African American. The other options are not considered risk factors for anal cancer.

To treat a cirrhotic patient with hepatic encephalopathy, lactulose (Cephulac), rifaximin (Xifaxan), and a proton pump inhibitor are ordered. The patient's family wants to know why the laxative is ordered. What is the best explanation the nurse can give to the patient's family? a. It reduces portal venous pressure. b. It eliminates blood from the GI tract. c. It traps ammonia and eliminates it in the feces. d. It decreases bacteria to decrease ammonia formation.

c. It traps ammonia and eliminates it in the feces. Ammonia must be reduced to treat hepatic encephalopathy. The laxative, lactulose, decreases ammonia by trapping the ammonia and eliminating it in the feces. A β-adrenergic blocker will be used to decrease portal venous pressure and decrease variceal bleeding. The proton pump inhibitor will decrease gastric acidity but will not eliminate blood already in the GI tract. The rifaximin will decrease bacterial flora and therefore decrease ammonia formation from protein metabolism.

What should the nurse teach the patient with diverticulosis to do? a. Use anticholinergic drugs routinely to prevent bowel spasm. b. Have an annual colonoscopy to detect malignant changes in the lesions. c. Maintain a high-fiber diet and use bulk laxatives to increase fecal volume. d. Exclude whole grain breads and cereals from the diet to prevent irritating the bowel.

c. Maintain a high-fiber diet and use bulk laxatives to increase fecal volume. Formation of diverticula is common when decreased bulk of stool, combined with a more narrowed lumen in the sigmoid colon, causes high intraluminal pressures that result in saccular dilation or outpouching of the mucosa through the muscle of the intestinal wall. To prevent the high intraluminal pressure, fecal volume should be increased with use of high-fiber diets and bulk laxatives, such as psyllium (Metamucil). Anticholinergic drugs are used only during an acute episode of diverticulitis and the lesions are not premalignant.

During the treatment of the patient with bleeding esophageal varices, what is the most important thing the nurse should do? a. Prepare the patient for immediate portal shunting surgery. b. Perform guaiac testing on all stools to detect occult blood. c. Maintain the patient's airway and prevent aspiration of blood. d. Monitor for the cardiac effects of IV vasopressin and nitroglycerin.

c. Maintain the patient's airway and prevent aspiration of blood. Bleeding esophageal varices are a medical emergency. During an episode of bleeding, management of the airway and prevention of aspiration of blood are critical factors. Occult blood as well as fresh blood from the GI tract would be expected. Vasopressin causes vasoconstriction, decreased heart rate, and decreased coronary blood flow. IV nitroglycerin may be given with the vasopressin to counter these side effects. Portal shunting surgery is performed for esophageal varices but not during an acute hemorrhage

What is the patient with chronic pancreatitis more likely to have than the patient with acute pancreatitis? a. The need to abstain from alcohol b. Experience acute abdominal pain c. Malabsorption and diabetes mellitus d. Require a high-carbohydrate, high-protein, low-fat diet

c. Malabsorption and diabetes mellitus Chronic damage to the pancreas causes a deficiency of digestive enzymes and insulin resulting in malabsorption and diabetes mellitus. Abstinence from alcohol is necessary in both types of pancreatitis, as is a high-carbohydrate, high-protein, and low-fat diet. Although abdominal pain is a major manifestation of chronic pancreatitis, more commonly a constant heavy, gnawing feeling occurs.

The patient is having an esophagoenterostomy with anastomosis of a segment of the colon to replace the resected portion. What initial postoperative care should the nurse expect when this patient returns to the nursing unit? a. Turn, deep breathe, cough, and use spirometer every 4 hours. b. Maintain an upright position for at least 2 hours after eating. c. NG will have bloody drainage, and it should not be repositioned d. Keep in a supine position to prevent movement of the anastomosis.

c. NG will have bloody drainage, and it should not be repositioned

What treatment measure is used in the management of the patient with acute pancreatitis? a. Surgery to remove the inflamed pancreas b. Pancreatic enzyme supplements administered with meals c. Nasogastric (NG) suction to prevent gastric contents from entering the duodenum d. Endoscopic pancreatic sphincterotomy using endoscopic retrograde cholangiopancreatography (ERCP)

c. Nasogastric (NG) suction to prevent gastric contents from entering the duodenum Pancreatic rest and suppression of secretions are promoted by preventing any gastric contents from entering the duodenum, which would stimulate pancreatic activity. Surgery is not indicated for acute pancreatitis but may be used to drain abscesses or cysts. An endoscopic retrograde cholangiopancreatography (ERCP) pancreatic sphincterotomy may be performed when pancreatitis is related to gallstones. Pancreatic enzyme supplements are necessary in chronic pancreatitis if a deficiency in secretion occurs

The patient has a diagnosis of a biliary obstruction from gallstones. What type of jaundice is the patient experiencing and what serum bilirubin results would be expected? a. Hemolytic jaundice with normal conjugated bilirubin b. Posthepatic icteris with decreased unconjugated bilirubin c. Obstructive jaundice with elevated unconjugated and conjugated bilirubin d. Hepatocellular jaundice with altered conjugated bilirubin in severe disease

c. Obstructive jaundice with elevated unconjugated and conjugated bilirubin Gallstones cause obstructive or posticteric jaundice and may elevate both conjugated and unconjugated bilirubin.

In planning care for the patient with Crohn's disease, the nurse recognizes that a major difference between ulcerative colitis and Crohn's disease is that Crohn's disease a. Frequently results in toxic megacolon. b. Causes fewer nutritional deficiencies than ulcerative colitis. c. Often recurs after surgery, whereas ulcerative colitis is curable with a colectomy. d. Is manifested by rectal bleeding and anemia more frequently than is ulcerative colitis.

c. Often recurs after surgery, whereas ulcerative colitis is curable with a colectomy.

A patient with acute pancreatitis has a nursing diagnosis of pain related to distention of the pancreas and peritoneal irritation. In addition to effective use of analgesics, what should the nurse include in this patient's plan of care? a. Provide diversional activities to distract the patient from the pain. b. Provide small, frequent meals to increase the patient's tolerance to food. c. Position the patient on the side with the head of the bed elevated 45 degrees for pain relief. d. Ambulate the patient every 3 to 4 hours to increase circulation and decrease abdominal congestion.

c. Position the patient on the side with the head of the bed elevated 45 degrees for pain relief. Positions that flex the trunk and draw the knees up to the abdomen help to relieve the pain of acute pancreatitis and positioning the patient on the side with the head elevated decreases abdominal tension. Diversional techniques are not as helpful as positioning in controlling the pain. The patient is usually NPO because food intake increases the pain and inflammation. Bed rest is indicated during the acute attack because of hypovolemia and pain.

Following administration of a dose of metoclopramide (Reglan) to the patient, the nurse determines that the medication has been effective when what is noted? a. Decreased blood pressure b. Absence of muscle tremors c. Relief of nausea and vomiting d. No further episodes of diarrhea

c. Relief of nausea and vomiting

A patient with pancreatic cancer is admitted to the hospital for evaluation of possible treatment options. The patient asks the nurse to explain the Whipple procedure that the surgeon has described. The explanation includes the information that a Whipple procedure involves a. Creating a bypass around the obstruction caused by the tumor by joining the gallbladder to the jejunum. b. Resection of the entire pancreas and the distal portion of the stomach, with anastomosis of the common bile duct and the stomach into the duodenum. c. Removal of part of the pancreas, part of the stomach, the duodenum, and the gallbladder, with joining of the pancreatic duct, the common bile duct, and the stomach into the jejunum. d. Radical removal of the pancreas, the duodenum, and the spleen, and attachment of the stomach to the jejunum, which requires oral supplementation of pancreatic digestive enzymes and insulin replacement therapy.

c. Removal of part of the pancreas, part of the stomach, the duodenum, and the gallbladder, with joining of the pancreatic duct, the common bile duct, and the stomach into the jejunum.

During discharge instructions for a patient following a laparoscopic cholecystectomy, what should the nurse include in the teaching? a. Keep the incision areas clean and dry for at least a week. b. Report the need to take pain medication for shoulder pain. c. Report any bile-colored or purulent drainage from the incisions. d. Expect some postoperative nausea and vomiting for a few days.

c. Report any bile-colored or purulent drainage from the incisions. Bile-colored drainage or pus from any incision may indicate an infection and should be reported to the health care provider immediately. The bandages on the puncture sites should be removed the day after surgery, followed by bathing or showering. Referred shoulder pain is a common and expected problem following laparoscopic procedures, when carbon dioxide used to inflate the abdominal cavity is not readily absorbed by the body. Nausea and vomiting are not expected postoperatively and may indicate damage to other abdominal organs and should be reported to the health care provider.

Following bowel resection, a patient has a nasogastric (NG) tube to suction, but complains of nausea and abdominal distention. The nurse irrigates the tube as necessary as ordered, but the irrigating fluid does not return. What should be the priority action by the nurse? a. Notify the physician. b. Auscultate for bowel sounds. c. Reposition the tube and check for placement. d. Remove the tube and replace it with a new one.

c. Reposition the tube and check for placement.

A patient is hospitalized with metastatic cancer of the liver. The nurse plans care for the patient based on what knowledge? a. Chemotherapy is highly successful in the treatment of liver cancer. b. The patient will undergo surgery to remove the involved portions of the liver. c. Supportive care that is appropriate for all patients with severe liver damage is indicated. d. Metastatic cancer of the liver is more responsive to treatment than primary carcinoma of the liver

c. Supportive care that is appropriate for all patients with severe liver damage is indicated. Because the prognosis for cancer of the liver is poor and treatment is largely palliative, supportive nursing care is appropriate. The patient exhibits clinical manifestations of liver failure, as seen in any patient with advanced liver failure. Whether the cancer is primary or metastatic, there is usually a poor response to chemotherapy and surgery is indicated in the few patients that have localization of the tumor when there is no evidence of invasion of hepatic blood vessels.

Corticosteroid medications are associated with the development of peptic ulcers because of which probable pathophysiologic mechanism? a. The enzyme urease is produced. b. Secretion of hydrochloric acid is increased. c. The rate of mucous cell renewal is decreased. d. The synthesis of mucus and prostaglandins is inhibited.

c. The rate of mucous cell renewal is decreased. Corticosteroids decrease the rate of mucous cell renewal. H. pylori produces the enzyme urease. Alcohol ingestion increases the secretion of hydrochloric acid. Aspirin and NSAIDs inhibit the synthesis of mucus and prostaglandins.

The patient asks the nurse why she needs to have surgery for a femoral, strangulated hernia. What is the best explanation the nurse can give the patient? a. The surgery will relieve her constipation. b. The abnormal hernia must be replaced into the abdomen. c. The surgery is needed to allow intestinal flow and prevent necrosis. d. The hernia is because the umbilical opening did not close after birth as it should have.

c. The surgery is needed to allow intestinal flow and prevent necrosis. A strangulated femoral hernia obstructs intestinal flow and blood supply, thus requiring emergency surgery. The other options are incorrect.

The patient comes to the emergency department with intermittent crampy abdominal pain, nausea, projectile vomiting, and dehydration. The nurse suspects a GI obstruction. Based on the manifestations, what area of the bowel should the nurse suspect is obstructed? a. Large intestine b. Esophageal sphincter c. Upper small intestine d. Lower small intestine

c. Upper small intestine Intermittent crampy abdominal pain, nausea, projectile vomiting, and dehydration are characteristics of mechanical upper small intestinal obstruction. With continued vomiting, metabolic alkalosis may occur. Large bowel obstruction is characterized by constipation, low-grade abdominal pain, and abdominal distention. Fecal vomiting is seen with lower small intestinal obstruction.

What is the most important thing the nurse should do when caring for a patient who has contracted Clostridium difficile? a. Clean the entire room with ammonia. b. Feed the patient yogurt with probiotics. c. Wear gloves and wash hands with soap and water. d. Teach the family to use alcohol-based hand cleaners.

c. Wear gloves and wash hands with soap and water. Wearing gloves will avoid hand contamination and washing hands with soap and water will remove more Clostridium difficile spores than alcohol-based hand cleaners and ammonia-based disinfectants. The entire room will need to be disinfected with a 10% solution of household bleach. Probiotics may help to prevent diarrhea in the patient on antibiotics by replacing normal intestinal bacteria.

Which laboratory findings should the nurse expect in the patient with persistent vomiting? a. ↓ pH, ↑ sodium, ↓ hematocrit b. ↑ pH, ↓ chloride, ↓ hematocrit c. ↑ pH, ↓ potassium, ↑ hematocrit d. ↓ pH, ↓ potassium, ↑ hematocrit

c. ↑ pH, ↓ potassium, ↑ hematocrit The loss of gastric hydrochloric acid causes metabolic alkalosis and an increase in pH; loss of potassium, sodium, and chloride; and loss of fluid, which increases the hematocrit.

The family of a patient newly diagnosed with hepatitis A asks the nurse what they can do to prevent becoming ill themselves. Which response by the nurse is most appropriate? a. "The hepatitis vaccine will provide immunity from this exposure and future exposures." b. "I am afraid there is nothing you can do since the patient was infectious before admission." c. "You will need to be tested first to make sure you don't have the virus before we can treat you." d. "An injection of immunoglobulin will need to be given to prevent or minimize the effects from this exposure."

d. "An injection of immunoglobulin will need to be given to prevent or minimize the effects from this exposure."

A patient with cirrhosis asks the nurse about the possibility of a liver transplant. What is the best response by the nurse? a. "Liver transplants are indicated only in young people with irreversible liver disease." b. "If you are interested in a transplant, you really should talk to your doctor about it." c. "Rejection is such a problem in liver transplants that it is seldom attempted in patients with cirrhosis." d. "Cirrhosis is an indication for transplantation in some cases. Have you talked to your doctor about this?"

d. "Cirrhosis is an indication for transplantation in some cases. Have you talked to your doctor about this?" Liver transplantation is indicated for patients with cirrhosis as well as for many adults and children with other irreversible liver diseases. Although health care providers make the decisions regarding the patient's qualifications for transplantation, nurses should be knowledgeable about the indications for transplantation and be able to discuss the patient's questions and concerns related to transplantation. Rejection is less of a problem in liver transplants than with other organs such as the kidney.

At the first visit to the clinic, the female patient with a BMI of 29 kg/m2 tells the nurse that she does not want to become obese. Which question used for assessing weight issues is the most important question for the nurse to ask? a. "What factors contributed to your current body weight?" b. "How is your overall health affected by your body weight?" c. "What is your history of gaining weight and losing weight?" d. "In what ways are you interested in managing your weight differently?"

d. "In what ways are you interested in managing your weight differently?"

The severely obese patient has elected to have the Roux-en-Y gastric bypass (RYGB) procedure. The nurse will know the patient understands the preoperative teaching when the patient makes which statement? a. "This surgery will preserve the function of my stomach." b. "This surgery will remove the fat cells from my abdomen." c. "This surgery can be modified whenever I need it to be changed." d. "This surgery decreases how much I can eat and how many calories I can absorb."

d. "This surgery decreases how much I can eat and how many calories I can absorb."

Which patient is at highest risk for developing metabolic syndrome? a. A 62-year-old white man who has coronary artery disease with chronic stable angina b. A 54-year-old Hispanic woman who is sedentary and has nephrogenic diabetes insipidus c. A 27-year-old Asian American woman who has preeclampsia and gestational diabetes mellitus d. A 38-year-old Native American man who has diabetes mellitus and elevated hemoglobin A1C

d. A 38-year-old Native American man who has diabetes mellitus and elevated hemoglobin A1C

The nurse is reviewing the home medication list for a 44-year-old man admitted with suspected hepatic failure. Which medication could cause hepatotoxicity? a. Nitroglycerin b. Digoxin (Lanoxin) c. Ciprofloxacin (Cipro) d. Acetaminophen (Tylenol)

d. Acetaminophen (Tylenol)

What causes the systemic effects of viral hepatitis? a. Cholestasis b. Impaired portal circulation c. Toxins produced by the infected liver d. Activation of the complement system by antigen-antibody complexes

d. Activation of the complement system by antigen-antibody complexes The systemic manifestations of rash, angioedema, arthritis, fever, and malaise in viral hepatitis are caused by the activation of the complement system by circulating immune complexes. Liver manifestations include jaundice from hepatic cell damage and cholestasis as well as anorexia. Impaired portal circulation usually does not occur in uncomplicated viral hepatitis but would be a liver manifestation.

This bariatric surgical procedure involves creating a stoma and gastric pouch that is reversible, and no malabsorption occurs. What surgical procedure is this? a. Vertical gastric banding b. Biliopancreatic diversion c. Roux-en-Y gastric bypass d. Adjustable gastric banding

d. Adjustable gastric banding

Although HAV antigens are not tested in the blood, they stimulate specific immunoglobulin M (IgM) and immunoglobulin G (IgG) antibodies. Which antibody indicates there is acute HAV infection? a. Anti-HBc IgG b. Anti-HBc IgM c. Anti-HAV IgG d. Anti-HAV IgM

d. Anti-HAV IgM Anti-HAV immunoglobulin M (IgM) indicates acute HAV infection. Anti-HBc immunoglobulin G (IgG) indicates previous or ongoing infection with HBV. Anti- HBc IgM indicates acute HBV infection. Anti-HAV IgG indicates previous infection with HAV.

A nursing intervention that is most appropriate to decrease postoperative edema and pain after an inguinal herniorrhaphy is a. Applying a truss to the hernia site. b. Allowing the patient to stand to void. c. Supporting the incision during coughing. d. Applying a scrotal support with ice bag.

d. Applying a scrotal support with ice bag.

When caring for a patient with an acute exacerbation of a peptic ulcer, the nurse finds the patient doubled up in bed with shallow, grunting respirations. Which action should the nurse take first? a. Irrigate the patient's NG tube. b. Notify the health care provider. c. Place the patient in high-Fowler's position. d. Assess the patient's abdomen and vital signs.

d. Assess the patient's abdomen and vital signs. Abdominal pain that causes the knees to be drawn up and shallow, grunting respirations in a patient with peptic ulcer disease are characteristic of perforation and the nurse should assess the patient's vital signs and abdomen before notifying the health care provider. Irrigation of the NG tube should not be performed because the additional fluid may be spilled into the peritoneal cavity and the patient should be placed in a position of comfort, usually on the side with the head slightly elevated.

In instituting a bowel training program for a patient with fecal incontinence, what should the nurse first plan to do? a. Teach the patient to use a perianal pouch. b. Insert a rectal suppository at the same time every morning. c. Place the patient on a bedpan 30 minutes before breakfast. d. Assist the patient to the bathroom at the time of the patient's normal defecation.

d. Assist the patient to the bathroom at the time of the patient's normal defecation. The first intervention to establish bowel regularity includes promoting bowel evacuation at a regular time each day, preferably by placing the patient on the bedpan, using a bedside commode, or walking the patient to the bathroom. To take advantage of the gastrocolic reflex, an appropriate time is 30 minutes after the first meal of the day or at the patient's usual individual time. Perianal pouches are used to protect the skin only when regularity cannot be established and evacuation suppositories are also used only if other techniques are not successful.

Which conditions contribute to the formation of abdominal ascites? a. Esophageal varices contribute to 80% of variceal hemorrhages b. Increased colloidal oncotic pressure caused by decreased albumin production c. Hypoaldosteronism causes increased sodium reabsorption by the renal tubules d. Blood flow through the portal system is obstructed, which causes portal hypertension

d. Blood flow through the portal system is obstructed, which causes portal hypertension Blood flow through the portal system is obstructed and causes portal hypertension that increases the blood pressure in the portal venous system. Decreased albumin production leads to decreased serum colloidal oncotic pressure that contributes to ascites. Hyperaldosteronism increases sodium and water retention and contributes to increased fluid retention, hypokalemia, and decreased urinary output. The retained fluid has low oncotic colloidal pressure and it escapes into the interstitial spaces, causing peripheral edema. Portal hypertension also contributes to esophageal varices. Reduced renal blood flow and increased serum levels of antidiuretic hormone (ADH) also contribute to impaired water excretion and ascites.

The patient with suspected pancreatic cancer is having many diagnostic studies done. Which one can be used to establish the diagnosis of pancreatic adenocarcinoma and for monitoring the response to treatment? a. Spiral CT scan b. A PET/CT scan c. Abdominal ultrasound d. Cancer-associated antigen 19-9

d. Cancer-associated antigen 19-9

The nurse is preparing to administer a scheduled dose of docusate sodium (Colace) when the patient reports an episode of loose stool and does not want to take the medication. What is the appropriate action by the nurse? a. Write an incident report about this untoward event. b. Attempt to have the family convince the patient to take the ordered dose. c. Withhold the medication at this time and try to administer it later in the day. d. Chart the dose as not given on the medical record and explain in the nursing progress notes.

d. Chart the dose as not given on the medical record and explain in the nursing progress notes.

The health care provider orders lactulose for a patient with hepatic encephalopathy. The nurse will monitor for effectiveness of this medication for this patient by assessing what? a. Relief of constipation b. Relief of abdominal pain c. Decreased liver enzymes d. Decreased ammonia levels

d. Decreased ammonia levels

The nurse evaluates that management of the patient with upper GI bleeding is effective when assessment and laboratory findings reveal which result? a. Hematocrit (Hct) of 35% b. Urinary output of 20 mL/hr c. Urine specific gravity of 1.030 d. Decreasing blood urea nitrogen (BUN)

d. Decreasing blood urea nitrogen (BUN) The patient's blood urea nitrogen (BUN) is usually elevated with a significant hemorrhage because blood proteins are subjected to bacterial breakdown in the GI tract. With control of bleeding, the BUN will return to normal. During the early stage of bleeding, the hematocrit (Hct) is not always a reliable indicator of the amount of blood lost or the amount of blood replaced and may be falsely high or low. A urinary output of ≤20 mL/hr indicates impaired renal perfusion and hypovolemia and a urine specific gravity of 1.030 indicates concentrated urine typical of hypovolemia.

The nurse asks a 68-year-old patient scheduled for colectomy to sign the operative permit as directed in the physician's preoperative orders. The patient states that the physician has not really explained very well what is involved in the surgical procedure. What is the most appropriate action by the nurse? a. Ask family members whether they have discussed the surgical procedure with the physician. b. Have the patient sign the form and state the physician will visit to explain the procedure before surgery. c. Explain the planned surgical procedure as well as possible and have the patient sign the consent form. d. Delay the patient's signature on the consent and notify the physician about the conversation with the patient.

d. Delay the patient's signature on the consent and notify the physician about the conversation with the patient.

When teaching the patient about the diet for diverticular disease, which foods should the nurse recommend? a. White bread, cheese, and green beans b. Fresh tomatoes, pears, and corn flakes c. Oranges, baked potatoes, and raw carrots d. Dried beans, All Bran (100%) cereal, and raspberries

d. Dried beans, All Bran (100%) cereal, and raspberries

A nurse is completing discharge teaching to a client who has irritable bowel syndrome (IBS). Which of the following should be included in the teaching? a. Increase dietary intake of dairy products b. Consume 15 to 20 g of fiber daily c. Plan three moderate to large meals per day d. Drink at least 2 L of fluids each day

d. Drink at least 2 L of fluids each day

The patient experienced a blood transfusion reaction. How should the nurse explain to the patient the cause of the hemolytic jaundice that occurred? a. Results from hepatocellular disease b. Due to a malaria parasite breaking apart red blood cells (RBCs) c. Results from decreased flow of bile through the liver or biliary system d. Due to increased breakdown of RBCs that caused elevated serum unconjugated bilirubin

d. Due to increased breakdown of RBCs that caused elevated serum unconjugated bilirubin Hemolytic jaundice from a blood transfusion reaction is from increased breakdown of RBCs producing increased unconjugated bilirubin in the blood. Hepatocellular jaundice results from hepatocellular disease. Hemolytic jaundice occurs with malaria. Obstructive jaundice is from obstructed bile flow through the liver or biliary duct system.

In a radical pancreaticoduodenectomy (Whipple procedure) for treatment of cancer of the pancreas, what anatomic structure is completely resected that will affect the patient's nutritional status? a. Stomach b. Pancreas c. Common bile duct d. Duodenum adjoining the pancreas

d. Duodenum adjoining the pancreas In a Whipple procedure the head of the pancreas, gallbladder, part of the duodenum adjacent to the pancreas, and sometimes the pylorus of the stomach are removed. The duodenum is responsible for the breakdown of food in the small intestine and regulates the rate of stomach emptying, which affects the patient's nutritional status.

A nurse is completing an admission assessment of a client who has pancreatitis. Which of the following is an expected finding? a. Pain in the right upper quadrant radiating to right shoulder b. Report of pain being worse when sitting upright c. Pain relieved with defecation d. Epigastric pain radiating to left shoulder

d. Epigastric pain radiating to left shoulder

What extraintestinal manifestations are seen in both ulcerative colitis and Crohn's disease? a. Celiac disease and gallstones b. Peptic ulcer disease and uveitis c. Conjunctivitis and colonic dilation d. Erythema nodosum and osteoporosis

d. Erythema nodosum and osteoporosis Ulcerative colitis and Crohn's disease have many of the same extraintestinal symptoms, including erythema nodosum and osteoporosis, as well as gallstones, uveitis, and conjunctivitis. Colonic dilation and celiac disease are not extraintestinal.

A male patient who has undergone an anterior-posterior repair is worried about his sexuality. What is an appropriate nursing intervention for this patient? a. Have the patient's sexual partner reassure the patient that he is still desirable. b. Reassure the patient that sexual function will return when healing is complete. c. Remind the patient that affection can be expressed in ways other than through sexual intercourse. d. Explain that physical and emotional factors can affect sexual function but not necessarily the patient's sexuality.

d. Explain that physical and emotional factors can affect sexual function but not necessarily the patient's sexuality. Sexual dysfunction may result from an anterior-posterior repair but the nurse should discuss with the patient that different nerve pathways affect erection, ejaculation, and orgasm and that a dysfunction of one does not mean total sexual dysfunction and also that an alteration in sexual activity does not have to alter sexuality. Simple reassurance of desirability and ignoring concerns about sexual function do not help the patient to regain positive feelings of sexuality.

Which patient has the highest morbidity risk? a. Male 6 ft. 1 in. tall, BMI 29 kg/m2 b. Female 5 ft. 6 in. tall, weight 150 lb. c. Male with waist circumference 46 in. d. Female 5 ft. 10 in. tall, obesity Class III

d. Female 5 ft. 10 in. tall, obesity Class III

Following a needle stick, what is used as prophylaxis against HBV? a. Interferon b. HBV vaccine c. Hepatitis B immune globulin (HBIG) d. HBV vaccine and HBIG

d. HBV vaccine and HBIG HBV vaccine and hepatitis B immune globulin (HBIG) are used together prophylactically after a needle stick. Interferon is used to treat chronic HBV.

A nurse is completing the admission assessment of a client who has acute pancreatitis. Which of the following findings is the priority to be reported to the provider? a. History of cholelithiasis b. Serum amylase levels three times greater than the expected value c. Client report of severe pain radiating to the back that is rated at an "8" d. Hand spasms present when blood pressure is checked

d. Hand spasms present when blood pressure is checked

A 22-year-old patient calls the outpatient clinic complaining of nausea and vomiting and right lower abdominal pain. What should the nurse advise the patient to do? a. Use a heating pad to relax the muscles at the site of the pain. b. Drink at least 2 quarts of juice to replace the fluid lost in vomiting. c. Take a laxative to empty the bowel before examination at the clinic. d. Have the symptoms evaluated by a health care provider right away.

d. Have the symptoms evaluated by a health care provider right away. The patient is having symptoms of an acute abdomen and should be evaluated by a health care provider immediately. The patient's age, location of pain, and other symptoms are characteristic of appendicitis. Heat application and laxatives should not be used in patients with undiagnosed abdominal pain because they may cause perforation of the appendix or other inflammations. Fluids should not be taken until vomiting is controlled, nor should they be taken in the event that surgery may be performed.

Following a laparoscopic cholecystectomy, what should the nurse expect to be part of the plan of care? a. Return to work in 2 to 3 weeks b. Be hospitalized for 3 to 5 days postoperatively c. Have a T-tube placed in the common bile duct to provide bile drainage d. Have up to four small abdominal incisions covered with small dressings

d. Have up to four small abdominal incisions covered with small dressings The laparoscopic cholecystectomy requires one to four small abdominal incisions to visualize and remove the gallbladder and the patient has small dressings placed over these incisions. The patient with an incisional cholecystectomy is usually hospitalized for 2 to 3 days, whereas the laparoscopic procedure allows same-day or next-day discharge with return to work within 1 week. A T-tube is placed in the common bile duct after exploration of the duct during an incisional cholecystectomy.

The patient returned from a 6-week mission trip to Somalia with complaints of nausea, malaise, fatigue, and achy muscles. Which type of hepatitis is this patient most likely to have contracted? a. Hepatitis B (HBV) b. Hepatitis C (HCV) c. Hepatitis D (HDV) d. Hepatitis E (HEV)

d. Hepatitis E (HEV) Hepatitis E virus (HEV) is associated with poor sanitation and contaminated water in developing countries.

The nurse is caring for a 68-year-old patient admitted with abdominal pain, nausea, and vomiting. The patient has an abdominal mass, and a bowel obstruction is suspected. The nurse auscultating the abdomen listens for which type of bowel sounds that are consistent with the patient's clinical picture? a. Low-pitched and rumbling above the area of obstruction b. High-pitched and hypoactive below the area of obstruction c. Low-pitched and hyperactive below the area of obstruction d. High-pitched and hyperactive above the area of obstruction

d. High-pitched and hyperactive above the area of obstruction

A patient who has hepatitis B surface antigen (HBsAg) in the serum is being discharged with pain medication after knee surgery. Which medication order should the nurse question because it is most likely to cause hepatic complications? a. Tramadol (Ultram) b. Hydromorphone (Dilaudid) c. Oxycodone with aspirin (Percodan) d. Hydrocodone with acetaminophen (Vicodin)

d. Hydrocodone with acetaminophen (Vicodin)

The patient with a new ileostomy needs discharge teaching. What should the nurse plan to include in this teaching? a. The pouch can be worn for up to 2 weeks before changing it. b. Decrease the amount of fluid intake to decrease the amount of drainage. c. The pouch can be removed when bowel movements have been regulated. d. If leakage occurs, promptly remove the pouch, clean the skin, and apply a new pouch

d. If leakage occurs, promptly remove the pouch, clean the skin, and apply a new pouch The ileostomy drainage is extremely irritating to the skin, so the skin must be cleaned and a new solid skin barrier and pouch applied as soon as a leak occurs to prevent skin damage. The pouch is usually worn for 4 to 7 days unless there is a leak. Because the initial drainage from the ileostomy is high, the fluid intake must not be decreased. The pouch must always be worn, as the liquid drainage, not formed bowel movements, is frequent.

A nurse is providing teaching for a client who has a new diagnosis of dumping syndrome following gastric surgery. Which of the following should be included in the teaching? a. Eat three moderate-sized meals a day b. Drink at least one glass of water with each meal c. Eat a bedtime snack that contains a milk product d. Increase protein in the diet

d. Increase protein in the diet

The nurse is preparing to administer a dose of bisacodyl (Dulcolax). In explaining the medication to the patient, the nurse would explain that it acts in what way? a. Increases bulk in the stool b. Lubricates the intestinal tract to soften feces c. Increases fluid retention in the intestinal tract d. Increases peristalsis by stimulating nerves in the colon wall

d. Increases peristalsis by stimulating nerves in the colon wall

The patient asks why the serologic test of HBV DNA quantitation is being done. What is the best rationale for the nurse to explain the test to the patient? a. Indicates ongoing infection with HBV b. Indicates co-infection with HBV and HDV c. Indicates previous infection or immunization to HBV d. Indicates viral replication and effectiveness of therapy for chronic HBV

d. Indicates viral replication and effectiveness of therapy for chronic HBV HBV DNA quantitation is the best indicator of viral replication and effectiveness of therapy for chronic HBV. HBsAg is present in acute or chronic infection. HBeAg indicates high infectivity and can also be used to determine clinical management of patients with chronic HBV. Anti-HBcIgM indicates acute infection and anti- HBcIgG indicates ongoing infection. Anti-HDV is present in past or current infection with HDV and therefore HBV. Anti-HBs indicate previous infection with HBV or immunization.

When planning care for a patient with cirrhosis, the nurse will give highest priority to which nursing diagnosis? a. Impaired skin integrity related to edema, ascites, and pruritus b. Imbalanced nutrition: less than body requirements related to anorexia c. Excess fluid volume related to portal hypertension and hyperaldosteronism d. Ineffective breathing pattern related to pressure on diaphragm and reduced lung volume

d. Ineffective breathing pattern related to pressure on diaphragm and reduced lung volume

What manifestation in the patient does the nurse recognize as an early sign of hepatic encephalopathy? a. Manifests asterixis b. Becomes unconscious c. Has increasing oliguria d. Is irritable and lethargic

d. Is irritable and lethargic Early signs (grade 1) of this neurologic condition include changes in mentation (e.g., depression, apathy, irritability, confusion, agitation, drowsiness, lethargy). Loss of consciousness (grade 4) is usually preceded by asterixis (grades 2 and 3), disorientation, hyperventilation, hypothermia, and alterations in reflexes. Increasing oliguria is a sign of hepatorenal syndrome.

Ondansetron (Zofran) is prescribed for a patient with cancer chemotherapy-induced vomiting. What should the nurse understand about this drug? a. It is a derivative of cannabis and has a potential for abuse. b. It has a strong antihistamine effect that provides sedation and induces sleep. c. It is used only when other therapies are ineffective because of side effects of anxiety and hallucinations. d. It relieves vomiting centrally by action in the vomiting center and peripherally by promoting gastric emptying.

d. It relieves vomiting centrally by action in the vomiting center and peripherally by promoting gastric emptying. Ondansetron (Zofran) is one of several serotonin antagonists that act both centrally and peripherally to reduce vomiting: centrally on the vomiting center in the brainstem and peripherally by promoting gastric emptying. Dronabinol (Marinol) is an orally active cannabinoid that causes sedation and has a potential for abuse and it is used when other therapies are ineffective. Antihistamines used as antiemetics also cause sedation.

The nurse would question the use of which cathartic agent in a patient with renal insufficiency? a. Bisacodyl (Dulcolax) b. Lubiprostone (Amitiza) c. Cascara sagrada (Senekot) d. Magnesium hydroxide (Milk of Magnesia)

d. Magnesium hydroxide (Milk of Magnesia)

A severely obese patient has undergone Roux-en-Y gastric bypass surgery. In planning postoperative care, the nurse anticipates the patient a. May have severe diarrhea early in the postoperative period b. Will not be allowed to ambulate for 1 to 2 days postoperatively c. Will require nasogastric suction until the incision heals d. May have only liquids orally, and in very limited amounts, during the postoperative period

d. May have only liquids orally, and in very limited amounts, during the postoperative period

After an abdominal hysterectomy, a 45-year-old woman complains of severe gas pains. Her abdomen is distended. It is most appropriate for the nurse to administer which prescribed medication? a. Morphine sulfate b. Ondansetron (Zofran) c. Acetaminophen (Tylenol) d. Metoclopramide (Reglan)

d. Metoclopramide (Reglan)

The RN coordinating the care for a patient who is 2 days postoperative following an anterior- posterior resection with colostomy may delegate which interventions to the licensed practical nurse (LPN) (select all that apply)? a. Irrigate the colostomy. b. Teach ostomy and skin care. c. Assess and document stoma appearance. d. Monitor and record the volume, color, and odor of the drainage. e. Empty the ostomy bag and measure and record the amount of drainage.

d. Monitor and record the volume, color, and odor of the drainage. e. Empty the ostomy bag and measure and record the amount of drainage. The licensed practical nurse (LPN) can monitor and record observations related to the drainage and can measure and record the amount. The LPN could also monitor the skin around the stoma for breakdown. LPNs can irrigate a colostomy in a stable patient but this patient is only 2 days postoperative. The other actions are responsibilities of the RN (teaching, assessing stoma, and developing a care plan).

The patient with liver failure has had a liver transplant. What should the nurse teach the patient about care after the transplant? a. Alcohol intake is now okay. b. HBIG will be required to prevent rejection. c. Elevate the head 30 degrees to improve ventilation when sleeping. d. Monitor closely for infection because of the immunosuppressive medication.

d. Monitor closely for infection because of the immunosuppressive medication. Monitoring for viral, fungal, and bacterial infection after the liver transplant is essential, as only fever may be present with an infection. Alcohol will not be any better for the patient after the transplant than it was before the transplant. HBIG is given for postexposure protection from HBV. The head of the bed is elevated to improve ventilation with severe ascites.

Following a Billroth II procedure, a patient develops dumping syndrome. The nurse should explain that the symptoms associated with this problem are caused by a. Distention of the smaller stomach by too much food and fluid intake. b. Hyperglycemia caused by uncontrolled gastric emptying into the small intestine. c. Irritation of the stomach lining by reflux of bile salts because the pylorus has been removed. d. Movement of fluid into the small bowel because concentrated food and fluids move rapidly into the intestine.

d. Movement of fluid into the small bowel because concentrated food and fluids move rapidly into the intestine. Because there is no sphincter control of food taken into the stomach following a Billroth II procedure, concentrated food and fluid move rapidly into the small intestine, creating a hypertonic environment that pulls fluid from the bowel wall into the lumen of the intestine, reducing plasma volume and distending the bowel. Postprandial hypoglycemia occurs when the concentrated carbohydrate bolus in the small intestine results in hyperglycemia and the release of excessive amounts of insulin into the circulation, resulting in symptoms of hypoglycemia. Irritation of the stomach by bile salts causes epigastric distress after meals, not dumping syndrome.

While caring for a patient following a subtotal gastrectomy with a gastroduodenostomy anastomosis, the nurse determines that the NG tube is obstructed. Which action should the nurse take first? a. Replace the tube with a new one. b. Irrigate the tube until return can be aspirated. c. Reposition the tube and then attempt irrigation. d. Notify the surgeon to reposition or replace the tube.

d. Notify the surgeon to reposition or replace the tube. If the patient's NG tube becomes obstructed following a gastrectomy with an intestinal anastomosis, gastric secretions may put a strain on the sutured anastomosis and cause serious complications. Be sure that the suction is working and the health care provider may order periodic gentle irrigation with normal saline solution. Because of the danger of perforating the gastric mucosa or disrupting the suture line, the nurse should notify the health care provider if the tube needs to be repositioned or replaced.

A patient with upper GI bleeding is treated with several drugs. Which drug should the nurse recognize as an agent that is used to decrease bleeding and decrease gastric acid secretions? a. Nizatidine (Axid) b. Omeprazole (Prilosec) c. Vasopressin (Pitressin) d. Octreotide (Sandostatin)

d. Octreotide (Sandostatin) Octreotide is a somatostatin analog that has been shown to reduce upper GI bleeding and inhibit the release of GI hormones such as gastrin, thereby decreasing hydrochloric acid secretion. Nizatidine is a histamine (H2)-receptor blocker that decreases acid secretion and omeprazole inhibits the proton pump necessary for the secretion of hydrochloric acid. Vasopressin has a vasoconstriction action useful in controlling upper GI bleeding.

The nurse explains to the patient with gastroesophageal reflux disease (GERD) that this disorder a. Results in acid erosion of the esophagus from frequent vomiting. b. Will require surgical wrapping or repair of the pyloric sphincter to control the symptoms. c. Is the protrusion of a portion of the stomach into the esophagus through an opening in the diaphragm. d. Often involves relaxation of the lower esophageal sphincter, allowing stomach contents to back up into the esophagus.

d. Often involves relaxation of the lower esophageal sphincter, allowing stomach contents to back up into the esophagus.

In the immediate postoperative period a nurse cares for a severely obese 72-year-old man who had surgery for repair of a lower leg fracture. Which assessment would be most important for the nurse to make? a. Cardiac rhythm b. Surgical dressing c. Postoperative pain d. Oxygen saturation

d. Oxygen saturation

The ED nurse has inspected, auscultated, and palpated the abdomen with no obvious abnormalities, except pain. When the nurse palpates the abdomen for rebound tenderness, there is severe pain. The nurse should know that this could indicate what problem? a. Hepatic cirrhosis b. Hypersplenomegaly c. Gall bladder distention d. Peritoneal inflammation

d. Peritoneal inflammation

The patient presents with jaundice and itching, steatorrhea, and liver enlargement. This patient has also had ulcerative colitis for several years. What diagnosis should the nurse expect for this patient? a. Cirrhosis b. Acute liver failure c. Hepatorenal syndrome d. Primary sclerosing cholangitis

d. Primary sclerosing cholangitis The majority of patients with primary sclerosing cholangitis (PSC) also have ulcerative colitis. The manifestations are otherwise similar to cirrhosis and PSC may lead to cirrhosis, liver failure, and liver cancer

The nurse is teaching the patient and family that peptic ulcers are a. Caused by a stressful lifestyle and other acid-producing factors such as H. pylori. b. Inherited within families and reinforced by bacterial spread of Staphylococcus aureus in childhood. c. Promoted by factors that tend to cause oversecretion of acid, such as excess dietary fats, smoking, and H. pylori. d. Promoted by a combination of factors that may result in erosion of the gastric mucosa, including certain drugs and alcohol.

d. Promoted by a combination of factors that may result in erosion of the gastric mucosa, including certain drugs and alcohol.

What is a postoperative nursing intervention for the obese patient who has undergone bariatric surgery? a. Irrigating and repositioning the nasogastric (NG) tube as needed b. Delaying ambulation until the patient has enough strength to support self c. Keeping the patient positioned on the side to facilitate respiratory function d. Providing adequate support to the incision during coughing, deep breathing, and turning

d. Providing adequate support to the incision during coughing, deep breathing, and turning Turning, coughing and deep breathing are essential to prevent postoperative complications. Protecting the incision from strain is important since wound dehiscence is a problem for obese patients. If a nasogastric (NG) tube that is present following gastric surgery for severe obesity becomes blocked or needs repositioning, the health care provider should be notified. Ambulation is usually started on the evening of surgery and additional help will be needed to support the patient. Respiratory function is promoted by keeping the head of the bed elevated at an angle of 35 to 40 degrees.

What is the rationale for treating acute exacerbation of peptic ulcer disease with NG intubation? a. Stop spillage of GI contents into the peritoneal cavity b. Remove excess fluids and undigested food from the stomach c. Feed the patient the nutrients missing from the lack of ingestion d. Remove stimulation for hydrochloric acid and pepsin secretion by keeping the stomach empty

d. Remove stimulation for hydrochloric acid and pepsin secretion by keeping the stomach empty NG intubation is used with acute exacerbation of peptic ulcer disease to remove the stimulation for hydrochloric acid (HCl) and pepsin secretion by keeping the stomach empty. Stopping the spillage of GI contents into the peritoneal cavity is used for peritonitis. Removing excess fluids and undigested food from the stomach is the rationale for using NG intubation for gastric outlet obstruction.

The nurse is performing a focused abdominal assessment of a patient who has been recently admitted. In order to palpate the patient's liver, where should the nurse palpate the patient's abdomen? a. Left lower quadrant b. Left upper quadrant c. Right lower quadrant d. Right upper quadrant

d. Right upper quadrant

A 72-year-old patient was admitted with epigastric pain due to a gastric ulcer. Which patient assessment warrants an urgent change in the nursing plan of care? a. Chest pain relieved with eating or drinking water b. Back pain 3 or 4 hours after eating a meal c. Burning epigastric pain 90 minutes after breakfast d. Rigid abdomen and vomiting following indigestion

d. Rigid abdomen and vomiting following indigestion

In report, the nurse learns that the patient has a transverse colostomy. What should the nurse expect when providing care for this patient? a. Semiliquid stools with increased fluid requirements b. Liquid stools in a pouch and increased fluid requirements c. Formed stools with a pouch, needing irrigation, but no fluid needs d. Semiformed stools in a pouch with the need to monitor fluid balance

d. Semiformed stools in a pouch with the need to monitor fluid balance The patient with a transverse colostomy has semiliquid to semiformed stools needing a pouch and needs to have fluid balance monitored. The ascending colostomy has semiliquid stools needing a pouch and increased fluid. The ileostomy has liquid to semiliquid stools needing a pouch and increased fluid. The sigmoid colostomy has formed stools and may or may not need a pouch but will need irrigation.

A nurse is doing a nursing assessment on a patient with chronic constipation. What data obtained during the interview may be a factor contributing to the constipation? a. Taking methylcellulose (Citrucel) daily b. High dietary fiber with high fluid intake c. History of hemorrhoids and hypertension d. Suppressing the urge to defecate while at work

d. Suppressing the urge to defecate while at work Ignoring the urge to defecate causes the muscles and mucosa in the rectal area to become insensitive to the presence of feces and drying of the stool occurs. The urge to defecate is decreased and stool becomes more difficult to expel. Taking a bulk-forming agent with fluids or high fiber diet with fluids prevent constipation. Hemorrhoids are the most common complication of chronic constipation, caused by straining to pass hardened stool. The straining may cause problems in patients with hypertension but these do not cause constipation. Other things that may cause constipation are a history of diverticulosis, which is seen in individuals with low fiber intake, small stool mass, and hard stools. Chronic laxative use and chronic dilation and loss of colonic tone may also cause chronic constipation.

What is an accurate description of eosinophilic esophagitis? a. Adenocarcinoma or squamous cell tumors of the esophagus b. Dilated veins in the esophagus caused by portal hypertension c. Inflammation of the esophagus from irritants or gastric reflux d. Swelling of the esophagus caused by an allergic response to food or environmental triggers

d. Swelling of the esophagus caused by an allergic response to food or environmental triggers Eosinophilic esophagitis is swelling of the esophagus caused by infiltration of eosinophils in response to food triggers or environmental allergens. Esophageal cancer is usually caused by adenocarcinoma, with the remainder being squamous cell tumors. Esophageal varices are dilated vessels in the esophagus caused by portal hypertension. Esophagitis is inflammation of the esophagus commonly seen with GERD.

What should the nurse instruct the patient to do to best enhance the effectiveness of a daily dose of docusate sodium (Colace)? a. Take a dose of mineral oil at the same time. b. Add extra salt to food on at least one meal tray. c. Ensure dietary intake of 10 g of fiber each day. d. Take each dose with a full glass of water or other liquid.

d. Take each dose with a full glass of water or other liquid.

Following a hemorrhoidectomy, what should the nurse advise the patient to do? a. Use daily laxatives to facilitate bowel emptying. b. Use ice packs to the perineum to prevent swelling. c. Avoid having a bowel movement for several days until healing occurs. d. Take warm sitz baths several times a day to promote comfort and cleaning.

d. Take warm sitz baths several times a day to promote comfort and cleaning. Warm sitz baths provide comfort, healing, and cleansing of the area following all anorectal surgery and may be done three or four times a day for 1 to 2 weeks. Stool softeners may be prescribed for several days postoperatively to help keep stools soft for passage but laxatives may cause irritation and trauma to the anorectal area and are not used postoperatively. Early passage of a bowel movement, although painful, is encouraged to prevent drying and hardening of stool, which would result in an even more painful bowel movement.

A nurse is admitting a client who has bleeding esophageal varices. The nurse should anticipate a prescription for which of the following medications? a. Propanolol (Inderal) b. Metoclopramide (Reglan) c. Ranitidine (Zantac) d. Terlipressin (synthetic vasopressin)

d. Terlipressin (synthetic vasopressin)

The patient with a history of lung cancer and hepatitis C has developed liver failure and is considering liver transplantation. After the comprehensive evaluation, the nurse knows that which factor discovered may be a contraindication for liver transplantation? a. Has completed a college education b. Has been able to stop smoking cigarettes c. Has well-controlled type 1 diabetes mellitus d. The chest x-ray showed another lung cancer lesion.

d. The chest x-ray showed another lung cancer lesion.

A nurse is reviewing a new prescription for ursodiol (Ursodeoxycholic Acid) with a client who has cholelithiasis. Which of the following should be included in the teaching? a. This medication reduces biliary spasms b. This medication reduces inflammation in the biliary tract c. This medication dilates the bile duct to promote passage of bile d. This medication dissolves gall stones

d. This medication dissolves gall stones

The occurrence of acute liver failure is most common in which situation? a. An individual with hepatitis A b. An individual with hepatitis C c. Antihypertensive medication use d. Use of acetaminophen with alcohol abuse

d. Use of acetaminophen with alcohol abuse The most common cause of acute liver failure is drugs, usually acetaminophen in combination with alcohol abuse. HBV is the second most common cause.

An optimal teaching plan for an outpatient with stomach cancer receiving radiation therapy should include information about a. Cancer support groups, alopecia, and stomatitis. b. Avitaminosis, ostomy care, and community resources. c. Prosthetic devices, skin conductance, and grief counseling. d. Wound and skin care, nutrition, drugs, and community resources.

d. Wound and skin care, nutrition, drugs, and community resources.


Set pelajaran terkait

ACC211 Chapter 4 learning assignment

View Set

Introduction to Computer Concepts Final CHAPTER 12 STUDY SET

View Set

Mid term Intro to ECON (ECO 140-231) Summer class 2021

View Set

American Government Chapters 5-9 review

View Set

Poultry Nutrition Exam#2 Dr. Bailey

View Set

Genetics Final Price Summer 2015 UVU

View Set

AP Psych, Unit 1 prologue Vocabulary

View Set

Chapter 21: Transnational Actors

View Set